You are on page 1of 121

Probability

MARCH 26, 2024


REVISION: 2142

AZIZ MANVA
AZIZMANVA@GMAIL.COM

ALL RIGHTS RESERVED


Get all the files at: https://bit.ly/azizhandouts
Aziz Manva (azizmanva@gmail.com)

TABLE OF CONTENTS
1.7 Multiplication Rule-II: No Tree Diagrams 72
TABLE OF CONTENTS ................................. 2 1.8 Parity and Divisibility 84
1. BASICS ...................................................... 3 2. FURTHER TOPICS ................................. 95
1.1 Intuitive Probability 3 2.1 Dependent Outcomes 95
1.2 Sample Spaces and Equiprobable Outcomes 9 2.2 Bayes Theorem 107
1.3 Calculating Probabilities 18 2.3 Conditional Independence 121
1.4 Application: Number Theory 34 2.4 Further Topics 121
1.5 Addition Rule 47
1.6 Multiplication Rule-I: Tree Diagrams 61

P a g e 2 | 121
Get all the files at: https://bit.ly/azizhandouts
Aziz Manva (azizmanva@gmail.com)

1. BASICS
1.1 Intuitive Probability
A. Diagrams

1.1: Intuitive Definition of Probability


𝑆𝑢𝑐𝑐𝑒𝑠𝑠𝑓𝑢𝑙 𝑂𝑢𝑡𝑐𝑜𝑚𝑒
𝑃𝑟𝑜𝑏𝑎𝑏𝑖𝑙𝑖𝑡𝑦 =
𝑇𝑜𝑡𝑎𝑙 𝑂𝑢𝑡𝑐𝑜𝑚𝑒𝑠

➢ This formula assumes that all outcomes are equally likely.


➢ We will address the issue of outcomes not being equally likely later.

Example 1.2 3
Consider the table to the right. If you select one of the cells in the grid shown at
random, what is the probability that the cell contains: F Q Z
A. A square 7
B. A circle
C. A red object a 10
D. A blue object
E. A letter A b
F. A number
G. A capital letter
H. A vowel
I. A consonant
J. A capital vowel
K. A capital consonant
L. A blank space
M. Number greater than 100

There are 25 cells in all.


7
𝑆𝑞𝑢𝑎𝑟𝑒:
25
7
𝐶𝑖𝑟𝑐𝑙𝑒:
25
8
𝑅𝑒𝑑 𝑂𝑏𝑗𝑒𝑐𝑡:
25
6
𝐵𝑙𝑢𝑒 𝑂𝑏𝑗𝑒𝑐𝑡:
25
6
𝐿𝑒𝑡𝑡𝑒𝑟:
25
2
𝑁𝑢𝑚𝑏𝑒𝑟:
25
4
𝐶𝑎𝑝𝑖𝑡𝑎𝑙 𝐿𝑒𝑡𝑡𝑒𝑟:
25
2
𝑉𝑜𝑤𝑒𝑙:
25
4
𝐶𝑜𝑛𝑠𝑜𝑛𝑎𝑛𝑡:
25
1
𝐶𝑎𝑝𝑖𝑡𝑎𝑙 𝑉𝑜𝑤𝑒𝑙:
25

P a g e 3 | 121
Get all the files at: https://bit.ly/azizhandouts
Aziz Manva (azizmanva@gmail.com)

3
𝐶𝑎𝑝𝑖𝑡𝑎𝑙 𝐶𝑜𝑛𝑠𝑜𝑛𝑎𝑛𝑡:
25
2
𝐵𝑙𝑎𝑛𝑘 𝑆𝑝𝑎𝑐𝑒:
25
0
𝑁𝑢𝑚𝑏𝑒𝑟 𝑔𝑟𝑒𝑎𝑡𝑒𝑟 𝑡ℎ𝑎𝑛 25: =0
25

If the letter “Q” is replaced with the letter “e”, what is the probability of getting:
A capital vowel

B. Days of the Week

Example 1.3
A week has seven days, of which Saturday and Sunday are considered to be on the weekend, and the other days
are considered weekdays.
What is the probability that if a random day is selected, it is a:
A. Weekday
B. Weekend
C. Not a weekday
D. Not a weekend

There are seven days in a week.


5
𝑊𝑒𝑒𝑘𝑑𝑎𝑦:
7
2
𝑊𝑒𝑒𝑘𝑒𝑛𝑑 =
7
5 2
𝑁𝑜𝑡 𝑎 𝑊𝑒𝑒𝑘𝑑𝑎𝑦 = 1 − =
7 7
2 5
𝑁𝑜𝑡 𝑎 𝑊𝑒𝑒𝑘𝑒𝑛𝑑 = 1 − =
7 7

What is the probability, if a random day is selected, that its name begins with a T?
The days of the week are:
𝑀𝑜𝑛𝑑𝑎𝑦, 𝑇𝑢𝑒𝑠𝑑𝑎𝑦, 𝑊𝑒𝑑𝑛𝑒𝑠𝑑𝑎𝑦, 𝑇ℎ𝑢𝑟𝑠𝑑𝑎𝑦, 𝐹𝑟𝑖𝑑𝑎𝑦, 𝑆𝑎𝑡𝑢𝑟𝑑𝑎𝑦, 𝑆𝑢𝑛𝑑𝑎𝑦
2
7
C. Calendar

Example 1.4
The table shows the number of days that the months of the year
Jan 31 May 31 Sep 30
have in a non-leap year. If I pick a random month of the year in a Feb 28 June 30 Oct 31
non-leap year, what is the probability that it has exactly: Mar 31 July 31 Nov 30
A. 28 days Apr 30 Aug 31 Dec 31
B. 29 days
C. 30 days
D. 31 days

P a g e 4 | 121
Get all the files at: https://bit.ly/azizhandouts
Aziz Manva (azizmanva@gmail.com)

1
28 𝐷𝑎𝑦𝑠:
12
0
29 𝐷𝑎𝑦𝑠: =0
12
4 1
30 𝐷𝑎𝑦𝑠: =
12 3
7
31 𝐷𝑎𝑦𝑠:
12
D. Numbers

1.5: Simplifying Probabilities


Probabilities should be expressed as a simplified fraction, as far as possible.

Example 1.6
Charlotte has a bag with a set of twenty wooden blocks. The blocks have the numbers from one to twenty
written on them. If a block is drawn from the bag without looking, and the number written on the block is
recorded, what is the probability that:
A. The number is prime
B. The number is composite
C. The number is odd
D. The number is even

8 2
2,3,5,7,11,13,17,19 ⇒ 8 𝑁𝑢𝑚𝑏𝑒𝑟𝑠 ⇒ 𝑃 = =
20 5
11
20 − 8 − 1 = 11 𝑁𝑢𝑚𝑏𝑒𝑟𝑠 ⇒ 𝑃 =
20
10 1
1,3,5,7,9,11,13,15,17,19 ⇒ 10 𝑁𝑢𝑚𝑏𝑒𝑟𝑠 ⇒ =
20 2
1
𝐸𝑣𝑒𝑛:
2
E. Tossing Coins

1.7: Fair Coin


If, when we toss a coin, the probability of heads and tails is the same, the coin is a fair coin.

➢ If we are not told, we cannot assume that a coin is fair.


➢ If a coin is not fair, it is an unfair or weighted coin.

Example 1.8
What is the probability that we get:
A. A head when we toss a fair coin
B. A tail when we toss a coin.

1
2
Can’t say unless we know whether the coin is fair.

P a g e 5 | 121
Get all the files at: https://bit.ly/azizhandouts
Aziz Manva (azizmanva@gmail.com)

F. Rolling Dice

1.9: Die
A standard six-sided die has the outcomes
{1,2,3,4,5,6}

Example 1.10: Rolling a single Dice


If you roll a fair six-sided die numbered from 1 to 6, what is the probability of getting:
A. A three
B. A three or a four
C. An even number
D. A prime number
E. A composite number
F. An integer less than zero.
G. An odd number less than 3.
H. A number greater than 4.

The possible outcomes are:

The probabilities:
1
6
2 1
=
6 3
3 1
=
6 2
3 1
=
6 2
2 1
=
6 3
0
1
6
2
6
G. Drawing from a Pack of Cards

Example 1.11
When drawing a single card at random from a pack of cards, what is the probability of getting:
A. A club
B. A queen
C. A red king

13 1
=
52 4
4 1
=
52 13

P a g e 6 | 121
Get all the files at: https://bit.ly/azizhandouts
Aziz Manva (azizmanva@gmail.com)

2 1
=
52 26
H. Spinning a Spinner

1.12: Spinner
You can think of a standard spinner as equivalent to a die.

Example 1.13: Spinning a Spinner


A spinner has the numbers {1,2,3,4,5,7,8} on it. If you spin the spinner, what is the probability of getting:
A. A multiple of 3
B. A multiple of 5
C. An odd number
D. An even number
E. A prime number
F. A composite number

1
{3} ⇒
7
1
{5} ⇒
7
4
{1,3,5,7} ⇒
7
3
{2,4,8} ⇒
7
4
{2,3,5,7} ⇒
7
2
{4,8} ⇒
7
I. Back Calculations

Example 1.14
1
A. I have an urn with ten balls in it. If the probability of picking a green ball is 2, what is the number of
green balls in the urn?
B. I have an urn with ten balls in it, which are either green or yellow. If the probability of picking a green
2
ball is , what is the number of yellow balls in the urn?
5
C. I have an urn with twenty balls in it, which are one of the following colours: green, red, black, or yellow.
The probability of picking a red or a black ball is 25%. The number of red balls is four. What is the
number of black balls?

Part A
1
𝑁𝑜. 𝑜𝑓 𝐺𝑟𝑒𝑒𝑛 𝐵𝑎𝑙𝑙𝑠 = × 10 = 5
2
Part B
2 3
𝑃(𝑌𝑒𝑙𝑙𝑜𝑤) = 1 − 𝑃(𝐺𝑟𝑒𝑒𝑛) = 1 − =
5 5
3
𝑁𝑜. 𝑜𝑓 𝑌𝑒𝑙𝑙𝑜𝑤 𝐵𝑎𝑙𝑙𝑠 = × 10 = 6
5
Part C
𝑇𝑜𝑡𝑎𝑙 𝑁𝑢𝑚𝑏𝑒𝑟 𝑜𝑓 𝐵𝑎𝑙𝑙𝑠 = 20

P a g e 7 | 121
Get all the files at: https://bit.ly/azizhandouts
Aziz Manva (azizmanva@gmail.com)

1 1
𝑃(𝑅𝑒𝑑 𝑜𝑟 𝐵𝑙𝑎𝑐𝑘) = 25% = ⇒ 𝑁𝑜. 𝑜𝑓 𝑅𝑒𝑑 𝑜𝑟 𝐵𝑙𝑎𝑐𝑘 𝐵𝑎𝑙𝑙𝑠 = 𝑡ℎ 𝑜𝑓 20 = 5
4 4

Out of the 5 red or black balls, the number of red balls is four.
∴ 𝑁𝑜. 𝑜𝑓 𝐵𝑙𝑎𝑐𝑘 𝐵𝑎𝑙𝑙𝑠 = 5 − 4 = 1

Example 1.15
Answer the questions below. If the question cannot be answered with the given information, write cannot be
determined.

I have an urn with sixty balls in it, which are one of the following colours: green, red, black, or yellow. The
1 1
probability of picking a green ball is 2. The probability of picking a red ball is 4.
A. What is the probability of picking a green or a red ball?
B. What is the probability of picking a ball which is either black or yellow?
C. Find the number of
a) Green balls
b) Red balls
c) Black balls
d) Yellow balls
e) Black and yellow balls together

Part A
1 1 3
𝑃(𝐺𝑟𝑒𝑒𝑛 𝑜𝑟 𝑅𝑒𝑑) = 𝑃(𝐺𝑟𝑒𝑒𝑛) + 𝑃(𝑅𝑒𝑑) = + =
2 4 4
Part B
3 1
𝑃(𝐵𝑙𝑎𝑐𝑘 𝑜𝑟 𝑌𝑒𝑙𝑙𝑜𝑤) = 1 − 𝑃(𝐺𝑟𝑒𝑒𝑛 𝑜𝑟 𝑅𝑒𝑑) = 1 − =
4 4
Part C
1 1
𝐺𝑟𝑒𝑒𝑛 = 𝑜𝑓 60 = 30, 𝑅𝑒𝑑 = 𝑡ℎ 𝑜𝑓 60 = 15
2 4
We can find the number of balls which are either black or yellow, which is
1
𝑡ℎ 𝑜𝑓 60 = 15
4
However, within this, we do not know how many are black and how many are yellow, since we do not have
enough information to answer the question.

J. Ratios and Odds

1.16: Odds
𝐹𝑎𝑣𝑜𝑢𝑟𝑎𝑏𝑙𝑒 𝑂𝑢𝑡𝑐𝑜𝑚𝑒𝑠
𝑂𝑑𝑑𝑠 =
𝑈𝑛𝑓𝑎𝑣𝑜𝑢𝑟𝑎𝑏𝑙𝑒 𝑂𝑢𝑡𝑐𝑜𝑚𝑒𝑠

Example 1.17
11
An urn has blue and green balls in it. The probability of picking a blue ball is 17.
A. Find the ratio of blue balls to green balls.
B. Find the odds of picking a blue ball.
C. Find the odds of picking a green ball.

P a g e 8 | 121
Get all the files at: https://bit.ly/azizhandouts
Aziz Manva (azizmanva@gmail.com)

Part A
𝑏 11 𝑔 𝑏 11 6
= ⇒ =1− =1− =
𝑡 17 𝑡 𝑡 17 17
𝑏 𝑔 𝑏 𝑡 𝑏
Use the property that ÷ = × =
𝑡 𝑡 𝑡 𝑔 𝑔
11 6 11 17 11
÷ = × =
17 17 17 6 6
Part B
𝐹𝑎𝑣𝑜𝑢𝑟𝑎𝑏𝑙𝑒 𝑂𝑢𝑡𝑐𝑜𝑚𝑒𝑠 11
𝑂𝑑𝑑𝑠 = =
𝑈𝑛𝑓𝑎𝑣𝑜𝑢𝑟𝑎𝑏𝑙𝑒 𝑂𝑢𝑡𝑐𝑜𝑚𝑒𝑠 6
Part C
𝐹𝑎𝑣𝑜𝑢𝑟𝑎𝑏𝑙𝑒 𝑂𝑢𝑡𝑐𝑜𝑚𝑒𝑠 6
𝑂𝑑𝑑𝑠 = =
𝑈𝑛𝑓𝑎𝑣𝑜𝑢𝑟𝑎𝑏𝑙𝑒 𝑂𝑢𝑡𝑐𝑜𝑚𝑒𝑠 11

Challenge 1.18
3
An urn has blue and yellow balls. In an urn, the probability of picking a blue ball is . Anand adds his favorite
5
7
ball to the urn, and now the probability of picking a blue ball becomes . Find the number of balls in the urn
11
after Anand added his blue ball.

Compare the two probabilities. Since: When you add one blue ball, the probability
3 7 becomes:
<
5 11 𝑏+1 7
The probability of getting a blue ball has increased, =
𝑡 + 1 11
and hence, the ball which was added was blue. 11𝑏 + 11 = 7𝑡 + 7
3
Substitute 𝑏 = 5 𝑡
Method I 3
We can do this with trial and error 11 ( 𝑡) + 4 = 7𝑡
5
3+1 4 2 35𝑡 33𝑡
= = − =4
5+1 6 3 5 5
6+1 7 2𝑡
= =4
10 + 1 11 5
Method II 𝑡 = 10
𝑏 3 3
= ⇒𝑏= 𝑡 𝑡 + 1 = 11
𝑡 5 5

1.2 Sample Spaces and Equiprobable Outcomes


A. Basics

1.19: Experiments
A procedure that has a set of well-defined outcomes is called an experiment.

Well defined is important. If the experiment is not well-defined, you will find it difficult to apply the laws of
probability to it.

1.20: Random Experiment


Experiments whose outcomes cannot be predicted when repeated under the same conditions are called random
P a g e 9 | 121
Get all the files at: https://bit.ly/azizhandouts
Aziz Manva (azizmanva@gmail.com)

experiments.

Examples of Random Experiments:


➢ Rolling a standard six-sided fair die
➢ Tossing a fair coin
➢ Tossing a weighted coins
➢ Drawing a card from a well-shuffled standard pack of cards
➢ Picking a ball from an urn

1.21: Sample Space


The sample space is the set of all possible outcomes from a random experiment.

B. Listing Sample Spaces


One way to analyze a sample space is to list the outcomes using enumeration.

1.22: Listing Sample Spaces


Sample spaces can be listed using a set in Roster Format:
{1,2,3, … ,10}

Example 1.23: Tossing Coins


Enumerate the sample space for each situation below.
A. Tossing a coin
B. Tossing a coin twice
C. Tossing two identical coins
D. Tossing a coin thrice
E. Tossing a coin 𝑛 times (Count number of outcomes, do not enumerate)

Part A
We can write the answer in set notation, listing out the outcomes in curly braces:
{H, T}
Part B
{𝐻𝐻, 𝐻𝑇, 𝑇𝐻, 𝑇𝑇}
Part C

{𝐻𝐻, 𝐻𝑇, 𝑇𝐻, 𝑇𝑇}


Observe that the answers to Part B and Part C are the same. Tossing a coin is the same (in terms of probability)
as tossing two identical coins.
Part D
{𝐻𝐻𝐻, 𝐻𝐻𝑇, 𝐻𝑇𝐻, 𝐻𝑇𝑇, 𝑇𝐻𝐻, 𝑇𝐻𝑇, 𝑇𝑇𝐻, 𝑇𝑇𝑇}

You count the number of outcomes directly using the multiplication principle with repetition:

2 × ⏟ 2 × ⏟ 2 = 23 = 8
𝐹𝑖𝑟𝑠𝑡 𝑆𝑒𝑐𝑜𝑛𝑑 𝑇ℎ𝑖𝑟𝑑
𝐶𝑜𝑖𝑛 𝐶𝑜𝑖𝑛 𝐶𝑜𝑖𝑛
Part E

2 × ⏟
2 2 = 2𝑛
× …× ⏟
𝐹𝑖𝑟𝑠𝑡 𝑆𝑒𝑐𝑜𝑛𝑑 𝑛𝑡ℎ
𝐶𝑜𝑖𝑛 𝐶𝑜𝑖𝑛 𝐶𝑜𝑖𝑛

(Rolling Dice) Example 1.24


P a g e 10 | 121
Get all the files at: https://bit.ly/azizhandouts
Aziz Manva (azizmanva@gmail.com)

Enumerate the sample space for each situation below.


A. Rolling a standard six-sided die once
B. Rolling a standard ten-sided die once
C. Rolling a six sided die with the vowels of the English alphabet (Consider 𝑌 as a vowel)

{1,2,3,4,5,6}
{1,2,3, … ,10}
{𝐴, 𝐸, 𝐼, 𝑂, 𝑈, 𝑌}

Example 1.25
There are fifteen balls in an urn, numbered 1 to 15. Write the sample space when drawing a ball from the urn.

{1,2,3, … ,15}

Example 1.26
There are eight yellow balls in an urn, numbered 1 to 8, and seven blue balls, numbered 1 to 7. Consider the two
ways of writing the sample when drawing a ball from the urn. Explain which one is better and why?
{𝑦𝑒𝑙𝑙𝑜𝑤, 𝑏𝑙𝑢𝑒}
{𝑏1 , 𝑏2 , … , 𝑏7 , 𝑦1 , 𝑦2 , … , 𝑦8 }

8
𝑃(𝑌𝑒𝑙𝑙𝑜𝑤) =
15
7
𝑃(𝐵𝑙𝑢𝑒) =
15
While the above is correct, the outcomes are not equiprobable.
We can write equiprobable outcomes as follows:

Example 1.27
A circular spinner is divided into seven sectors, each of which has equal area. The sectors are numbered with
the first seven odd prime numbers. Write the sample space when the spinner is spun.

{3,5,7,11,13,17,19}
C. Number Lines

1.28: Sample Spaces on a Number Line


We can represent a sample space on a number line.

(Natural Numbers) Example 1.29


A bag contains five balls, numbered 1 to 5. Represent the sample space of the outcomes when you draw a single
ball using:
A. Set Notation in Roster Form
B. Set Notation in Set Builder Form
C. A Number Line

Set Notation in Roster Form:


{1,2,3, … ,5}

P a g e 11 | 121
Get all the files at: https://bit.ly/azizhandouts
Aziz Manva (azizmanva@gmail.com)

Set Notation in Set Builder Form:


{𝑥: 𝑥 ≤ 5, 𝑥 ∈ ℕ}
Which is read as:
𝑥 𝑠𝑢𝑐ℎ 𝑡ℎ𝑎𝑡 𝑥 𝑖𝑠 𝑙𝑒𝑠𝑠 𝑡ℎ𝑎𝑛 𝑜𝑟 𝑒𝑞𝑢𝑎𝑙 𝑡𝑜 5, 𝑥 𝑖𝑠 𝑎 𝑛𝑎𝑡𝑢𝑟𝑎𝑙 𝑛𝑢𝑚𝑏𝑒𝑟

D. Grids

Example 1.30
Enumerate the sample space when tossing a coin.

We can list the sample space using a set


{𝐻, 𝑇}
We can also list it on a number line (drawn alongside), with Heads and Tails.

Example 1.31
Enumerate the sample space when tossing a coin twice.

To represent it on a grid, we need one more number line. Let the


➢ 𝑥 − 𝑎𝑥𝑖𝑠 represent the first coin toss
➢ 𝑦 − 𝑎𝑥𝑖𝑠 represent the second coin toss

𝑅𝑒𝑑 𝐷𝑜𝑡 = (1,1) = 𝐻𝐻


𝐵𝑙𝑢𝑒 𝐷𝑜𝑡 = (2,1) = 𝑇𝐻
𝐺𝑟𝑒𝑒𝑛 𝐷𝑜𝑡 = (1,2) = 𝐻𝑇
𝑃𝑖𝑛𝑘 𝐷𝑜𝑡 = (2,2) = 𝑇𝑇

Example 1.32
Explain why it would be possible (but difficult) to visualize three coins being tossed using a grid.

For one coin, we needed one number line. For two coins, we needed two number lines, and hence two
dimensions.

For coins, we would need three number line in three dimensions, which can be done but is difficult to draw, or
to visualize.

Example 1.33
Rolling Two Dice

P a g e 12 | 121
Get all the files at: https://bit.ly/azizhandouts
Aziz Manva (azizmanva@gmail.com)

You can represent rolling two dice on a grid as drawn in the diagram
alongside.

Example 1.34
Toss a coin, and spin a spinner with the numbers from 1 to 4.

E. Tables: Rolling Two Dice

Example 1.35
You roll two standard six-sided dice. What is the sample space of outcomes?

We can represent this using a table.


First Die
Each cell has the number on the die as an ordered
1 2 3 4 5 6
pair: Second 1 (1,1) (2,1) (3,1) (4,1) (5,1) (6,1)
(1𝑠𝑡 𝐷𝑖𝑒, 2𝑛𝑑 𝐷𝑖𝑒) Die 2 (1,2) (2,2) (3,2) (4,2) (5,2) (6,2)
3 (1,3) (2,3) (3,3) (4,3) (5,3) (6,3)
4 (1,4) (2,4) (3,4) (4,4) (5,4) (6,4)
5 (1,5) (2,5) (3,5) (4,5) (5,5) (6,5)
6 (1,6) (2,6) (3,6) (4,6) (5,6) (6,6)

Example 1.36
You roll two standard six-sided dice and find the sum of the numbers on the tops of the dice. What is the sample
space of outcomes?

Total
1 2 3 4 5 6
1 2 3 4 5 6 7
2 3 4 5 6 7 8
3 4 5 6 7 8 9
4 5 6 7 8 9 10
5 6 7 8 9 10 11
6 7 8 9 10 11 12

Example 1.37
You roll two standard six-sided dice and find the product of the numbers on the tops of the dice. What is the
sample space of outcomes?

Total

P a g e 13 | 121
Get all the files at: https://bit.ly/azizhandouts
Aziz Manva (azizmanva@gmail.com)

1 2 3 4 5 6
1 1 2 3 4 5 6
2 2 4 6 8 10 12
3 3 6 9 12 15 18
4 4 8 12 16 20 24
5 5 10 15 20 25 30
6 6 12 18 24 30 36

Example 1.38
You roll two standard tetrahedral dice (four-sided dice). What is the sample space of:
A. Outcomes
B. Sum of Outcomes
C. Product of Outcomes

Part A
First Die
1 2 3 4
Second 1 (1,1) (2,1) (3,1) (4,1)
Die 2 (1,2) (2,2) (3,2) (4,2)
3 (1,3) (2,3) (3,3) (4,3)
4 (1,4) (2,4) (3,4) (4,4)
Part B

First Die
1 2 3 4
Second 1 2 3 4 5
Die 2 3 4 5 6
3 4 5 6 7
4 5 6 7 8

Part C
First Die
1 2 3 4
Second 1 1 2 3 4
Die 2 2 4 6 8
3 3 6 9 12
4 4 8 12 16

F. Tables: Drawing a Card

Example 1.39
A standard pack of playing cards has 52 cards divided into 4 suits:
➢ Clubs
➢ Spades
➢ Hearts
➢ Diamonds

P a g e 14 | 121
Get all the files at: https://bit.ly/azizhandouts
Aziz Manva (azizmanva@gmail.com)

Cards are of two types:


➢ Black: Clubs and Spades
➢ Red: Hearts and Diamonds

Each suit has 13 cards:


➢ Numbered Cards: Ace, 2, 3, 4, 5, 6, 7, 8, 9, 10
➢ Face Cards: Jack, Queen and King
Note: In many problems, Ace is considered as a Card with number 1. However, you should rely on instructions
from the problem for final decision.

Ace 2 3 4 5 6 7 8 9 10 Jack Queen King Total


Clubs Face Face Face 13
Card Card Card
Spades Face Face Face 13
Card Card Card
Hearts Face Face Face 13
Card Card Card
Diamonds Face Face Face 13
Card Card Card
4 4 4 4 4 4 4 4 4 4 4 4 4 52

G. Equiprobable Outcomes
Outcomes have great importance in probability.

1.40: Equiprobable Outcomes


Equiprobable outcomes are those that have an equal chance of happening.

Example 1.41
Classify the following as equiprobable or non-equiprobable.
A. {𝐻, 𝑇} when tossing a fair coin
B. {𝐻, 𝑇} when tossing an unfair coin
C. {1, 2, 3, 4, 5, 6} when rolling a fair six-sided die
D. {1, 2, 3, 4, 5, 6} when rolling an unfair six-sided die

E. {𝐻𝑒𝑎𝑟𝑡𝑠, 𝐶𝑙𝑢𝑏𝑠, ⏟
𝑆𝑝𝑎𝑑𝑒𝑠 𝑜𝑟 𝐷𝑖𝑎𝑚𝑜𝑛𝑑𝑠} when drawing a card from a well-shuffled standard pack of
𝑇𝑜𝑔𝑒𝑡ℎ𝑒𝑟
cards
F. {𝑅𝑒𝑑 𝐶𝑎𝑟𝑑, 𝐵𝑙𝑎𝑐𝑘 𝐶𝑎𝑟𝑑} when drawing a card from a well-shuffled standard pack of cards
G. {𝐹𝑎𝑐𝑒 𝐶𝑎𝑟𝑑, 𝑁𝑜𝑛 𝐹𝑎𝑐𝑒 𝐶𝑎𝑟𝑑} when drawing a card from a well-shuffled standard pack of cards
H. {𝑃𝑟𝑖𝑚𝑒 𝑁𝑢𝑚𝑏𝑒𝑟, 𝐶𝑜𝑚𝑝𝑜𝑠𝑖𝑡𝑒 𝑁𝑢𝑚𝑏𝑒𝑟} when rolling a fair six-sided die
I. {𝑂𝑑𝑑 𝑁𝑢𝑚𝑏𝑒𝑟, 𝐸𝑣𝑒𝑛 𝑁𝑢𝑚𝑏𝑒𝑟} when rolling a fair six-sided die

𝐴: 𝐸𝑞𝑢𝑖𝑝𝑟𝑜𝑏𝑎𝑏𝑙𝑒
𝐵: 𝑁𝑜𝑡 𝐸𝑞𝑢𝑖𝑝𝑟𝑜𝑏𝑎𝑏𝑙𝑒
𝐶: 𝐸𝑞𝑢𝑖𝑝𝑟𝑜𝑏𝑎𝑏𝑙𝑒
𝐷: 𝑁𝑜𝑡 𝐸𝑞𝑢𝑖𝑝𝑟𝑜𝑏𝑎𝑏𝑙𝑒
1 1 1
𝑃(𝐻) = , 𝑃(𝐶) = , 𝑃( 𝑆𝑝𝑎𝑑𝑒𝑠 𝑜𝑟 𝐷𝑖𝑎𝑚𝑜𝑛𝑑𝑠) = ⇒ 𝑁𝑜𝑛 − 𝑒𝑞𝑢𝑖𝑝𝑟𝑜𝑏𝑎𝑏𝑙𝑒
4 4 2
26 1
𝑃(𝑅𝑒𝑑) = 𝑃(𝐵𝑙𝑎𝑐𝑘) = = ⇒ 𝐸𝑞𝑢𝑖𝑝𝑟𝑜𝑏𝑎𝑏𝑙𝑒
52 2

P a g e 15 | 121
Get all the files at: https://bit.ly/azizhandouts
Aziz Manva (azizmanva@gmail.com)

12 3 10
𝑃(𝐹𝑎𝑐𝑒 𝐶𝑎𝑟𝑑) = = , 𝑃(𝑁𝑜𝑛 − 𝐹𝑎𝑐𝑒 − 𝐶𝑎𝑟𝑑 ) = ⇒ 𝑁𝑜𝑛 − 𝑒𝑞𝑢𝑖𝑝𝑟𝑜𝑏𝑎𝑏𝑙𝑒
52 12 13
3 2
𝑃(𝑃𝑟𝑖𝑚𝑒) = , 𝑃(𝐶𝑜𝑚𝑝𝑜𝑠𝑖𝑡𝑒) = ⇒ 𝑁𝑜𝑛 − 𝑒𝑞𝑢𝑖𝑝𝑟𝑜𝑏𝑎𝑏𝑙𝑒
6 6
1
𝑃(𝑂𝑑𝑑) = 𝑃(𝐸𝑣𝑒𝑛) = ⇒ 𝐸𝑞𝑢𝑖𝑝𝑟𝑜𝑏𝑎𝑏𝑙𝑒
2

1.42: Probability for Equiprobable Outcomes


If outcomes in the sample space are equiprobable, then:
Favourable Outcomes
Probability of A = 𝑃(𝐴) =
Total Outcomes

Warning: If the outcomes are not equiprobable, then this formula cannot be used.

Example 1.43
An urn has yellow and green balls. When drawing a ball from the urn, the sample space below is equiprobable:
{𝐷𝑎𝑟𝑘 𝑌𝑒𝑙𝑙𝑜𝑤, 𝐿𝑖𝑔ℎ𝑡 𝑌𝑒𝑙𝑙𝑜𝑤, 𝐺𝑟𝑒𝑒𝑛}
A. Determine 𝑃(𝑌𝑒𝑙𝑙𝑜𝑤)
B. Determine 𝑃(𝐺𝑟𝑒𝑒𝑛)

2
𝑃(𝑌𝑒𝑙𝑙𝑜𝑤) =
3
1
𝑃(𝐺𝑟𝑒𝑒𝑛) =
3

Example 1.44
Two fair dice are rolled. Consider the sum of the numbers on the two faces that come up top.
{2,3,4,5,6,7,8,9,10,11,12}
A. Is the sample space below equiprobable?
Favourable Outcomes
B. Can we use this sample space with the formula 𝑃(𝐴) =
Total Outcomes

No. Total
No. 1 2 3 4 5 6
1 2 3 4 5 6 7
2 3 4 5 6 7 8
3 4 5 6 7 8 9
4 5 6 7 8 9 10
5 6 7 8 9 10 11
6 7 8 9 10 11 12
H. Tossing Coins

Example 1.45: Tossing Two Coins


What is the probability of getting:
A. getting two heads when we toss two fair coins
B. a head and a tail when we toss two fair coins

When we toss two coins, the following set of outcomes is possible:


⏟ ,⏟
𝐻𝐻 𝐻𝑇, 𝑇𝐻 , 𝑇𝑇 ⏟
𝐵𝑜𝑡ℎ 𝐻𝑒𝑎𝑑𝑠 𝑂𝑛𝑒 𝑇𝑎𝑖𝑙 𝑎𝑛𝑑 𝐵𝑜𝑡ℎ 𝑇𝑎𝑖𝑙𝑠
𝑂𝑛𝑒 𝐻𝑒𝑎𝑑

P a g e 16 | 121
Get all the files at: https://bit.ly/azizhandouts
Aziz Manva (azizmanva@gmail.com)

Favourable Outcomes 1
P(𝑇𝑤𝑜 𝐻𝑒𝑎𝑑𝑠) = =
Total Outcomes 4
1
2

Example 1.46: Tossing Three Coins


When we toss three fair coins, what is the probability of getting:
A. three heads
B. two tails
C. one head

(𝑯𝑯𝑯)(𝐻𝐻𝑇)(𝐻𝑇𝐻)(𝐻𝑇𝑇)(𝑇𝐻𝐻)(𝑇𝐻𝑇)(𝑇𝑇𝐻)(𝑇𝑇𝑇)
(𝐻𝐻𝐻)(𝐻𝐻𝑇)(𝐻𝑇𝐻)(𝑯𝑻𝑻)(𝑇𝐻𝐻)(𝑻𝑯𝑻)(𝑻𝑻𝑯)(𝑇𝑇𝑇)
I. Sample Spaces

Example 1.47
The sample space when tossing a coin is {𝐻, 𝑇}. Hence, the probability of getting a heads is:
Favourable Outcomes 1
𝑃(𝐻) = =
Total Outcomes 2

The sample space with respect to the ceiling of a hotel room falling today is {𝐹𝑎𝑙𝑙, 𝑁𝑜𝑡 𝐹𝑎𝑙𝑙}. The probability
that it will fall is
Favourable Outcomes 1
𝑃(𝐹𝑎𝑙𝑙) = =
Total Outcomes 2

Identify the logical flaw, if any, in the thought process above.

The formula that has been used has the underlying assumption that the outcomes listed in the sample space are
equiprobable, which means that they have equal probability of happening.

In the case of the coin, it is not mentioned to be fair.


Similarly, there is no reason that the ceiling is as likely to fall as it is not.

Example 1.48
A bag has balls numbered 1 to 20. The sample space below for drawing a single ball from the bag is not
equiprobable. Also, it is not exhaustive (all the outcomes are not listed).
{𝑃𝑟𝑖𝑚𝑒, 𝐶𝑜𝑚𝑝𝑜𝑠𝑖𝑡𝑒}
Write the sample space in such a way that the outcomes are equiprobable, and 𝑃(𝑃𝑟𝑖𝑚𝑒) and 𝑃(𝐶𝑜𝑚𝑝𝑜𝑠𝑖𝑡𝑒)
can be calculated individually.

{1,2,3, … ,20}

Example 1.49
An urn has one green ball, two blue balls and three yellow balls. The sample space below for drawing a single
ball from the urn is correct, but not equiprobable.
{𝐺𝑟𝑒𝑒𝑛, 𝐵𝑙𝑢𝑒, 𝑌𝑒𝑙𝑙𝑜𝑤}

You wish to calculate the probability of the three different colors. Write the sample space in such a way that the
outcomes are equiprobable, and the probabilities of all three colors can be calculated individually.

P a g e 17 | 121
Get all the files at: https://bit.ly/azizhandouts
Aziz Manva (azizmanva@gmail.com)

Attempt I


{𝐺𝑟𝑒𝑒𝑛𝐵𝑙𝑢𝑒 , ⏟
𝑌𝑒𝑙𝑙𝑜𝑤 } ⇒ 𝐸𝑞𝑢𝑖𝑝𝑟𝑜𝑏𝑎𝑏𝑙𝑒
3 𝑂𝑢𝑡𝑐𝑜𝑚𝑒𝑠 3 𝑂𝑢𝑡𝑐𝑜𝑚𝑒𝑠
And this tells us that
1
𝑃(𝑌𝑒𝑙𝑙𝑜𝑤) =
2
1
𝑃(𝐺𝑟𝑒𝑒𝑛 𝑜𝑟 𝐵𝑙𝑢𝑒) =
2

But, it does not tell us:


𝑃(𝐺𝑟𝑒𝑒𝑛) =?
𝑃(𝐵𝑙𝑢𝑒) =?
Correct Answer
Imagine that the color of the balls is assigned to the faces of a standard six-sided die:
{1𝐺𝑟𝑒𝑒𝑛 , 2𝐵𝑙𝑢𝑒 , 3𝐵𝑙𝑢𝑒 , 4𝑌𝑒𝑙𝑙𝑜𝑤 , 5𝑌𝑒𝑙𝑙𝑜𝑤 , 6𝑌𝑒𝑙𝑙𝑜𝑤 }

We already know that the sample space above is equiprobable.

Then, the outcome when you roll the die will correspond to an outcome when you draw a ball.

1.3 Calculating Probabilities


A. Rolling Dice: Sums

Example 1.50
Two standard 6-sided dice are tossed. What is the probability that the sum of the numbers shown on the dice is
a prime number? Express your answer as a common fraction.
Total
(MathCounts 2002 State Sprint)
1 2 3 4 5 6
1 2 3 4 5 6 7
See the table for counting how many times a number occurs:
2 3 4 5 6 7 8
➢ Each instance of a number is given the same color.
3 4 5 6 7 8 9
➢ The numbers occur in a triangular pattern.
4 5 6 7 8 9 10
➢ This gives the diagram its name. 5 6 7 8 9 10 11
6 7 8 9 10 11 12
Prime Numbers in the range 2 − 12 are:
{2,3, 5, 7,11}
2: 1 𝑂𝑢𝑡𝑐𝑜𝑚𝑒
3: 2 𝑂𝑢𝑡𝑐𝑜𝑚𝑒𝑠
5: 4 𝑂𝑢𝑡𝑐𝑜𝑚𝑒
7: 6 𝑂𝑢𝑡𝑐𝑜𝑚𝑒𝑠
11: 2 𝑂𝑢𝑡𝑐𝑜𝑚𝑒𝑠
Hence, the probability of a prime number is
𝑆𝑢𝑐𝑐𝑒𝑠𝑠𝑓𝑢𝑙 𝑂𝑢𝑡𝑐𝑜𝑚𝑒𝑠 1 + 2 + 4 + 6 + 2 15 5
𝑃= = = =
𝑇𝑜𝑡𝑎𝑙 𝑂𝑢𝑡𝑐𝑜𝑚𝑒𝑠 36 36 12

Example 1.51
Two standard 6-sided dice are tossed. What is the probability that the sum of the numbers shown on the dice is
greater than 10?

P a g e 18 | 121
Get all the files at: https://bit.ly/azizhandouts
Aziz Manva (azizmanva@gmail.com)

Total
3 1 1 2 3 4 5 6
𝑃= = 1 2 3 4 5 6 7
36 12
2 3 4 5 6 7 8
3 4 5 6 7 8 9
4 5 6 7 8 9 10
5 6 7 8 9 10 11
6 7 8 9 10 11 12

Example 1.52
Two standard 6-sided dice are tossed. What is the probability that the sum of the numbers shown on the dice is
composite?

The composite numbers are: Total


{4,6,8,10,12} 1 2 3 4 5 6
21 7 1 2 3 4 5 6 7
𝑃= =
36 12 2 3 4 5 6 7 8
3 4 5 6 7 8 9
4 5 6 7 8 9 10
5 6 7 8 9 10 11
6 7 8 9 10 11 12

Example 1.53
Two standard 6-sided dice are tossed. What is the probability that the sum of the numbers shown on the dice is
a multiple of 3, but not a multiple of 4?

The numbers that meet the conditions: Total


{3,6,9} 1 2 3 4 5 6
11 1 2 3 4 5 6 7
𝑃= 2 3 4 5 6 7 8
36
3 4 5 6 7 8 9
4 5 6 7 8 9 10
5 6 7 8 9 10 11
6 7 8 9 10 11 12

Example 1.54
Two standard 6-sided dice are tossed. What is the probability that the sum of the numbers shown on the dice is
co-prime to 12?

The numbers that meet the conditions: Total


{5,7,11} 1 2 3 4 5 6
1 2 3 4 5 6 7
12 1 2 3 4 5 6 7 8
𝑃= = 3 4 5 6 7 8 9
36 3
4 5 6 7 8 9 10
5 6 7 8 9 10 11
6 7 8 9 10 11 12

P a g e 19 | 121
Get all the files at: https://bit.ly/azizhandouts
Aziz Manva (azizmanva@gmail.com)

Example 1.55
Two standard 6-sided dice are tossed. What is the probability that the sum of the numbers shown on the dice is
A. odd?
B. Even?

Part A Total
See the shaded cells in the table alongside. In every row, there 1 2 3 4 5 6
are exactly 3 numbers out of 6, which are odd. 1 2 3 4 5 6 7
Hence, the probability is: 2 3 4 5 6 7 8
3 1 3 4 5 6 7 8 9
= 4 5 6 7 8 9 10
6 2
Part B 5 6 7 8 9 10 11
See the non-shaded cells in the table alongside. In every row, 6 7 8 9 10 11 12
there are exactly 3 numbers out of 6, which are even. Hence, the probability is:
3 1
=
6 2

Example 1.56
Two standard 5-sided dice are rolled. What is the probability that the sum of the numbers shown on the dice is
odd?

See the shaded cells in the table alongside.


12 Total
𝑃= 1 2 3 4 5
25
1 2 3 4 5 6
2 3 4 5 6 7
3 4 5 6 7 8
4 5 6 7 8 9
5 6 7 8 9 10

Example 1.57
Two standard dice are rolled. One of them has 𝑛 faces, numbered from 1 to 𝑛, whereas the other has 𝑚 faces,
numbered from 1 to 𝑚. 𝑛 is odd, and 𝑚 is even. What is the probability that the sum of the numbers shown on
the dice is odd?

Since the question asks for a single answer that is applicable to


Total
any value of 𝑛 and 𝑚, we can choose any 𝑛 and 𝑚 that meet the
1 2 3 4 5 6
conditions.
1 2 3 4 5 6 7
2 3 4 5 6 7 8
Take 3 4 5 6 7 8 9
𝑛 = 6, 𝑚 = 5 4 5 6 7 8 9 10
Which is done in the table alongside. 5 6 7 8 9 10 11

Every row has equal number of odd and even numbers. Hence, the probability that the sum of the numbers is
odd is
1
2

Example 1.58

P a g e 20 | 121
Get all the files at: https://bit.ly/azizhandouts
Aziz Manva (azizmanva@gmail.com)

I roll two fair dice, each with 𝑛 outcomes, 𝑛 ≥ 2. I find the sum of the numbers on the dice. The probability that
1
the sum is odd is not 2. Then, 𝑛 must be:
A. Even
B. Odd
C. Prime
D. Composite

1
If the probability is not 2, the number of outcomes in the equiprobable sample space must be odd.

For the number of outcomes to be odd,


𝑛 𝑚𝑢𝑠𝑡 𝑏𝑒 𝑜𝑑𝑑 ⇒ Option B

B. Rolling Dice: Products

Example 1.59
If two dice are tossed, the probability that the product of the numbers showing on the tops of the dice is greater
than 10 is: (AMC 8 1992/23)

Total
17 1 2 3 4 5 6
𝑃=
36 1 1 2 3 4 5 6
2 2 4 6 8 10 12
3 3 6 9 12 15 18
4 4 8 12 16 20 24
5 5 10 15 20 25 30
6 6 12 18 24 30 36

Example 1.60
If two standard 6-sided dice are tossed, find the probability that the product of the numbers showing on the
tops of the dice is a prime number.

Prime numbers are Total


{2,3,5} 1 2 3 4 5 6
6 1 1 1 2 3 4 5 6
𝑃= = 2 2 4 6 8 10 12
36 6
3 3 6 9 12 15 18
4 4 8 12 16 20 24
5 5 10 15 20 25 30
6 6 12 18 24 30 36

Example 1.61
If two standard 6-sided dice are tossed, the probability that the product of the numbers showing on the tops of
the dice is an odd number?

Total
1 2 3 4 5 6
1 1 2 3 4 5 6

P a g e 21 | 121
Get all the files at: https://bit.ly/azizhandouts
Aziz Manva (azizmanva@gmail.com)

2 2 4 6 8 10 12
Note that there is a pattern. 1 out of every four cells is what we 3 3 6 9 12 15 18
want. 4 4 8 12 16 20 24
Hence, the probability is: 5 5 10 15 20 25 30
9 1 6 6 12 18 24 30 36
𝑃= =
36 4

Example 1.62
If two dice are tossed, what is the probability that the product of the numbers showing on the tops of the dice is
a multiple of 4, but not 3?

{4,8,16,20} Total
1 2 3 4 5 6
8 2 1 1 2 3 4 5 6
𝑃= =
36 9 2 2 4 6 8 10 12
3 3 6 9 12 15 18
4 4 8 12 16 20 24
5 5 10 15 20 25 30
Example 1.63 6 6 12 18 24 30 36
Ten tiles numbered 1 through 10 are turned face down. One tile is turned up at random, and a standard six-
sided die is rolled. What is the probability that the product of the numbers on the tile and the die will be a
prime?

Total
1 2 3 4 5 6 7 8 9 10
7 1 1 2 3 4 5 6 7 8 9 10
𝑃= 2 2 4 6 8 10 12 14 16 18 20
60
3 3 6 9 12 15 18 21 24 27 30
4 4 8 12 16 20 24 28 32 36 40
5 5 10 15 20 25 30 35 40 45 50
6 6 12 18 24 30 36 42 48 54 60

Example 1.64
Ten tiles numbered 1 through 10 are turned face down. One tile is turned up at random, and a die is rolled.
What is the probability that the product of the numbers on the tile and the die will be a square? (AMC 8
2008/24)

11 Total
𝑃= 1 2 3 4 5 6 7 8 9 10
60
1 1 2 3 4 5 6 7 8 9 10
2 2 4 6 8 10 12 14 16 18 20
3 3 6 9 12 15 18 21 24 27 30
4 4 8 12 16 20 24 28 32 36 40
5 5 10 15 20 25 30 35 40 45 50
6 6 12 18 24 30 36 42 48 54 60

P a g e 22 | 121
Get all the files at: https://bit.ly/azizhandouts
Aziz Manva (azizmanva@gmail.com)

C. Miscellaneous

Example 1.65
Tamika selects two different numbers at random from the set 8,9,10 and adds them. Carlos takes two different
numbers at random from the set 3, 5, 6 and multiplies them. What is the probability that Tamika's result is
greater than Carlos' result? (AMC 8 1998/19)
Carlos
You can select two different numbers from the set {8,9,10} in three
15 18 30
different ways:
Tamika 17
8 + 9 = 17
18
8 + 10 = 18
19
9 + 10 = 19
You can select two different numbers from the set {3,5,6} in three different ways:
3 × 5 = 15
3 × 6 = 18
5 × 6 = 30

𝑆𝑢𝑐𝑒𝑠𝑠𝑓𝑢𝑙 𝑂𝑢𝑡𝑐𝑜𝑚𝑒𝑠 4
𝑃𝑟𝑜𝑏𝑎𝑏𝑖𝑙𝑖𝑡𝑦 = =
𝑇𝑜𝑡𝑎𝑙 𝑂𝑢𝑡𝑐𝑜𝑚𝑒𝑠 9
D. Cards

Example 1.66
I draw a single card from a standard pack of 52 cards. Consider Ace as 1, and face cards as not having a number.
What is the probability that the card is:

A. Red K. Black, but not diamonds or hearts


B. Black L. Either hearts or a face card
C. A spades M. Either black or not a face card
D. A Hearts N. A red face card
E. A face card O. A black non-face card
F. A non-face card P. A face card that is Spades
G. Odd numbered Q. A non-face card that is not Spades
H. Prime numbered R. A black odd numbered card
I. Composite numbered S. An odd numbered Spades
J. Red, but not a diamond

26 1 5
𝑃(𝑅𝑒𝑑) = = 𝑃(𝑂𝑑𝑑 𝑁𝑢𝑚𝑏𝑒𝑟𝑒𝑑) =
52 2 13
26 1 4
𝑃(𝐵𝑙𝑎𝑐𝑘) = = 𝑃(𝑃𝑟𝑖𝑚𝑒 𝑁𝑢𝑚𝑏𝑒𝑟𝑒𝑑) =
52 2 13
13 1 5
𝑃(𝑆𝑝𝑎𝑑𝑒𝑠) = = 𝑃(𝐶𝑜𝑚𝑝𝑜𝑠𝑖𝑡𝑒 𝑁𝑢𝑚𝑏𝑒𝑟𝑒𝑑) =
52 4 13
13 1 13 1
𝑃(𝐻𝑒𝑎𝑟𝑡𝑠) = = 𝑃(𝑅𝑒𝑑, 𝑏𝑢𝑡 𝑛𝑜𝑡 𝑎 𝑑𝑖𝑎𝑚𝑜𝑛𝑑) = =
52 4 52 4
12 3 26 1
𝑃(𝐹𝑎𝑐𝑒 𝐶𝑎𝑟𝑑) = = 𝑃(𝐵𝑙𝑎𝑐𝑘, 𝑏𝑢𝑡 𝑛𝑜𝑡 𝑑𝑖𝑎𝑚𝑜𝑛𝑑𝑠, 𝑜𝑟 ℎ𝑒𝑎𝑟𝑡𝑠) = =
52 13 52 2
10 𝑃(𝐸𝑖𝑡ℎ𝑒𝑟 𝐻𝑒𝑎𝑟𝑡𝑠 𝑜𝑟 𝑎 𝐹𝑎𝑐𝑒 𝐶𝑎𝑟𝑑)
𝑃(𝑁𝑜𝑛 𝐹𝑎𝑐𝑒 𝐶𝑎𝑟𝑑) =
13

P a g e 23 | 121
Get all the files at: https://bit.ly/azizhandouts
Aziz Manva (azizmanva@gmail.com)

13 + 9 22 11 20 10
= = = 𝑃(𝐵𝑙𝑎𝑐𝑘 𝑁𝑜𝑛 𝐹𝑎𝑐𝑒 𝐶𝑎𝑟𝑑) = =
52 52 26 52 26
𝑃(𝐸𝑖𝑡ℎ𝑒𝑟 𝐵𝑙𝑎𝑐𝑘 𝑜𝑟 𝑁𝑜𝑡 𝑎 𝐹𝑎𝑐𝑒 𝐶𝑎𝑟𝑑) 3
26 + 20 46 23 𝑃(𝑆𝑝𝑎𝑑𝑒𝑠 𝐹𝑎𝑐𝑒 𝐶𝑎𝑟𝑑) =
52
= = = 30 15
52 52 26 𝑃(𝑁𝑜𝑛 𝐹𝑎𝑐𝑒 & 𝑁𝑜𝑡 𝑆𝑝𝑎𝑑𝑒𝑠) = =
52 26
6 3 10 5
𝑃(𝑅𝑒𝑑 𝐹𝑎𝑐𝑒 𝐶𝑎𝑟𝑑) = = 𝑃(𝐵𝑙𝑎𝑐𝑘, 𝑂𝑑𝑑 𝑁𝑢𝑚𝑏𝑒𝑟𝑒𝑑) = =
52 26 52 26
5
𝑃(𝑂𝑑𝑑 𝑁𝑢𝑚𝑏𝑒𝑟𝑒𝑑 𝑆𝑝𝑎𝑑𝑒𝑠) =
52

Ace 2 3 4 5 6 7 8 9 10 Jack Queen King Total


Clubs Face Face Face 13
Card Card Card
Spades Face Face Face 13
Card Card Card
Hearts Face Face Face 13
Card Card Card
Diamonds Face Face Face 13
Card Card Card
4 4 4 4 4 4 4 4 4 4 4 4 4 52

E. Jars, Urns, Bags and Vases

Example 1.67
A jar contains 28 marbles. Half of the marbles are red. Half of the non-red marbles are white and the rest are
blue. Todd chose a white marble at random and kept it. What is the probability that when Hosea now draws a
marble it will also be white? Express your answer as a common fraction. (MathCounts 2006 School Team)

28
𝑅𝑒𝑑 = = 14
2
14
𝑂𝑟𝑖𝑔𝑖𝑛𝑎𝑙 𝑊ℎ𝑖𝑡𝑒 = =7
2
𝑊ℎ𝑖𝑡𝑒 = 7 − 1 = 6

6 2
𝑃𝑟𝑜𝑏𝑎𝑏𝑖𝑙𝑖𝑡𝑦 = =
27 9

Example 1.68
What is the probability, expressed as a decimal, of drawing one marble which is either red or blue from a bag
containing 3 red, 2 blue, and 5 yellow marbles? (MathCounts 1994 Warm-Up 4)

3
𝑃(𝑅𝑒𝑑) =
10
2
𝑃(𝐵𝑙𝑢𝑒) =
10
3 2 5 1
𝑃(𝑅𝑒𝑑 𝑜𝑟 𝐵𝑙𝑢𝑒) = + = = = 0.5
10 10 10 2

P a g e 24 | 121
Aziz Manva (azizmanva@gmail.com)

Example 1.69
A box contains 15 red, 20 blue, and 16 green jelly beans. Jack first chooses a green jelly bean and eats it. Then he
chooses a blue jelly bean and eats it. If each of the remaining jelly beans is equally likely to be chosen, what is
the probability that Jack chooses a red jelly bean next? (Gauss 7 2020/14)

15 𝑅𝑒𝑑, 20 𝐵𝑙𝑢𝑒, 16 𝐺𝑟𝑒𝑒𝑛


Jack eats a green jelly bean and a blue jelly bean leaving:
15 𝑅𝑒𝑑, 19 𝐵𝑙𝑢𝑒, 15 𝐺𝑟𝑒𝑒𝑛

Now, the probability that Jack will choose a red jelly bean is given by:
𝑆𝑢𝑐𝑐𝑒𝑠𝑠𝑓𝑢𝑙 𝑂𝑢𝑡𝑐𝑜𝑚𝑒𝑠 15 15
= =
𝑇𝑜𝑡𝑎𝑙 𝑂𝑢𝑡𝑐𝑜𝑚𝑒𝑠 15 + 19 + 15 49

Example 1.70
Leonardo has two green balls, three yellow balls, and four red balls. He paints the green balls yellow, the yellow
balls red, and the red balls green. If he now chooses a ball at random, what is the probability that the ball is
neither yellow nor green?

2 𝐺𝑟𝑒𝑒𝑛, 3 𝑌𝑒𝑙𝑙𝑜𝑤, 4 𝑅𝑒𝑑


2 𝑌𝑒𝑙𝑙𝑜𝑤, 3 𝑅𝑒𝑑, 4 𝐺𝑟𝑒𝑒𝑛

𝑆𝑢𝑐𝑐𝑒𝑠𝑠𝑓𝑢𝑙 𝑂𝑢𝑡𝑐𝑜𝑚𝑒𝑠 3 3 1
= = =
𝑇𝑜𝑡𝑎𝑙 𝑂𝑢𝑡𝑐𝑜𝑚𝑒𝑠 2+3+4 9 3

Example 1.71
Jason has an urn has four yellow balls, and three green balls. He draws a ball at random from the urn, and rolls a
standard eight-sided die. What is the probability that he gets a prime number on the roll, and a green ball from
the urn?

Draw the sample, which consists of 1 2 3 4 5 6 7 8


7 × 8 = 56 𝐸𝑞𝑢𝑖𝑝𝑟𝑜𝑏𝑎𝑏𝑙𝑒 𝑂𝑢𝑡𝑐𝑜𝑚𝑒𝑠 Green
Green
12 3 Green
= Yellow
56 14
Yellow
Yellow
Example 1.72 Yellow
Percy has an urn with 3 black and 4 white balls. He draws a
ball from the urn at random, replaces it, and then draws another ball at random. What is the probability that:
A. Both balls are black
B. Both balls are white
C. First ball is black and second ball is white
D. Second ball is black and first ball is white

Black Black Black White White White White


Black
Black
Black
White
White

P a g e 25 | 121
Aziz Manva (azizmanva@gmail.com)

White
White

9
49
16
49
12
49
12
49
F. Spinning a Spinner

Example 1.73
The two spinners shown are spun once and each lands on one of the numbered sectors.
What is the probability that the sum of the numbers in the two sectors is prime? (AMC 8
2009/12)

{1,3,5} 𝑎𝑛𝑑 {2,4,6} First Spinner


1 3 5
𝑆𝑒𝑐𝑜𝑛𝑑 2 3 5 7
Shade the cell which are not prime. Hence, the probability is 𝑆𝑝𝑖𝑛𝑛𝑒𝑟 4 5 7 9
2 7 6 7 9 11
1− =
9 9
Example 1.74
Spinner I is divided into four equal sections labeled 2, 3, 4 and 5. Spinner II is divided into five equal sections
labeled 1, 3, 5, 7 and 9. If each spinner is spun and the resulting numbers are multiplied, what is the probability
that the product is a two-digit even number? Express your answer as a common fraction. (MathCounts 2009
State Countdown)

1 3 5 7 9
7 2 2 6 10 14 18
𝑃𝑟𝑜𝑏𝑎𝑏𝑖𝑙𝑖𝑡𝑦 = 3 3 9 15 21 27
20
4 4 12 20 28 36
5 5 15 25 35 45

Example 1.75
A game show has a spinner divided into five equal areas. The areas are numbered with the first five prime
numbers. The spinner is spun twice. What is the probability that the product of the two numbers is even?

𝐸𝑣𝑒𝑛 × 𝑂𝑑𝑑 = 𝐸𝑣𝑒𝑛 2 3 5 7 11


𝑂𝑑𝑑 × 𝑒𝑣𝑒𝑛 = 𝐸𝑣𝑒𝑛 2 4 6 10 14 22
𝐸𝑣𝑒𝑛 × 𝐸𝑣𝑒𝑛 = 𝐸𝑣𝑒𝑛 3 6
𝑂𝑑𝑑 × 𝑂𝑑𝑑 = 𝑂𝑑𝑑 5 10
1 1 1 10 1 9 7 14
+ − = − = 11 22

5 ⏟
5 ⏟
25 25 25 25
1𝑠𝑡 𝑅𝑜𝑙𝑙 2𝑛𝑑 𝑅𝑜𝑙𝑙 𝐵𝑜𝑡ℎ 𝑅𝑜𝑙𝑙𝑠
𝑖𝑠 𝐸𝑣𝑒𝑛 𝑖𝑠 𝐸𝑣𝑒𝑛 𝑎𝑟𝑒 𝐸𝑣𝑒𝑛
Complementary Probability

P a g e 26 | 121
Aziz Manva (azizmanva@gmail.com)

4 4 16 9
𝑃(𝐸𝑣𝑒𝑛) = 1 − 𝑃(𝑂𝑑𝑑) = 1 − ( ) ( ) = 1 − =
5 5 25 25

Example 1.76
A game show has a spinner divided into five equal areas. The areas are numbered with the first five prime
numbers. The spinner is spun twice. Given that the two outcomes of the spinner are distinct, what is the
probability that the product of the two numbers is even?

8 2 2 3 5 7 11
𝑃= = 2 6 10 14 22
20 5
3 6
5 10
7 14
11 22

G. Points on a Grid

Example 1.77
A 10 by 10 grid is created using 100 points, as shown. Point 𝑃 is given. One
of the other 99 points is randomly chosen to be 𝑄. What is the probability
that the line segment 𝑃𝑄 is vertical or horizontal? (Gauss Grade 7
2016/21)

Mark the dots that make PQ vertical with a blue vertical box. There are 10 points in the column, but we can’t
include P itself, so have
10 − 1 = 9 𝑃𝑜𝑖𝑛𝑡𝑠
Mark the dots that make PQ horizontal with a blue horizontal box. There
are 10 points in the row, but we can’t include P itself, so again we have
10 − 1 = 9 𝑃𝑜𝑖𝑛𝑡𝑠

Then, the required probability is


𝑆𝑢𝑐𝑒𝑠𝑠𝑓𝑢𝑙 𝑂𝑢𝑡𝑐𝑜𝑚𝑒𝑠 9 + 9 18 2
= = =
𝑇𝑜𝑡𝑎𝑙 𝑂𝑢𝑡𝑐𝑜𝑚𝑒𝑠 99 99 11

(Continuation) Example 1.78


What is the probability that the line segment 𝑃𝑄 is diagonal? (Diagonal in this case means that for every unit
that it goes right, it also goes up or down 𝑒𝑥𝑎𝑐𝑡𝑙𝑦 1 unit).

13
99
H. Money

Example 1.79
Carolyn has a $5 bill, a $10 bill, a $20 bill, and a $50 bill in her wallet. She closes her eyes and removes one of

P a g e 27 | 121
Aziz Manva (azizmanva@gmail.com)

the four bills from her wallet. What is the probability that the total value of the three bills left in her wallet is
greater than $70? (Gauss 8 2015/14)

𝑆𝑢𝑚 𝑜𝑓 𝐷𝑜𝑙𝑙𝑎𝑟 𝐵𝑖𝑙𝑙𝑠 = 5 + 10 + 20 + 50 = 85


85 − 5 = 80
85 − 10 = 75
85 − 20 = 65
85 − 50 = 35
Removing $5 bill, $10 bill is ok
Successful Outcomes=2
2 1
𝑃= =
4 2

Example 1.80
A jar contains quarters (worth $0.25 each), nickels (worth $0.05 each) and pennies (worth $0.01 each). The
value of the quarters is $10.00. The value of the nickels is $10.00. The value of the pennies is $10.00. If Judith
randomly chooses one coin from the jar, what is the probability that it is a quarter? (CEMC 2009 Gauss 8)

Ratio of values
= 1: 1: 1
Imagine that you 25 + 5 + 1 = 31 𝐶𝑒𝑛𝑡𝑠 in the jar:
⏟1 : ⏟
5 : 25

𝑄𝑢𝑎𝑟𝑡𝑒𝑟 𝑁𝑖𝑐𝑘𝑒𝑙𝑠 𝑃𝑒𝑛𝑛𝑖𝑒𝑠
The probability of getting a quarter is:
1
31
I. Range of Values

1.81: Minimum and Maximum Value of Probability


Probabilities are nonnegative, and less than 1
0 ≤ 𝑝(𝑎) ≤ 1 for all 𝑎

➢ The minimum value that we can assign to the probability of an event is 0.


➢ The maximum value that we can assign to the probability of an event is 1.

1.82: Impossible Events


Impossible events have a probability of zero.

Example 1.83

1.84: Definite Events


Definite events have a probability of 1.

All other probabilities (which are not zero or 1), must be between zero and one.
✓ In particular, probabilities cannot be negative.
✓ And, probabilities cannot be more than one.

Example 1.85
P a g e 28 | 121
Aziz Manva (azizmanva@gmail.com)

J. Back-Calculations

Example 1.86
5
A bag contains a number of candies. The probability of Judith choosing a red candy from this bag is 6. The total
number of candies in the bag must be a multiple of (CEMC Gauss 2020/6, Adapted)

𝑀𝑢𝑙𝑡𝑖𝑝𝑙𝑒 𝑜𝑓 6

Example 1.87
A bag contains red marbles, white marbles, green marbles, and blue marbles. There are an equal number of red
marbles and white marbles, and five times as many green marbles as blue marbles. There is a 35% chance of
selecting a red marble first. What is the fewest possible number of green marbles in the bag? (MathCounts 2009
Chapter Team)

𝑊ℎ𝑖𝑡𝑒 = 𝑅𝑒𝑑 = 35%


𝑃(𝐺𝑟𝑒𝑒𝑛) = 25%
𝑃(𝐵𝑙𝑢𝑒) = 5%
Ratio is:
35: 35: 25: 5 = 7: 7: ⏟
5 :1
𝑮𝒓𝒆𝒆𝒏

K. Algebra

Example 1.88
In a bag with 20 marbles, five of them are blue. How many blue marbles must be added to the bag so that the
1
probability of selecting a blue marble at random is ? (MathCounts 2006 Chapter Countdown)
2

Let the number of blue marbles to be added be 𝑏. Then, after adding the blue marbles:
5+𝑏 1
𝑃(𝐵𝑙𝑢𝑒) = = ⇒ 𝑏 = 10
20 + 𝑏 2

Example 1.89
A bag contains only blue balls and green balls. There are 6 blue balls. If the probability of drawing a blue ball at
1
random from this bag is 4, then the number of green balls in the bag is: (AMC 8 1990/14)

Let the number of 𝑔𝑟𝑒𝑒𝑛 𝑏𝑎𝑙𝑙𝑠 = 𝑔


6 1
= ⇒ 24 = 6 + 𝑔 ⇒ 𝑔 = 18
6+𝑔 4

Example 1.90
A box contains some green marbles and exactly four red marbles. The probability of selecting a red marble is
𝑥%. If the number of green marbles is doubled, the probability of selecting one of the four red marbles from the
box is (𝑥 − 15)%. How many green marbles are in the box before the number of green marbles is doubled?
(MathCounts 2007 State Sprint)

P a g e 29 | 121
Aziz Manva (azizmanva@gmail.com)

Suppose the number of green marbles is 𝑔.


4 4
= 𝑥% , = (𝑥 − 15)%
𝑔+4
⏟ 2𝑔 + 4

𝑬𝒒𝒖𝒂𝒕𝒊𝒐𝒏 𝑰 𝑬𝒒𝒖𝒂𝒕𝒊𝒐𝒏 𝑰𝑰
Subtract Equation II from Equation I:
4 4 15
− =
𝑔 + 4 2𝑔 + 4 100
Add the fractions:
4(2𝑔 + 4) − 4(𝑔 + 4) 3
=
(𝑔 + 4)(2𝑔 + 4) 20
Simplify:
4𝑔 3
=
2𝑔2 + 12𝑔 + 16 20
Cross-multiply:
80𝑔 = 6𝑔2 + 36𝑔 + 48
Simplify, and collate terms on one side:
3𝑔2 − 22𝑔 + 24 = 0
3𝑔2 − 18𝑔 − 4𝑔 + 24 = 0
3𝑔(𝑔 − 6) − 4(𝑔 − 6) = 0
(3𝑔 − 4)(𝑔 − 6) = 0
4
𝑔 = 6 𝑂𝑅 𝑔 =
3
𝑔=6

L. Ratios

Example 1.91
At a party there are only single women and married men with their wives. The probability that a randomly
2
selected woman is single is 5. What fraction of the people in the room are married men? (AMC 8 2004/22)

Make a set(𝑆 = 𝑆𝑖𝑛𝑔𝑙𝑒, 𝑀 = 𝑀𝑎𝑟𝑟𝑖𝑒𝑑)

S S M M M

Each of the married women must also have their husband (𝐻 = 𝐻𝑢𝑠𝑏𝑎𝑛𝑑)

S S M M M
H H H

3
𝑀𝑎𝑟𝑟𝑖𝑒𝑑 𝑀𝑒𝑛 =
8
M. Odds

1.92: Probability
𝐹𝑎𝑣𝑜𝑟𝑎𝑏𝑙𝑒 𝑂𝑢𝑡𝑐𝑜𝑚𝑒𝑠
𝑃𝑟𝑜𝑏𝑎𝑏𝑖𝑙𝑖𝑡𝑦 =
𝑇𝑜𝑡𝑎𝑙 𝑂𝑢𝑡𝑐𝑜𝑚𝑒𝑠
𝑇𝑜𝑡𝑎𝑙 𝑂𝑢𝑡𝑐𝑜𝑚𝑒𝑠 = 𝐹𝑎𝑣𝑜𝑟𝑎𝑏𝑙𝑒 𝑂𝑢𝑡𝑐𝑜𝑚𝑒𝑠 + 𝑈𝑛𝑓𝑎𝑣𝑜𝑟𝑎𝑏𝑙𝑒 𝑂𝑢𝑡𝑐𝑜𝑚𝑒𝑠

P a g e 30 | 121
Aziz Manva (azizmanva@gmail.com)

1.93: Odds in favor of an event


𝐹𝑎𝑣𝑜𝑟𝑎𝑏𝑙𝑒 𝑂𝑢𝑡𝑐𝑜𝑚𝑒𝑠
𝑂𝑑𝑑𝑠 𝑖𝑛 𝐹𝑎𝑣𝑜𝑟 =
𝑈𝑛𝑓𝑎𝑣𝑜𝑢𝑟𝑎𝑏𝑙𝑒 𝑂𝑢𝑡𝑐𝑜𝑚𝑒𝑠

➢ Odds focuses on the ratio of favorable outcomes to unfavorable outcomes.


➢ Odds have traditionally been used in gambling games, horse races, and the like.
➢ They are also used in data science.
➢ Odds are usually expressed as a ratio.

Example 1.94
An urn has nine yellow balls and two white balls. What are the odds of picking a yellow ball?

𝐹𝑎𝑣𝑜𝑟𝑎𝑏𝑙𝑒 𝑂𝑢𝑡𝑐𝑜𝑚𝑒𝑠 9
𝑂𝑑𝑑𝑠 = = = 9: 2
𝑈𝑛𝑓𝑎𝑣𝑜𝑢𝑟𝑎𝑏𝑙𝑒 𝑂𝑢𝑡𝑐𝑜𝑚𝑒𝑠 2

Example 1.95
I toss a coin three times. What are the odds in favor of getting at least two heads?

If you toss a coin 3 times, the total number of outcomes are:


𝑇𝑜𝑡𝑎𝑙 𝑂𝑢𝑡𝑐𝑜𝑚𝑒𝑠 = 23 = 8

𝐹𝑎𝑣𝑜𝑢𝑟𝑎𝑏𝑙𝑒 𝑂𝑢𝑡𝑐𝑜𝑚𝑒𝑠 = {𝐻𝐻𝑇, 𝑇𝐻𝐻, 𝐻𝑇𝐻, 𝐻𝐻𝐻}


𝑈𝑛𝑓𝑎𝑣𝑜𝑢𝑟𝑎𝑏𝑙𝑒 𝑂𝑢𝑡𝑐𝑜𝑚𝑒𝑠 = 8 − 4 = 4

𝐹𝑎𝑣𝑜𝑟𝑎𝑏𝑙𝑒 𝑂𝑢𝑡𝑐𝑜𝑚𝑒𝑠 4 1
𝑂𝑑𝑑 𝑖𝑛 𝐹𝑎𝑣𝑜𝑟 = = = = 1: 1
𝑈𝑛𝑓𝑎𝑣𝑜𝑢𝑟𝑎𝑏𝑙𝑒 𝑂𝑢𝑡𝑐𝑜𝑚𝑒𝑠 4 1

Example 1.96
I toss a coin five times. What are the odds in favor of getting at least two heads?

If you toss a coin 5 times, the total number of outcomes are:


𝑇𝑜𝑡𝑎𝑙 𝑂𝑢𝑡𝑐𝑜𝑚𝑒𝑠 = 25 = 32

This time, we will use complementary counting.


Instead of counting favorable outcomes, we will count unfavorable outcomes, which are:
𝑂𝑢𝑡𝑐𝑜𝑚𝑒𝑠 𝑤𝑖𝑡ℎ 𝑍𝑒𝑟𝑜 𝐻𝑒𝑎𝑑𝑠 = {𝑇𝑇𝑇𝑇𝑇} ⇒ 1 𝑂𝑢𝑡𝑐𝑜𝑚𝑒
𝑂𝑢𝑡𝑐𝑜𝑚𝑒𝑠 𝑤𝑖𝑡ℎ 1 𝐻𝑒𝑎𝑑𝑠 = {𝑇𝑇𝑇𝑇𝐻, 𝑇𝑇𝑇𝐻𝑇, 𝑇𝑇𝐻𝑇𝑇, 𝑇𝐻𝑇𝑇𝑇, 𝐻𝑇𝑇𝑇𝑇} ⇒ 5 𝑂𝑢𝑡𝑐𝑜𝑚𝑒𝑠

𝑇𝑜𝑡𝑎𝑙 𝑈𝑛𝑓𝑎𝑣𝑜𝑟𝑎𝑏𝑙𝑒 𝑂𝑢𝑡𝑐𝑜𝑚𝑒𝑠 = ⏟


1 + ⏟
5 =6
𝑍𝑒𝑟𝑜 𝑂𝑛𝑒
𝐻𝑒𝑎𝑑𝑠 𝐻𝑒𝑎𝑑𝑠

𝐹𝑎𝑣𝑜𝑢𝑟𝑎𝑏𝑙𝑒 𝑂𝑢𝑡𝑐𝑜𝑚𝑒𝑠 = 32 − 6 = 26

𝐹𝑎𝑣𝑜𝑟𝑎𝑏𝑙𝑒 𝑂𝑢𝑡𝑐𝑜𝑚𝑒𝑠 26 13
𝑂𝑑𝑑 𝑖𝑛 𝐹𝑎𝑣𝑜𝑟 = = = = 13: 3
𝑈𝑛𝑓𝑎𝑣𝑜𝑢𝑟𝑎𝑏𝑙𝑒 𝑂𝑢𝑡𝑐𝑜𝑚𝑒𝑠 6 3

1.97: Odds against an event

P a g e 31 | 121
Aziz Manva (azizmanva@gmail.com)

𝑈𝑛𝑓𝑎𝑣𝑜𝑟𝑎𝑏𝑙𝑒 𝑂𝑢𝑡𝑐𝑜𝑚𝑒𝑠 1
𝑂𝑑𝑑𝑠 𝐴𝑔𝑎𝑖𝑛𝑠𝑡 = =
𝐹𝑎𝑣𝑜𝑢𝑟𝑎𝑏𝑙𝑒 𝑂𝑢𝑡𝑐𝑜𝑚𝑒𝑠 𝑂𝑑𝑑𝑠 𝐹𝑜𝑟

➢ 𝑂𝑑𝑑𝑠 𝐴𝑔𝑎𝑖𝑛𝑠𝑡 is the reciprocal of 𝑂𝑑𝑑𝑠 𝐹𝑜𝑟.

Example 1.98
You found the odds for various events in the examples above. Now, those events are repeated again for your
reference. This time, find the odds against those events:
A. Getting at least two heads when you toss a coin three times
B. Getting at least two heads when you toss a coin five times

Part A
1 1
𝑂𝑑𝑑𝑠 𝑓𝑜𝑟 = 1: 1 = ⇒ 𝑂𝑑𝑑𝑠 𝐴𝑔𝑎𝑛𝑖𝑠𝑡 =
1 1
Part B
13 3
𝑂𝑑𝑑𝑠 𝑓𝑜𝑟 = 13: 3 = ⇒ 𝑂𝑑𝑑𝑠 𝐴𝑔𝑎𝑛𝑖𝑠𝑡 = = 3: 13
3 13

Example 1.99
If I toss a coin four times, what are the odds
A. in favor of getting more than one head
B. against getting more than one head
C. in favor of getting exactly one head
D. against getting exactly one head
Part A
The total number of outcomes is
24 = 16

Part A
We use complementary counting. The number of ways of getting zero heads or one head is:
{𝑇𝑇𝑇𝑇, 𝑇𝑇𝑇𝐻, 𝑇𝑇𝐻𝑇, 𝑇𝐻𝑇𝑇, 𝐻𝑇𝑇𝑇} = 5 𝑂𝑢𝑡𝑐𝑜𝑚𝑒𝑠
The number of ways of getting more than one head
= 16 − 5 = 11

The odds in favor of getting more than one head


𝐹𝑎𝑣𝑜𝑢𝑟𝑎𝑏𝑙𝑒 𝑂𝑢𝑡𝑐𝑜𝑚𝑒𝑠 11
= = = 11: 5
𝑈𝑛𝑓𝑎𝑣𝑜𝑢𝑟𝑎𝑏𝑙𝑒 𝑂𝑢𝑡𝑐𝑜𝑚𝑒𝑠 5

Part B
Reciprocal of Part A
5
= = 5: 11
11
Part C
𝐹𝑎𝑣𝑜𝑢𝑟𝑎𝑏𝑙𝑒 𝑂𝑢𝑡𝑐𝑜𝑚𝑒𝑠 4 1
= = = 1: 3
𝑈𝑛𝑓𝑎𝑣𝑜𝑢𝑟𝑎𝑏𝑙𝑒 𝑂𝑢𝑡𝑐𝑜𝑚𝑒𝑠 12 3
Part D
Reciprocal of Part C
3
= = 3: 1
1

P a g e 32 | 121
Aziz Manva (azizmanva@gmail.com)

Example 1.100
𝐴𝑛𝑠𝑤𝑒𝑟 𝑒𝑎𝑐ℎ 𝑝𝑎𝑟𝑡 𝑏𝑒𝑙𝑜𝑤 𝑠𝑒𝑝𝑎𝑟𝑎𝑡𝑒𝑙𝑦.
If you roll a standard six-sided die, what are the odds in favor of getting a:
A. Prime Number
B. Composite Number
C. Even Number
D. Odd Number

𝐹𝑎𝑣𝑜𝑟𝑎𝑏𝑙𝑒 𝑂𝑢𝑡𝑐𝑜𝑚𝑒𝑠
𝑂𝑑𝑑𝑠 𝑖𝑛 𝐹𝑎𝑣𝑜𝑟 =
𝑈𝑛𝑓𝑎𝑣𝑜𝑟𝑎𝑏𝑙𝑒 𝑂𝑢𝑡𝑐𝑜𝑚𝑒𝑠

When you roll a standard six-sided die:


𝑇𝑜𝑡𝑎𝑙 𝑂𝑢𝑡𝑐𝑜𝑚𝑒𝑠 = 6
Part A
𝐹𝑎𝑣𝑜𝑟𝑎𝑏𝑙𝑒 𝑂𝑢𝑡𝑐𝑜𝑚𝑒𝑠 3
𝑂𝑑𝑑𝑠 𝑖𝑛 𝐹𝑎𝑣𝑜𝑟 = = = 1: 1
𝑈𝑛𝑓𝑎𝑣𝑜𝑟𝑎𝑏𝑙𝑒 𝑂𝑢𝑡𝑐𝑜𝑚𝑒𝑠 3
Part B
𝐹𝑎𝑣𝑜𝑟𝑎𝑏𝑙𝑒 𝑂𝑢𝑡𝑐𝑜𝑚𝑒𝑠 2 1
𝑂𝑑𝑑𝑠 𝑖𝑛 𝐹𝑎𝑣𝑜𝑟 = = = = 1: 2
𝑈𝑛𝑓𝑎𝑣𝑜𝑟𝑎𝑏𝑙𝑒 𝑂𝑢𝑡𝑐𝑜𝑚𝑒𝑠 4 2
Part C
𝐹𝑎𝑣𝑜𝑟𝑎𝑏𝑙𝑒 𝑂𝑢𝑡𝑐𝑜𝑚𝑒𝑠 3
𝑂𝑑𝑑𝑠 𝑖𝑛 𝐹𝑎𝑣𝑜𝑟 = = = 1: 1
𝑈𝑛𝑓𝑎𝑣𝑜𝑟𝑎𝑏𝑙𝑒 𝑂𝑢𝑡𝑐𝑜𝑚𝑒𝑠 3
Part D
𝐹𝑎𝑣𝑜𝑟𝑎𝑏𝑙𝑒 𝑂𝑢𝑡𝑐𝑜𝑚𝑒𝑠 3
𝑂𝑑𝑑𝑠 𝑖𝑛 𝐹𝑎𝑣𝑜𝑟 = = = 1: 1
𝑈𝑛𝑓𝑎𝑣𝑜𝑟𝑎𝑏𝑙𝑒 𝑂𝑢𝑡𝑐𝑜𝑚𝑒𝑠 3

Example 1.101
If you roll two dice, what are the odds that the sum of the dice is odd?
A. Product of the dice is odd
Total
𝑇𝑜𝑡𝑎𝑙 𝑂𝑢𝑡𝑐𝑜𝑚𝑒𝑠 = 62 = 36 B. 1 2 3 4 5 6
𝐹𝑎𝑣𝑜𝑟𝑎𝑏𝑙𝑒 𝑂𝑢𝑡𝑐𝑜𝑚𝑒𝑠 18 1 2 3 4 5 6 7
𝑂𝑑𝑑𝑠 = = = 1: 1
𝑈𝑛𝑓𝑎𝑣𝑜𝑟𝑎𝑏𝑙𝑒 𝑂𝑢𝑡𝑐𝑜𝑚𝑒𝑠 18 2 3 4 5 6 7 8
3 4 5 6 7 8 9
4 5 6 7 8 9 10
5 6 7 8 9 10 11
Example 1.102 6 7 8 9 10 11 12
If you roll two dice, what are the odds that the sum of the dice is odd?

Total
𝐹𝑎𝑣𝑜𝑟𝑎𝑏𝑙𝑒 𝑂𝑢𝑡𝑐𝑜𝑚𝑒𝑠 9 1 2 3 4 5 6
𝑂𝑑𝑑𝑠 = = = 1: 3
𝑈𝑛𝑓𝑎𝑣𝑜𝑟𝑎𝑏𝑙𝑒 𝑂𝑢𝑡𝑐𝑜𝑚𝑒𝑠 27 1 1 2 3 4 5 6
2 2 4 6 8 10 12
Example 1.103 3 3 6 9 12 15 18
If you roll three dice, what are the odds that the product of the 4 4 8 12 16 20 24
numbers on the top face of the dice is 1. 5 5 10 15 20 25 30
6 6 12 18 24 30 36
𝐹𝑎𝑣𝑜𝑟𝑎𝑏𝑙𝑒 𝑂𝑢𝑡𝑐𝑜𝑚𝑒𝑠 = (1,1,1) ⇒ 1 𝑂𝑢𝑡𝑐𝑜𝑚𝑒𝑠
𝑇𝑜𝑡𝑎𝑙 𝑂𝑢𝑡𝑐𝑜𝑚𝑒𝑠 = 63 = 216

P a g e 33 | 121
Aziz Manva (azizmanva@gmail.com)

𝑈𝑛𝑓𝑎𝑣𝑜𝑟𝑎𝑏𝑙𝑒 𝑂𝑢𝑡𝑐𝑜𝑚𝑒𝑠 = 216 − 1 = 215

𝐹𝑎𝑣𝑜𝑟𝑎𝑏𝑙𝑒 𝑂𝑢𝑡𝑐𝑜𝑚𝑒𝑠 1
𝑂𝑑𝑑𝑠 = = = 1: 215
𝑈𝑛𝑓𝑎𝑣𝑜𝑟𝑎𝑏𝑙𝑒 𝑂𝑢𝑡𝑐𝑜𝑚𝑒𝑠 215

Example 1.104
If you draw a card at random from a pack of cards, what are the odds in favor of getting a
A. Hearts
B. An Ace
C. Face Card

𝐹𝑎𝑣𝑜𝑟𝑎𝑏𝑙𝑒 𝑂𝑢𝑡𝑐𝑜𝑚𝑒𝑠 13
𝑂𝑑𝑑𝑠 = = = 1: 3
𝑈𝑛𝑓𝑎𝑣𝑜𝑟𝑎𝑏𝑙𝑒 𝑂𝑢𝑡𝑐𝑜𝑚𝑒𝑠 39
𝐹𝑎𝑣𝑜𝑟𝑎𝑏𝑙𝑒 𝑂𝑢𝑡𝑐𝑜𝑚𝑒𝑠 4 1
𝑂𝑑𝑑𝑠 = = = = 1: 12
𝑈𝑛𝑓𝑎𝑣𝑜𝑟𝑎𝑏𝑙𝑒 𝑂𝑢𝑡𝑐𝑜𝑚𝑒𝑠 48 12
𝐹𝑎𝑣𝑜𝑟𝑎𝑏𝑙𝑒 𝑂𝑢𝑡𝑐𝑜𝑚𝑒𝑠 12 3
𝑂𝑑𝑑𝑠 = = = = 3: 10
𝑈𝑛𝑓𝑎𝑣𝑜𝑟𝑎𝑏𝑙𝑒 𝑂𝑢𝑡𝑐𝑜𝑚𝑒𝑠 40 10

1.4 Application: Number Theory


A. Prime Numbers

Example 1.105
Let 𝑝 be a prime number between 40 and 60. What is the probability that 𝑝 + 12 is also a prime number?
Express your answer as a common fraction. (MathCounts 2004 State Countdown)

The values that 𝑝 can take are:


41,43,47,53,59 ⇒ 𝑇𝑜𝑡𝑎𝑙 𝑂𝑢𝑡𝑐𝑜𝑚𝑒𝑠
41 + 12 = 53
47 + 12 = 59
59 + 12 = 71

𝑆𝑢𝑐𝑒𝑠𝑠𝑓𝑢𝑙 𝑂𝑢𝑡𝑐𝑜𝑚𝑒𝑠 3
𝑃= =
𝑇𝑜𝑡𝑎𝑙 𝑂𝑢𝑡𝑐𝑜𝑚𝑒𝑠 5

Example 1.106
I pick a random two-digit integer. What is the probability that it is prime?

The number of primes from 1 to 100:


{2,3,5,7, … ,97} ⇒ 25 𝑝𝑟𝑖𝑚𝑒𝑠
We don’t want the four single digit primes {2,3,5,7}, leaving us with:
25 − 4 = 21 𝑝𝑟𝑖𝑚𝑒𝑠

Total number of two-digit integers is 90.

𝑆𝑢𝑐𝑒𝑠𝑠𝑓𝑢𝑙 𝑂𝑢𝑡𝑐𝑜𝑚𝑒𝑠 21 7
𝑃= = =
𝑇𝑜𝑡𝑎𝑙 𝑂𝑢𝑡𝑐𝑜𝑚𝑒𝑠 90 30

P a g e 34 | 121
Aziz Manva (azizmanva@gmail.com)

Example 1.107
I pick an odd natural number less than 90. What is the probability that it is prime?

Number of total outcomes


= 90

The number of primes from 1 to 100:


{2,3,5,7, … ,97} ⇒ 25 𝑝𝑟𝑖𝑚𝑒𝑠
We don’t want {2,97}, leaving us with
25 − 2 = 23 𝑝𝑟𝑖𝑚𝑒𝑠

𝑆𝑢𝑐𝑐𝑒𝑠𝑠𝑓𝑢𝑙 𝑂𝑢𝑡𝑐𝑜𝑚𝑒𝑠 23
𝑃𝑟𝑜𝑏𝑎𝑏𝑖𝑙𝑖𝑡𝑦 = =
𝑇𝑜𝑡𝑎𝑙 𝑂𝑢𝑡𝑐𝑜𝑚𝑒𝑠 45

Example 1.108
I pick a prime number less than 90. What is the probability that it is even?

𝑁𝑜. 𝑜𝑓 𝑃𝑟𝑖𝑚𝑒𝑠 < 100 = 25


𝑁𝑜. 𝑜𝑓 𝑃𝑟𝑖𝑚𝑒𝑠 < 90 = 24

And there is only one even prime.


𝑆𝑢𝑐𝑐𝑒𝑠𝑠𝑓𝑢𝑙 𝑂𝑢𝑡𝑐𝑜𝑚𝑒𝑠 1
𝑃𝑟𝑜𝑏𝑎𝑏𝑖𝑙𝑖𝑡𝑦 = =
𝑇𝑜𝑡𝑎𝑙 𝑂𝑢𝑡𝑐𝑜𝑚𝑒𝑠 24

B. Perfect Squares and Cubes

Example 1.109
I pick a number from 1 to 1000. What is the probability that it is a perfect square? (Recall that a natural number
is a perfect square when it is the square of a natural number).

The number of perfect squares from 1 to 1000 is:


{12 , 22 , … , 312 } = 31 𝑁𝑢𝑚𝑏𝑒𝑟𝑠

𝑆𝑢𝑐𝑒𝑠𝑠𝑓𝑢𝑙 𝑂𝑢𝑡𝑐𝑜𝑚𝑒𝑠 31
𝑃𝑟𝑜𝑏𝑎𝑏𝑖𝑙𝑖𝑡𝑦 = =
𝑇𝑜𝑡𝑎𝑙 𝑂𝑢𝑡𝑐𝑜𝑚𝑒𝑠 1000

Example 1.110
I pick an odd number from 1 to 1000. What is the probability that it is a perfect square? (Recall that a natural
number is a perfect square when it is the square of a natural number).

The number of perfect squares from 1 to 1000 is:


{12 , 32 , … , 312 }
We need to count the number of elements in the list above. Take square roots:
{1,3, … ,31}

Add 1:
{2,4, … ,32}

P a g e 35 | 121
Aziz Manva (azizmanva@gmail.com)

Divide 2:
{1,2, … ,16} = 16 𝑁𝑢𝑚𝑏𝑒𝑟𝑠

𝑆𝑢𝑐𝑐𝑒𝑠𝑠𝑓𝑢𝑙 𝑂𝑢𝑡𝑐𝑜𝑚𝑒𝑠 16 4
𝑃𝑟𝑜𝑏𝑎𝑏𝑖𝑙𝑖𝑡𝑦 = = =
𝑇𝑜𝑡𝑎𝑙 𝑂𝑢𝑡𝑐𝑜𝑚𝑒𝑠 500 125

Example 1.111
I pick a number from 1 to 1000. What is the probability that it is a perfect cube? (Recall that a natural number is
a perfect square when it is the square of a natural number).

𝑆𝑢𝑐𝑐𝑒𝑠𝑠𝑓𝑢𝑙 𝑂𝑢𝑡𝑐𝑜𝑚𝑒𝑠 1
𝑃𝑟𝑜𝑏𝑎𝑏𝑖𝑙𝑖𝑡𝑦 = =
𝑇𝑜𝑡𝑎𝑙 𝑂𝑢𝑡𝑐𝑜𝑚𝑒𝑠 100
C. Lists

Example 1.112
A number in the set {50, 51, 52, 53, ... , 999} is randomly selected. What is the probability that it is a two-digit
number? Express your answer as a common fraction. (MathCounts 2003 Chapter Countdown)

𝑆𝑢𝑐𝑐𝑒𝑠𝑠𝑓𝑢𝑙 𝑂𝑢𝑡𝑐𝑜𝑚𝑒𝑠 99 − 50 + 1 50 5 1
𝑃𝑟𝑜𝑏𝑎𝑏𝑖𝑙𝑖𝑡𝑦 = = = = =
𝑇𝑜𝑡𝑎𝑙 𝑂𝑢𝑡𝑐𝑜𝑚𝑒𝑠 999 − 50 + 1 950 95 19

Example 1.113
An integer is randomly selected from the integers 0 through 99, inclusive. What is the probability that the
integer is in the range 10 through 20, inclusive? Express your answer as a common fraction. (MathCounts 2008
National Countdown)

𝑆𝑢𝑐𝑐𝑒𝑠𝑠𝑓𝑢𝑙 𝑂𝑢𝑡𝑐𝑜𝑚𝑒𝑠 20 − 10 + 1 11
𝑃𝑟𝑜𝑏𝑎𝑏𝑖𝑙𝑖𝑡𝑦 = = =
𝑇𝑜𝑡𝑎𝑙 𝑂𝑢𝑡𝑐𝑜𝑚𝑒𝑠 99 − 0 + 1 100
D. Sequences

Example 1.114
Consider the sequence
0 1 2 3 102
, , , ,…,
5 5 5 5 5
If I pick a random number from the sequence above, what is the probability that it is a nonnegative integer?

𝑆𝑢𝑐𝑐𝑒𝑠𝑠𝑓𝑢𝑙 𝑂𝑢𝑡𝑐𝑜𝑚𝑒𝑠 20
𝑃𝑟𝑜𝑏𝑎𝑏𝑖𝑙𝑖𝑡𝑦 = =
𝑇𝑜𝑡𝑎𝑙 𝑂𝑢𝑡𝑐𝑜𝑚𝑒𝑠 103
E. Infinite Sequences

Example 1.115
Consider the infinite sequence below from which I pick a random term:
1 2 3
, , ,…
2 3 4

P a g e 36 | 121
Aziz Manva (azizmanva@gmail.com)

A. What is the probability that it is an irreducible fraction? (Recall that an irreducible fraction is one which
is in lowest terms, and cannot be reduced any further.)
B. If I pick a term with denominator less than 100, what is the probability that it is less than 0.9?
C. What is the probability that it is less than 0.9?

Part A
Note that each term in the sequence is irreducible. (In general, 𝑛 − 1 and 𝑛 always have HCF 1).
𝑃𝑟𝑜𝑏𝑎𝑏𝑖𝑙𝑖𝑡𝑦 = 1

Part B
Note that the sequence is increasing
1 2 3 4 9
= 0.5, = 0. 6̅, = 0.75, = 0.8, … , = 0.9
2 3 4 5 10

1 2 3 4 5 6 7 8
, , , , , , , ⇒ 8 𝑁𝑢𝑚𝑏𝑒𝑟𝑠
2 3 4 5 6 7 8 9

𝑆𝑢𝑐𝑐𝑒𝑠𝑠𝑓𝑢𝑙 𝑂𝑢𝑡𝑐𝑜𝑚𝑒𝑠 8 4
𝑃𝑟𝑜𝑏𝑎𝑏𝑖𝑙𝑖𝑡𝑦 = = =
𝑇𝑜𝑡𝑎𝑙 𝑂𝑢𝑡𝑐𝑜𝑚𝑒𝑠 98 49

Part C
𝑆𝑢𝑐𝑐𝑒𝑠𝑠𝑓𝑢𝑙 𝑂𝑢𝑡𝑐𝑜𝑚𝑒𝑠 8
𝑃𝑟𝑜𝑏𝑎𝑏𝑖𝑙𝑖𝑡𝑦 = = =0
𝑇𝑜𝑡𝑎𝑙 𝑂𝑢𝑡𝑐𝑜𝑚𝑒𝑠 ∞

Example 1.116
Consider the infinite sequence below from which I pick a random term:
1 1 1
1, , , , …
2 4 8
1 1
If I pick a number greater than 1024, what is the probability that the number is a factor of 2?

1
There are a total of ten numbers greater than 1024 in the sequence:
1 1 1 1
0
, 1 , 2 , … , 9 ⇒ 10 𝑁𝑢𝑚𝑏𝑒𝑟𝑠
2 2 2 2
1
Out of which all except the first number are factors of 2.

𝑆𝑢𝑐𝑐𝑒𝑠𝑠𝑓𝑢𝑙 𝑂𝑢𝑡𝑐𝑜𝑚𝑒𝑠 9
𝑃𝑟𝑜𝑏𝑎𝑏𝑖𝑙𝑖𝑡𝑦 = =
𝑇𝑜𝑡𝑎𝑙 𝑂𝑢𝑡𝑐𝑜𝑚𝑒𝑠 10
F. Number Systems

Example 1.117
What is the probability that a two-digit number selected at random has a tens digit less than its units digit?
Express your answer as a common fraction. (MathCounts 2001 National Countdown)

Consider Tens Digit


9: 𝑍𝑒𝑟𝑜 𝑁𝑢𝑚𝑏𝑒𝑟𝑠
8: 89 ⇒ 1 𝑁𝑢𝑚𝑏𝑒𝑟
7: 78, 79 ⇒ 2 𝑁𝑢𝑚𝑏𝑒𝑟𝑠
.

P a g e 37 | 121
Aziz Manva (azizmanva@gmail.com)

.
.
1: 12,13, … ,19 ⇒ 8 𝑁𝑢𝑚𝑏𝑒𝑟𝑠

Total Number of Numbers that satisfy the condition is:


8×9
1 + 2 + 3 + ⋯+ 8 = = 36
2

𝑆𝑢𝑐𝑐𝑒𝑠𝑠𝑓𝑢𝑙 𝑂𝑢𝑡𝑐𝑜𝑚𝑒𝑠 36 2
𝑃𝑟𝑜𝑏𝑎𝑏𝑖𝑙𝑖𝑡𝑦 = = =
𝑇𝑜𝑡𝑎𝑙 𝑂𝑢𝑡𝑐𝑜𝑚𝑒𝑠 90 5

Example 1.118
What is the probability that the square root of a randomly selected two-digit whole number is less than eight?
Express your answer as a common fraction. (MathCounts 2002 Chapter Sprint)

The valid outcomes are:


{10,11, . . . ,63} = 63 − 10 + 1 = 54 𝑁𝑢𝑚𝑏𝑒𝑟𝑠
𝑆𝑢𝑐𝑐𝑒𝑠𝑠𝑓𝑢𝑙 𝑂𝑢𝑡𝑐𝑜𝑚𝑒𝑠 54 3
𝑃𝑟𝑜𝑏𝑎𝑏𝑖𝑙𝑖𝑡𝑦 = = =
𝑇𝑜𝑡𝑎𝑙 𝑂𝑢𝑡𝑐𝑜𝑚𝑒𝑠 90 5
G. Multiples

Example 1.119
What is the probability that a nonnegative integer less than 500 is a multiple of 3?

The nonnegative integers less than 500 are


{0,1,2, … ,499} ⇒ 500 𝑁𝑢𝑚𝑏𝑒𝑟𝑠
Multiples of 3 are
{0,3,6, … ,498} = {3 × 0,3 × 1,3 × 2, … ,3 × 166} ⇒ 167 𝑁𝑢𝑚𝑏𝑒𝑟𝑠
𝑆𝑢𝑐𝑐𝑒𝑠𝑠𝑓𝑢𝑙 𝑂𝑢𝑡𝑐𝑜𝑚𝑒𝑠 167
𝑃𝑟𝑜𝑏𝑎𝑏𝑖𝑙𝑖𝑡𝑦 = =
𝑇𝑜𝑡𝑎𝑙 𝑂𝑢𝑡𝑐𝑜𝑚𝑒𝑠 500
Example 1.120
I pick a nonnegative integer less than 200. What is the probability that it is an integer multiple of 11?

The numbers that we want are:


0,11,22, … 198 → 11 × 0, 11 × 1, … ,11 × 18 → 19 𝑁𝑢𝑚𝑏𝑒𝑟𝑠
The total numbers of numbers are:
0,1,2, … ,199 → 200 𝑁𝑢𝑚𝑏𝑒𝑟𝑠
And hence:
𝑆𝑢𝑐𝑐𝑒𝑠𝑠𝑓𝑢𝑙 𝑂𝑢𝑡𝑐𝑜𝑚𝑒𝑠 19
𝑃𝑟𝑜𝑏𝑎𝑏𝑖𝑙𝑖𝑡𝑦 = =
𝑇𝑜𝑡𝑎𝑙 𝑂𝑢𝑡𝑐𝑜𝑚𝑒𝑠 200

Example 1.121
What is the probability that a positive integer less than 1000 is a multiple of both 3 and 4?

For a number to be a multiple of 3 and 4, it must be a multiple of


𝐿𝐶𝑀(3,4) = 12
𝑆𝑢𝑐𝑐𝑒𝑠𝑠𝑓𝑢𝑙 𝑂𝑢𝑡𝑐𝑜𝑚𝑒𝑠
𝑃𝑟𝑜𝑏𝑎𝑏𝑖𝑙𝑖𝑡𝑦 = =
𝑇𝑜𝑡𝑎𝑙 𝑂𝑢𝑡𝑐𝑜𝑚𝑒𝑠

P a g e 38 | 121
Aziz Manva (azizmanva@gmail.com)

Example 1.122
I pick a nonnegative integer less than 1000. What is the probability that it is an integer multiple of 5 and 7?

𝐿𝐶𝑀(5,7) = 35
Number of multiples
1000 200 𝑥 𝑥
= +1= + 1 = 28 + 1 = 29 ⇒ 29 𝑀𝑢𝑙𝑡𝑖𝑝𝑙𝑒𝑠
35 7 7 7
Number of Numbers
= 999 − 0 + 1 = 1000

𝑆𝑢𝑐𝑐𝑒𝑠𝑠𝑓𝑢𝑙 𝑂𝑢𝑡𝑐𝑜𝑚𝑒𝑠 29
𝑃𝑟𝑜𝑏𝑎𝑏𝑖𝑙𝑖𝑡𝑦 = =
𝑇𝑜𝑡𝑎𝑙 𝑂𝑢𝑡𝑐𝑜𝑚𝑒𝑠 1000
Example 1.123
What is the probability that a positive integer less than 700 is a multiple of 3, but not 4?

Number of multiples
700 1
= = 233 ⇒ 233 𝑀𝑢𝑙𝑡𝑖𝑝𝑙𝑒𝑠
3 3
Number of multiples of 𝐿𝐶𝑀(3,4) = 12 are
700 1
= = 58 ⇒ 58 𝑀𝑢𝑙𝑡𝑖𝑝𝑙𝑒𝑠
12 3
Multiples of 3, but not 4:
233 − 58 = 175
Number of Numbers
= 699
𝑆𝑢𝑐𝑐𝑒𝑠𝑠𝑓𝑢𝑙 𝑂𝑢𝑡𝑐𝑜𝑚𝑒𝑠 175
𝑃𝑟𝑜𝑏𝑎𝑏𝑖𝑙𝑖𝑡𝑦 = =
𝑇𝑜𝑡𝑎𝑙 𝑂𝑢𝑡𝑐𝑜𝑚𝑒𝑠 699

(Continuation) Example 1.124


If the above question were modified to be about nonnegative integers, then 0 is a multiple of 3 as well as 4, and
hence the probability will remain the same. Is this statement correct? If the statement is not correct, then give
the correct probability.

Statement above is correct with respect to number of Successful Outcomes.


However, number of Total Outcomes increases by 1.

Hence,
𝑆𝑢𝑐𝑐𝑒𝑠𝑠𝑓𝑢𝑙 𝑂𝑢𝑡𝑐𝑜𝑚𝑒𝑠 175 25 1
𝑃𝑟𝑜𝑏𝑎𝑏𝑖𝑙𝑖𝑡𝑦 = = = =
𝑇𝑜𝑡𝑎𝑙 𝑂𝑢𝑡𝑐𝑜𝑚𝑒𝑠 700 100 4

Example 1.125
What is the probability that a nonnegative integer less than or equal to 2020 is a multiple of either 4, or 5, but
not both?

Multiples of 4
2020
= + 1 = 505 + 1 = 506
4
P a g e 39 | 121
Aziz Manva (azizmanva@gmail.com)

Multiples of 5
2020
= + 1 = 404 + 1 = 405
5
Multiples of 4 and 5
2020
= + 1 = 101 + 1 = 102
20

Multiples of 4 and 5, but not both:


506 + 405 − 102 = 809

𝑆𝑢𝑐𝑐𝑒𝑠𝑠𝑓𝑢𝑙 𝑂𝑢𝑡𝑐𝑜𝑚𝑒𝑠 809


𝑃𝑟𝑜𝑏𝑎𝑏𝑖𝑙𝑖𝑡𝑦 = =
𝑇𝑜𝑡𝑎𝑙 𝑂𝑢𝑡𝑐𝑜𝑚𝑒𝑠 2020
H. Absolute Value

Example 1.126
I randomly pick an integer with absolute value less than 100. What is the probability that it is an integer
multiple of 7?

The numbers that we want are:


1,2,3, … ,99 ⇒ 99 𝑁𝑢𝑚𝑏𝑒𝑟𝑠
−1, −2, −3, … , −99 ⇒ 99 𝑁𝑢𝑚𝑏𝑒𝑟𝑠
0 ⇒ 1 𝑁𝑢𝑚𝑏𝑒𝑟

99 + 99 + 1 = 199 𝑁𝑢𝑚𝑏𝑒𝑟𝑠

The multiples of 7 with absolute value less than 100 are:


0, ±7, ±14, … , ±98 ⇒ 1 + 2 × 14 = 1 + 28 = 29

𝑆𝑢𝑐𝑐𝑒𝑠𝑠𝑓𝑢𝑙 𝑂𝑢𝑡𝑐𝑜𝑚𝑒𝑠 29
𝑃𝑟𝑜𝑏𝑎𝑏𝑖𝑙𝑖𝑡𝑦 = =
𝑇𝑜𝑡𝑎𝑙 𝑂𝑢𝑡𝑐𝑜𝑚𝑒𝑠 199

Example 1.127
I randomly pick an integer with absolute value less than 250. What is the probability that it is a integer multiple
of 12?

The numbers that we want are:


1,2,3, … ,249 ⇒ 249 𝑁𝑢𝑚𝑏𝑒𝑟𝑠
−1, −2, −3, … , −249 ⇒ 249 𝑁𝑢𝑚𝑏𝑒𝑟𝑠
0 ⇒ 1 𝑁𝑢𝑚𝑏𝑒𝑟

249 + 249 + 1 = 250 + 250 − 1 = 500 − 1 = 499 𝑁𝑢𝑚𝑏𝑒𝑟𝑠

The multiples of 12 with absolute value less than 250 are:


0, ±12, ±24, … , ±240 ⇒ 1 + 2 × 20 = 41

𝑆𝑢𝑐𝑐𝑒𝑠𝑠𝑓𝑢𝑙 𝑂𝑢𝑡𝑐𝑜𝑚𝑒𝑠 41
𝑃𝑟𝑜𝑏𝑎𝑏𝑖𝑙𝑖𝑡𝑦 = =
𝑇𝑜𝑡𝑎𝑙 𝑂𝑢𝑡𝑐𝑜𝑚𝑒𝑠 499

P a g e 40 | 121
Aziz Manva (azizmanva@gmail.com)

Example 1.128
I randomly pick an integer with absolute value less than 222. What is the probability that it is an integer
multiple of 19?

𝑆𝑢𝑐𝑐𝑒𝑠𝑠𝑓𝑢𝑙 𝑂𝑢𝑡𝑐𝑜𝑚𝑒𝑠 23
𝑃𝑟𝑜𝑏𝑎𝑏𝑖𝑙𝑖𝑡𝑦 = =
𝑇𝑜𝑡𝑎𝑙 𝑂𝑢𝑡𝑐𝑜𝑚𝑒𝑠 443

Example 1.129
I have positive integers, 𝑛 and 𝑚, where 𝑚 is less than 𝑛. I randomly pick an integer with absolute value less
than 𝑛. I calculate the probability that it is an integer multiple of 𝑚 and get
𝑆𝑢𝑐𝑐𝑒𝑠𝑠𝑓𝑢𝑙 𝑂𝑢𝑡𝑐𝑜𝑚𝑒𝑠 = 18
𝑇𝑜𝑡𝑎𝑙 𝑂𝑢𝑡𝑐𝑜𝑚𝑒𝑠 = 222
And hence, I calculate the probability as
𝑆𝑢𝑐𝑐𝑒𝑠𝑠𝑓𝑢𝑙 𝑂𝑢𝑡𝑐𝑜𝑚𝑒𝑠 18
𝑃𝑟𝑜𝑏𝑎𝑏𝑖𝑙𝑖𝑡𝑦 = =
𝑇𝑜𝑡𝑎𝑙 𝑂𝑢𝑡𝑐𝑜𝑚𝑒𝑠 222

Are my calculations correct?

𝑚, 2𝑚, … , 𝑝𝑚 ⇒ 𝑝 𝑚𝑢𝑙𝑡𝑖𝑝𝑙𝑒𝑠
−𝑚, −2𝑚, … , −𝑝𝑚 ⇒ 𝑝 𝑚𝑢𝑙𝑡𝑖𝑝𝑙𝑒𝑠
0 ⇒ 1 𝑀𝑢𝑙𝑡𝑖𝑝𝑙𝑒

𝑇𝑜𝑡𝑎𝑙 𝑀𝑢𝑙𝑡𝑖𝑝𝑙𝑒𝑠 = 𝑝 + 𝑝 + 1 = 2𝑝
⏟ + ⏟
1 = 𝑂𝑑𝑑 ⇒ 𝐶𝑜𝑛𝑡𝑟𝑎𝑑𝑖𝑐𝑡𝑖𝑜𝑛, 𝑏𝑒𝑐𝑎𝑢𝑠𝑒 18 𝑖𝑠 𝑒𝑣𝑒𝑛
𝐸𝑣𝑒𝑛 𝑂𝑑𝑑

1,2, … , 𝑛 ⇒ 𝑛 𝑛𝑢𝑚𝑏𝑒𝑟𝑠
−1, −2, … , −𝑛 ⇒ 𝑛 𝑛𝑢𝑚𝑏𝑒𝑟𝑠
0 ⇒ 1 𝑁𝑢𝑚𝑏𝑒𝑟

𝑇𝑜𝑡𝑎𝑙 𝑁𝑜. 𝑜𝑓 𝑁𝑜𝑠. = 𝑛 + 𝑛 + 1 = 2𝑛


⏟ + ⏟
1 ⇒ 𝑂𝑑𝑑 ⇒ 𝐶𝑜𝑛𝑡𝑟𝑎𝑑𝑖𝑐𝑡𝑖𝑜𝑛, 𝑏𝑒𝑐𝑎𝑢𝑠𝑒 222 𝑖𝑠 𝑒𝑣𝑒𝑛
𝐸𝑣𝑒𝑛 𝑂𝑑𝑑

I. Divisibility

Example 1.130
The positive three-digit integer 𝑁 has a ones digit of 0. What is the probability that 𝑁 is divisible by 4? Express
your answer as a common fraction. (MathCounts 2008 School Countdown)

The number must be from:


100
⏟ , 110, 120
⏟ , 130, 140
⏟ ,…, 980
⏟ , 990
𝐷𝑖𝑣𝑖𝑠𝑖𝑏𝑙𝑒 𝐷𝑖𝑣𝑖𝑠𝑖𝑏𝑙𝑒 𝐷𝑖𝑣𝑖𝑠𝑖𝑏𝑙𝑒 𝐷𝑖𝑣𝑖𝑠𝑖𝑏𝑙𝑒 𝑏𝑦
𝑏𝑦 4 𝑏𝑦 4 𝑏𝑦 4 4
We can form pairs, and in each pair, one of the two numbers is divisible by 4. Hence, the probability is
1
2
Method II
The number must be of the form
𝑎𝑏0
The hundred’s digit does not matter directly. However, it tells us that 𝑏 can any digit from 0 to 9.

P a g e 41 | 121
Aziz Manva (azizmanva@gmail.com)

𝐼𝑓 𝑏 𝑖𝑠 𝑒𝑣𝑒𝑛 ⇒ 𝑎𝑏0 𝑖𝑠 𝑑𝑖𝑣𝑖𝑠𝑖𝑏𝑙𝑒 𝑏𝑦 4

Hence, we need the probability that


𝑆𝑢𝑐𝑐𝑒𝑠𝑠𝑓𝑢𝑙 𝑂𝑢𝑡𝑐𝑜𝑚𝑒𝑠 5 1
𝑃(𝑏 𝑖𝑠 𝑒𝑣𝑒𝑛) = = =
𝑇𝑜𝑡𝑎𝑙 𝑂𝑢𝑡𝑐𝑜𝑚𝑒𝑠 10 2

(Continuation) Example 1.131


In the above, suppose 𝑁 is now a two-digit integer, instead of a three-digit integer, and other conditions remain
the same. Find the probability that 𝑁 is divisible by 4.

Zero is not an acceptable ten’s digit for a two-digit number.


Hence, the possible values of the number are:
10, 𝟐𝟎, 30, 𝟒𝟎, 50, 𝟔𝟎, 70, 𝟖𝟎, 90

𝑆𝑢𝑐𝑐𝑒𝑠𝑠𝑓𝑢𝑙 𝑂𝑢𝑡𝑐𝑜𝑚𝑒𝑠 4
𝑃𝑟𝑜𝑏𝑎𝑏𝑖𝑙𝑖𝑡𝑦 = =
𝑇𝑜𝑡𝑎𝑙 𝑂𝑢𝑡𝑐𝑜𝑚𝑒𝑠 9

Example 1.132
What is the probability that the last three digits of an eight-digit integer divisible by 8 are divisible by 3?

Multiples of 8:
999 7
= + 1 = 124 + 1 ⇒ 124 + 1 = 125 𝑀𝑢𝑙𝑡𝑖𝑝𝑙𝑒𝑠
8 8
Multiples of 24:
1000 2
= + 1 = 41 + 1 = 42
24 3

𝑆𝑢𝑐𝑐𝑒𝑠𝑠𝑓𝑢𝑙 𝑂𝑢𝑡𝑐𝑜𝑚𝑒𝑠 42
𝑃𝑟𝑜𝑏𝑎𝑏𝑖𝑙𝑖𝑡𝑦 = =
𝑇𝑜𝑡𝑎𝑙 𝑂𝑢𝑡𝑐𝑜𝑚𝑒𝑠 125

Example 1.133
What is the probability that an integer in the set {1,2,3, … ,100} is divisible by 2 and not divisible by 3? (AMC
10A 2003/14)

100
𝐷𝑖𝑣𝑖𝑠𝑖𝑏𝑙𝑒 𝑏𝑦 2 = = 50
2
Numbers which are divisible by 2 and 3 must be divisible by 𝐿𝐶𝑀(2,3) = 6
100 50 2
𝐷𝑖𝑣𝑖𝑠𝑖𝑏𝑙𝑒 𝑏𝑦 6 = = = 16 ⇒ 𝑅𝑜𝑢𝑛𝑑 𝑑𝑜𝑤𝑛 𝑡𝑜 16
6 3 3
𝐷𝑖𝑣𝑖𝑠𝑖𝑏𝑙𝑒 𝑏𝑦 2, 𝑏𝑢𝑡 𝑛𝑜𝑡 𝑑𝑖𝑣𝑖𝑠𝑖𝑏𝑙𝑒 𝑏𝑦 3 = 50 − 16 = 34
𝑆𝑢𝑐𝑐𝑒𝑠𝑠𝑓𝑢𝑙 𝑂𝑢𝑡𝑐𝑜𝑚𝑒𝑠 34 17
𝑃= = =
𝑇𝑜𝑡𝑎𝑙 𝑂𝑢𝑡𝑐𝑜𝑚𝑒𝑠 100 50

Example 1.134
When a fair six-sided dice is tossed on a table top, the bottom face cannot be seen. What is the probability that
the product of the faces that can be seen is divisible by 6? (AMC 8 2003/12)

For a number to be divisible by 6, it must be divisible by

P a g e 42 | 121
Aziz Manva (azizmanva@gmail.com)

2 𝑎𝑛𝑑 3
The faces on a six-sided dice are:
1,2,3,4,5,6

On matter which face you hide, the product of the remaining numbers is still divisible by 6.
Hence, the probability is
1
J. HCF

Example 1.135
I pick two natural numbers less than or equal to 100. The numbers are not necessarily distinct. What is the
probability that their HCF is two or a multiple of two?

For the HCF to be 2, or a multiple of 2, both the numbers must be even. First Pick
Odd Even
𝑆𝑢𝑐𝑐𝑒𝑠𝑠𝑓𝑢𝑙 𝑂𝑢𝑡𝑐𝑜𝑚𝑒𝑠 1 Second Odd
𝑃= = Pick Even
𝑇𝑜𝑡𝑎𝑙 𝑂𝑢𝑡𝑐𝑜𝑚𝑒𝑠 4

K. Co-prime

Example 1.136
A circular spinner is divided into 20 equal sections, numbered from 1 to 20. An arrow attached to the center of
the spinner is spun. What is the probability that the arrow stops in a section containing a number that is co-
prime to 20?

Any even numbers will have HCF 2 with 20. Hence, we only need to check the odd numbers.
20 = 22 × 5

Hence, we can also eliminate all multiples of 5. The numbers which are left are:
1,3,7,9,11,13,17,19 ⇒ 8 𝑁𝑢𝑚𝑏𝑒𝑟𝑠

𝑆𝑢𝑐𝑐𝑒𝑠𝑠𝑓𝑢𝑙 𝑂𝑢𝑡𝑐𝑜𝑚𝑒𝑠 8 2
𝑃= = =
𝑇𝑜𝑡𝑎𝑙 𝑂𝑢𝑡𝑐𝑜𝑚𝑒𝑠 20 5

Example 1.137
I pick two distinct natural numbers from 1 to 6. What is the probability that they are co-prime?

You can think of this as two dice being rolled, with the second Total
roll not allowed to be equal to the first roll. 1 2 3 4 5 6
1
This eliminates the black boxes in the table. 2
Since the order is not important, we can eliminate either the 3
4
upper half, or the lower half of the table. In this case, we chose to
5
eliminate the upper half.
6

P a g e 43 | 121
Aziz Manva (azizmanva@gmail.com)

𝑆𝑢𝑐𝑐𝑒𝑠𝑠𝑓𝑢𝑙 𝑂𝑢𝑡𝑐𝑜𝑚𝑒𝑠 11
𝑃= =
𝑇𝑜𝑡𝑎𝑙 𝑂𝑢𝑡𝑐𝑜𝑚𝑒𝑠 15

1.137 Total=6*5=30 Successful=30-6=24 because pairs (2,4), (2,6), (3,6), (4,2), (6,2), (6,3) are invalid.
Therefore, P=24/30=4/5

Example 1.138
I have a natural number 𝑛. I pick a random two-digit natural number. The probability that the number I pick is
coprime to 𝑛 is 1. What is the smallest possible value of 𝑛?

𝑛=1

Example 1.139
I have a natural number 𝑛 greater than 1. I pick a natural number at random from 1 to 101. The probability that
the number I pick is coprime to 𝑛 is 1. What is the smallest possible value of 𝑛?

Suppose
𝑛 = 2 ⇒ 2 𝑖𝑠 𝑛𝑜𝑡 𝑐𝑜𝑝𝑟𝑖𝑚𝑒 𝑡𝑜 2 𝑖𝑡𝑠𝑒𝑙𝑓 ⇒ 𝐷𝑜𝑒𝑠 𝑛𝑜𝑡 𝑤𝑜𝑟𝑘
𝑛 = 3 ⇒ 3 𝑖𝑠 𝑛𝑜𝑡 𝑐𝑜𝑝𝑟𝑖𝑚𝑒 𝑡𝑜 3 𝑖𝑡𝑠𝑒𝑙𝑓 ⇒ 𝐷𝑜𝑒𝑠 𝑛𝑜𝑡 𝑤𝑜𝑟𝑘
.
.
.
𝑛 = 101 ⇒ 100 𝑖𝑠 𝑛𝑜𝑡 𝑐𝑜𝑝𝑟𝑖𝑚𝑒 𝑡𝑜 3 𝑖𝑡𝑠𝑒𝑙𝑓 ⇒ 𝐷𝑜𝑒𝑠 𝑛𝑜𝑡 𝑤𝑜𝑟𝑘

The smallest 𝑛 that will work is a number larger than 101, which has no prime factor in common with any
number from 1 to 100.
In other words, we want the smallest prime number larger than 101, which is:
103

L. Divisors

Example 1.140
A circular spinner is divided into 20 equal sections, as shown. An arrow is attached to
the center of the spinner. The arrow is spun once. What is the probability that the
arrow stops in a section containing a number that is a divisor of 20? (Gauss Grade 7
2017/16)

There are 20 total outcomes when the spinner is spun. Of these, the divisors are:
1,2,4,5,10,20 ⇒ 6 𝑁𝑢𝑚𝑏𝑒𝑟𝑠
Hence, the probability is
𝑆𝑢𝑐𝑒𝑠𝑠𝑓𝑢𝑙 𝑂𝑢𝑡𝑐𝑜𝑚𝑒𝑠 6 3
𝑃= = =
𝑇𝑜𝑡𝑎𝑙 𝑂𝑢𝑡𝑐𝑜𝑚𝑒𝑠 20 10
M. Number of Divisors

1.141: Number of Divisors


The number of divisors 𝜏(𝑥) of a number 𝒙 with prime factorisation 𝒂𝒑 𝒃𝒒 𝒄𝒓 is given by (𝒑 + 𝟏)(𝒒 + 𝟏)(𝒓 + 𝟏).

P a g e 44 | 121
Aziz Manva (azizmanva@gmail.com)

𝑥 = 𝑎𝑝 𝑏 𝑞 𝑐 𝑟 ⇒ 𝜏(𝑥)
⏟ = (𝑝 + 1)(𝑞 + 1)(𝑟 + 1)
𝑁𝑢𝑚𝑏𝑒𝑟 𝑜𝑓
𝐹𝑎𝑐𝑡𝑜𝑟𝑠

Example 1.142
Find the number of factors of 12.

12 = 2𝟐 × 3𝟏 ⇒ 𝜏(𝑛) = (2 + 1)(1 + 1) = 3 × 2 = 6 𝑃𝑜𝑤𝑒𝑟𝑠 𝑜𝑓 2 𝑃𝑜𝑤𝑒𝑟𝑠 𝑜𝑓 3


1 1 1,3
2 3 2,6
4 4,12
3 𝐶ℎ𝑜𝑖𝑐𝑒𝑠 2 𝐶ℎ𝑜𝑖𝑐𝑒𝑠 6 𝐶ℎ𝑜𝑖𝑐𝑒𝑠

Example 1.143
What is the probability that a factor of 144 is a multiple of 3?

Number of Factors of 144


144 = 122 = (22 × 3)2 = 24 × 32
𝜏(144) = (4 + 1)(2 + 1) = (5)(3) = 15

Multiple Multiple Multiple


Factors of 144 which are multiples of 3 of 1 of 3 of 9
To be a factor of 144, which is a multiple of 3, we ensure that the number is a 1 3 9
multiple of 3. 2 6 18
144 = 3 × 48 4 12 36
8 24 72
And now, we count the factors of 16 48 144
48 = 24 × 31
𝜏(48) = (4 + 1)(1 + 1) = (5)(2) = 10

𝑆𝑢𝑐𝑒𝑠𝑠𝑓𝑢𝑙 𝑂𝑢𝑡𝑐𝑜𝑚𝑒𝑠 10 2
𝑃= = =
𝑇𝑜𝑡𝑎𝑙 𝑂𝑢𝑡𝑐𝑜𝑚𝑒𝑠 15 3

Example 1.144
What is the probability that a randomly drawn positive factor of 60 is less than 7? (AMC 10A 2003/8)

60 = 22 × 3 × 5 ⇒ 𝜏(60) = (3)(2)(2) = 12
𝐹𝑎𝑐𝑡𝑜𝑟𝑠 𝑜𝑓 12 < 60 = {1,2,3,4,5,6} = 6
6 1
𝑃𝑟𝑜𝑏𝑎𝑏𝑖𝑙𝑖𝑡𝑦 = =
12 2

Example 1.145
What is the probability that a positive integer less than or equal to 24 is a factor of 24? Express your answer as a
common fraction. (MathCounts 2006 School Sprint)

A number is chosen at random from the set of consecutive natural numbers {1, 2, 3,...24}. What is the
probability that the number chosen is a factor of 4!? Express your answer as a common fraction. (MathCounts
2003 Warm-Up 4)
24 = 2^3 * 3

P a g e 45 | 121
Aziz Manva (azizmanva@gmail.com)

Number of factors = 8/24 = 1/3

Challenge 1.146
𝑚
Let 𝑛 , in lowest terms, be the probability that a randomly chosen positive divisor of 1099 is an integer multiple
of 1088. Find 𝑚 + 𝑛. (AIME 1988/5)

1099
No. of multiples of 1088 that are also divisors of 1099 are 𝜏 ( )= 𝜏(1011 )
1088
Sucessful Outcomes 𝜏(1011 ) 𝜏(2 × 5 11 11 )
(11 + 1)(11 + 1) 144
𝑃= = = = =
Total Outcomes 𝜏(1099 ) 𝜏(299 × 599 ) (99 + 1)(99 + 1) 10000

Example 1.147
Ted is going to pick one number at random from the three numbers below:
200, 36, 250
Then he will pick a random factor of the number that he has picked. What is the probability that the factor he
selects is a composite number?

200 36 250
Factorization 23 × 52 22 × 32 2 × 53
No. of Factors 4 × 3 = 12 3×3=9 2×4=8
No. of Prime Factors 𝑛{2,5} = 2 𝑛{2,3} = 2 𝑛{2,5} = 2
No. of Non-prime, non- 2+1=3 2+1=3 2+1=3
Composite Factors
No. of Composite Factors 12 − 3 = 9 9−3=6 8−3=5
Probability 9 6 5
12 9 8

1 9 6 5 49
𝐹𝑖𝑛𝑎𝑙 𝐴𝑛𝑠𝑤𝑒𝑟 = ( + + ) =
3 12 9 8 72

Example 1.148
Ted is going to pick one number at random from the three numbers below:
200, 36, 250
Then he will pick a random factor of the number that he has picked. What is the probability that the factor he
selects is a perfect power of 2, or 3 or 5?

200 36 250
Factorization 23 × 5 2 22 × 32 2 × 53
No. of Factors 4 × 3 = 12 3×3=9 2×4
Perfect Power Factors of
2 4 3 2
3 0 3 0
5 3 0 4

Total 7 6 6
Subtract Common 1 1 1
20 = 30 = 50 = 1
6 5 5

P a g e 46 | 121
Aziz Manva (azizmanva@gmail.com)

Probability 6 5 5
12 9 8

1 6 5 5 121
𝐹𝑖𝑛𝑎𝑙 𝐴𝑛𝑠𝑤𝑒𝑟 = ( + + ) =
3 12 9 8 216

1.5 Addition Rule


A. Mutually Exclusive Outcomes

1.149: Mutually Exclusive Outcomes


Outcomes which cannot happen together are called mutually exclusive outcomes.

For events to be mutually exclusive, there should be no overlap between the two events.

Example 1.150
Decide whether the outcomes below are mutually exclusive or not.
A. {𝐻𝑒𝑎𝑑𝑠, 𝑇𝑎𝑖𝑙𝑠} when tossing a coin
B. {𝑀𝑜𝑛𝑡ℎ 𝑤𝑖𝑡ℎ 𝑙𝑒𝑠𝑠 𝑡ℎ𝑎𝑛 31 𝐷𝑎𝑦𝑠, 𝑀𝑜𝑛𝑡ℎ𝑠 𝑤𝑖𝑡ℎ 𝑚𝑜𝑟𝑒 𝑡ℎ𝑎𝑛 29 𝐷𝑎𝑦𝑠 } when picking a month of the year.

𝐴: 𝑀𝑢𝑡𝑢𝑎𝑙𝑙𝑦 𝐸𝑥𝑐𝑙𝑢𝑠𝑖𝑣𝑒

Months with 30 days satisfy both events. Hence, the events are not mutually exclusive.
𝐸: 𝑁𝑜𝑡 𝑀𝑢𝑡𝑢𝑎𝑙𝑙𝑦 𝐸𝑥𝑐𝑙𝑢𝑠𝑖𝑣𝑒

1.151: Set Theory Test for Mutually Exclusive Outcomes


𝐴 ∩ 𝐵 = 𝜙 ⇔ 𝐴 𝑎𝑛𝑑 𝐵 𝑎𝑟𝑒 𝑚𝑢𝑡𝑢𝑎𝑙𝑙𝑦 𝑒𝑥𝑐𝑙𝑢𝑠𝑖𝑣𝑒

A set of two events {𝐴, 𝐵} is mutually exclusive if their intersection is a null (empty) set.

Example 1.152
Decide whether the outcomes below are mutually exclusive or not:
A. {𝑀𝑢𝑙𝑡𝑖𝑝𝑙𝑒 𝑜𝑓 3, 𝐸𝑣𝑒𝑛 𝑁𝑢𝑚𝑏𝑒𝑟} when rolling a fair six-sided die
B. {𝐸𝑣𝑒𝑛 𝑁𝑢𝑚𝑏𝑒𝑟, 𝑂𝑑𝑑 𝑁𝑢𝑚𝑏𝑒𝑟} when rolling a fair six-sided die
C. {𝑁𝑜𝑡 𝑃𝑟𝑖𝑚𝑒, 𝑁𝑜𝑡 𝐶𝑜𝑚𝑝𝑜𝑠𝑖𝑡𝑒} when rolling a fair six-sided die

Part A
𝐴 = 𝑀𝑢𝑙𝑡𝑖𝑝𝑙𝑒 𝑜𝑓 3, 𝐵 = 𝐸𝑣𝑒𝑛 𝑁𝑢𝑚𝑏𝑒𝑟
{𝑀𝑢𝑙𝑡𝑖𝑝𝑙𝑒 𝑜𝑓 3, 𝐸𝑣𝑒𝑛 𝑁𝑢𝑚𝑏𝑒𝑟} = {𝐴, 𝐵}
𝐴 ∩ 𝐵 = {3,6}
⏟ ∩ {2,4,6}
⏟ = 6 ⇒ 𝑁𝑜𝑡 𝑚𝑢𝑡𝑢𝑎𝑙𝑙𝑦 𝑒𝑥𝑐𝑙𝑢𝑠𝑖𝑣𝑒
𝑀𝑢𝑙𝑡𝑖𝑝𝑙𝑒𝑠 𝐸𝑣𝑒𝑛
𝑜𝑓 3 𝑁𝑢𝑚𝑏𝑒𝑟
Part B

{2,4,6}
⏟ ∩ {1,3,5}
⏟ = 𝜙 ⇒ 𝑀𝑢𝑡𝑢𝑎𝑙𝑙𝑦 𝑒𝑥𝑐𝑙𝑢𝑠𝑖𝑣𝑒
𝐸𝑣𝑒𝑛 𝑂𝑑𝑑
𝑁𝑢𝑚𝑏𝑒𝑟 𝑁𝑢𝑚𝑏𝑒𝑟
{1,4,6,
⏟ … } ∩ {1,2,3,5,
⏟ … } = {1} ⇒ 𝑁𝑜𝑡 𝑚𝑢𝑡𝑢𝑎𝑙𝑙𝑦 𝑒𝑥𝑐𝑙𝑢𝑠𝑖𝑣𝑒
𝑁𝑜𝑡 𝑃𝑟𝑖𝑚𝑒 𝑁𝑜𝑡 𝐶𝑜𝑚𝑝𝑜𝑠𝑖𝑡𝑒

1.153: Mutually Exclusive Outcomes for more than 2 events


P a g e 47 | 121
Aziz Manva (azizmanva@gmail.com)

If you want 3 or more events to be mutually exclusive, no 2 of the events can happen together.

Example 1.154
Decide whether the outcomes below are mutually exclusive or not.
A. {𝑅𝑒𝑑 𝐶𝑎𝑟𝑑𝑠, 𝐶𝑙𝑢𝑏𝑠, 𝐷𝑖𝑎𝑚𝑜𝑛𝑑𝑠, 𝑆𝑝𝑎𝑑𝑒𝑠} when drawing a card from a pack of cards
B. 𝑅𝑒𝑑 𝐶𝑎𝑟𝑑 𝑎𝑛𝑑 𝐵𝑙𝑎𝑐𝑘 𝐶𝑎𝑟𝑑 when drawing a card from a well-shuffled standard pack of cards
C. 𝐹𝑎𝑐𝑒 and 𝑁𝑜𝑛 − 𝐹𝑎𝑐𝑒 𝐶𝑎𝑟𝑑 when drawing a card from a well-shuffled standard pack of cards

Part A
{𝑅𝑒𝑑 𝐶𝑎𝑟𝑑𝑠} ∩ {𝐷𝑖𝑎𝑚𝑜𝑛𝑑𝑠} = {𝐷𝑖𝑎𝑚𝑜𝑛𝑑𝑠} ⇒ 𝑁𝑜𝑡 𝑚𝑢𝑡𝑢𝑎𝑙𝑙𝑦 𝑒𝑥𝑐𝑙𝑢𝑠𝑖𝑣𝑒
For example consider the Ace of Diamonds
𝐴𝑐𝑒 𝑜𝑓 𝐷𝑖𝑎𝑚𝑜𝑛𝑑𝑠 𝑖𝑠 𝑎 𝐷𝑖𝑎𝑚𝑜𝑛𝑑
𝐴𝑐𝑒 𝑜𝑓 𝐷𝑖𝑎𝑚𝑜𝑛𝑑𝑠 𝑖𝑠 𝑎 𝑅𝑒𝑑 𝐶𝑎𝑟𝑑
As shown above, these four events are not mutually exclusive.

Part B
𝑀𝑢𝑡𝑢𝑎𝑙𝑙𝑦 𝐸𝑥𝑐𝑙𝑢𝑠𝑖𝑣𝑒
Part C
𝑀𝑢𝑡𝑢𝑎𝑙𝑙𝑦 𝐸𝑥𝑐𝑙𝑢𝑠𝑖𝑣𝑒

1.155: Exhaustive Outcomes


Outcomes which cover the entire sample space are exhaustive.

𝐶𝑜𝑖𝑛 𝑇𝑜𝑠𝑠: {𝐻, 𝑇} ⇒ 𝐻𝑒𝑎𝑑𝑠 𝑜𝑟 𝑇𝑎𝑖𝑙𝑠 𝑖𝑠 𝑒𝑥ℎ𝑎𝑢𝑠𝑡𝑖𝑣𝑒

Example 1.156
I roll a standard six-sided die. Is the set of outcomes {𝑃𝑟𝑖𝑚𝑒, 𝐶𝑜𝑚𝑝𝑜𝑠𝑖𝑡𝑒} exhaustive.

𝑆𝑎𝑚𝑝𝑙𝑒 𝑆𝑝𝑎𝑐𝑒: {1,2,3,4,5,6}

𝑃𝑟𝑖𝑚𝑒𝑠 = {2,3,5}
𝐶𝑜𝑚𝑝𝑜𝑠𝑖𝑡𝑒 = {4,6}

{𝑃𝑟𝑖𝑚𝑒, 𝐶𝑜𝑚𝑝𝑜𝑠𝑖𝑡𝑒} = 𝑃𝑟𝑖𝑚𝑒 ∪ 𝐶𝑜𝑚𝑝𝑜𝑠𝑖𝑡𝑒 = {2,3,4,5,6}

Since 1 is not included, the set of outcomes {𝑃𝑟𝑖𝑚𝑒, 𝐶𝑜𝑚𝑝𝑜𝑠𝑖𝑡𝑒} is not exhaustive.

Challenge 1.157
When rolling a standard twenty-sided die, we define two mutually exclusive events 𝐴 and 𝐵 such that each
outcome in B is a single number, and number of outcomes in 𝐵 is an odd number:
𝐸𝑣𝑒𝑛𝑡 𝐴 = {𝑁𝑢𝑚𝑏𝑒𝑟 𝑖𝑠 𝑎 𝑚𝑢𝑙𝑡𝑖𝑝𝑙𝑒 𝑜𝑓 2}

Are 𝐴 and 𝐵 exhaustive?

Proof by Contradiction
Assume to the contrary, that 𝐴 and 𝐵 can be exhaustive.

𝐴 = {2,4, … ,20} ⇒ 𝑛(𝐴) = 10



𝐸𝑣𝑒𝑛

P a g e 48 | 121
Aziz Manva (azizmanva@gmail.com)

Sample Space
𝑆 = {1,2,3, … ,20} ⇒ 𝑛(𝑆) = 20

𝐸𝑣𝑒𝑛

𝑛(𝐵) = 𝑛(𝑆) − 𝑛(𝐴) = 20 − 10 = 10 ⇒ 𝐸𝑣𝑒𝑛 𝑁𝑢𝑚𝑏𝑒𝑟 ⇒ 𝑁𝑜𝑡 𝑃𝑜𝑠𝑠𝑖𝑏𝑙𝑒 ⇒ 𝐶𝑜𝑛𝑡𝑟𝑎𝑑𝑖𝑐𝑡𝑖𝑜𝑛

Hence, our original assumption was wrong.


Hence, A and B are not exhaustive.

(Continuation) Challenge 1.158


A. What are the possible values of the number of outcomes in Event 𝐵?

𝐴 = {2,4, … ,20}

The events which can be a part of Event B:


{1,3, … ,19} ⇒ 10 𝑉𝑎𝑙𝑢𝑒𝑠

Event B can have minimum of zero outcomes, and maximum of 10 outcomes, giving us:
{0,1,2, … ,10} ⇒ 11 𝑃𝑜𝑠𝑠𝑖𝑏𝑖𝑙𝑖𝑡𝑖𝑒𝑠

Note that event B cannot be any even number. For example, B cannot be:
{1,3} ⇒ ⏟𝟐 𝑉𝑎𝑙𝑢𝑒𝑠 ⇒ 𝑁𝑜𝑡 𝑉𝑎𝑙𝑖𝑑
𝑬𝒗𝒆𝒏

Hence, the possible number of outcomes of for B is:


{1,3,5,7,9} ⇒ 5 𝑉𝑎𝑙𝑢𝑒𝑠

B. Addition Rule for Mutually Exclusive Outcomes

1.159: Addition Rule: Mutually Exclusive Events


If 𝐴 and 𝐵 are mutually exclusive events, then:
𝑃(𝐴 𝑜𝑟 𝐵) = 𝑃(𝐴) + 𝑃(𝐵)

➢ The condition that the events are mutually exclusive is critical. If this condition is not met, then the
formula does not hold.

Example 1.160
What is the probability that:
A. A card drawn from a standard pack of cards is either hearts, or clubs.
B. A die roll from a standard six-sided die is either prime or composite.

13 13 1 1 1
𝑃(𝐻𝑒𝑎𝑟𝑡𝑠) + 𝑃(𝐶𝑙𝑢𝑏𝑠) = + = + =
52 52 4 4 2
3 2 5
𝑃(𝑃𝑟𝑖𝑚𝑒) + 𝑃(𝐶𝑜𝑚𝑝𝑜𝑠𝑖𝑡𝑒) = + =
6 6 6
C. General Addition Rule
The general addition is applicable for events which are not mutually exclusive.

P a g e 49 | 121
Aziz Manva (azizmanva@gmail.com)

1.161: Addition Rule


𝑃(𝐴 𝑜𝑟 𝐵) = 𝑃(𝐴) + 𝑃(𝐵) − 𝑃(𝐴 𝑎𝑛𝑑 𝐵)
𝑃(𝐴 ∪ 𝐵) = 𝑃(𝐴) + 𝑃(𝐵) − 𝑃(𝐴 ∩ 𝐵)

This formula is equivalent to the one seen for the number of elements of
union of a set:
𝑛(𝐴 ∪ 𝐵) = 𝑛(𝐴) + 𝑛(𝐵) − 𝑛(𝐴 ∩ 𝐵)

Example 1.162
Compare the formulas
𝑛(𝐴 ∪ 𝐵) = 𝑛(𝐴) + 𝑛(𝐵) − 𝑛(𝐴 ∩ 𝐵)
𝑃(𝐴 𝑜𝑟 𝐵) = 𝑃(𝐴) + 𝑃(𝐵) − 𝑃(𝐴 𝑎𝑛𝑑 𝐵)

What is ∪ and ∩ from the first formula equivalent to in the second formula?

𝑈𝑛𝑖𝑜𝑛 =∪= 𝑂𝑅
𝐼𝑛𝑡𝑒𝑟𝑠𝑒𝑐𝑡𝑖𝑜𝑛 =∩ 𝐴 = 𝐴𝑁𝐷

Example 1.163
What is the probability that a card drawn from a standard pack is either an ace or black?

General Addition Rule


4
𝑃(𝐴𝑐𝑒) =
52
26
𝑃(𝐵𝑙𝑎𝑐𝑘 𝐶𝑎𝑟𝑑) =
52
2
𝑃(𝐵𝑙𝑎𝑐𝑘 𝐴𝑐𝑒) =
52

Use 𝑃(𝐴) + 𝑃(𝐵) − 𝑃(𝐴 𝑎𝑛𝑑 𝐵)


4 26 2 28 7
+ − = =

52 ⏟
52 ⏟
52 52 13
𝑃(𝐴𝑐𝑒) 𝑃(𝐵𝑙𝑎𝑐𝑘) 𝑃(𝐵𝑙𝑎𝑐𝑘)
𝐶𝑎𝑟𝑑 𝐴𝑐𝑒𝑠

Addition Rule for Mutually Exclusive Outcomes


26 2 28 7
+ = =

52 ⏟
52 52 13
𝑷(𝑩𝒍𝒂𝒄𝒌) 𝑷(𝑵𝒐𝒏−𝑩𝒍𝒂𝒄𝒌)
𝑪𝒂𝒓𝒅 𝑨𝒄𝒆

Counting Outcomes
26
⏟ + ⏟
2
𝐵𝑙𝑎𝑐𝑘 𝐶𝑎𝑟𝑑𝑠 𝑁𝑜𝑛−𝐵𝑙𝑎𝑐𝑘 𝐴𝑐𝑒𝑠 28 7
= =
52 52 13

Example 1.164
A. Two dice are thrown. What is the probability that the product of the two numbers is a multiple of 5?
(AMC 8 2001/18)

P a g e 50 | 121
Aziz Manva (azizmanva@gmail.com)

B. Two twenty-sided dice are thrown. What is the probability that the product of the two numbers is a
multiple of 5?
C. Two eighteen-sided dice are thrown. What is the probability that the product of the two numbers is a
multiple of 5?

Part A
Method I: Lattice Diagram
We can visualise the successful outcomes using a lattice diagram. We want the two yellow paths, which intersect
in the red box (and the hence the red box would be counted twice if we counted the yellow paths directly).

Pair of Numbers
1 2 3 4 5 6
1 (1,1) (2,1) (3,1) (4,1) (5,1) (6,1)
2 (1,2) (2,2) (3,2) (4,2) (5,2) (6,2)
3 (1,3) (2,3) (3,3) (4,3) (5,3) (6,3)
4 (1,4) (2,4) (3,4) (4,4) (5,4) (6,4)
5 (1,5) (2,5) (3,5) (4,5) (5,5) (6,5)
6 (1,6) (2,6) (3,6) (4,6) (5,6) (6,6)

Method II: Addition Rule


For the product to be a multiple of 5, at least one of the numbers must be a multiple of 5.
1
𝑃(1𝑠𝑡 𝐷𝑖𝑒 𝑖𝑠 𝑎 𝑀𝑢𝑙𝑡𝑖𝑝𝑙𝑒 𝑜𝑓 5) =
6
1
𝑃(2𝑛𝑑 𝐷𝑖𝑒 𝑖𝑠 𝑎 𝑀𝑢𝑙𝑡𝑖𝑝𝑙𝑒 𝑜𝑓 5) =
6
We could add the two probabilities. However, if both dice are a multiple of 5, then that gets counted in both
events. Hence, we need to subtract the probability that both dice are a multiple of 5.
1 1 1
𝑃(𝐵𝑜𝑡ℎ 𝐷𝑖𝑐𝑒 𝑎𝑟𝑒 𝑎 𝑚𝑢𝑙𝑡𝑖𝑝𝑙𝑒 𝑜𝑓 5) = × =
6 6 36
Hence, the final answer is:

𝑃(𝐴 𝑜𝑟 𝐵) = 𝑃(𝐴) + 𝑃(𝐵) − 𝑃(𝐴 𝑎𝑛𝑑 𝐵)


6 6 1 12 1 11
+ − = − =

36 ⏟
36 ⏟
36 36 36 36
𝐹𝑖𝑟𝑠𝑡 𝐷𝑖𝑐𝑒 𝑆𝑒𝑐𝑜𝑛𝑑 𝐷𝑖𝑐𝑒 𝐵𝑜𝑡ℎ 𝐷𝑖𝑐𝑒
𝑖𝑠 𝑎 5 𝑖𝑠 𝑎 5 𝑎𝑟𝑒 5′ 𝑠

Part B
4 1
𝑃(1𝑠𝑡 𝐷𝑖𝑐𝑒 𝑖𝑠 𝑎 5) = =
20 5
4 1
𝑃(2𝑛𝑑 𝐷𝑖𝑐𝑒 𝑖𝑠 𝑎 5) = =
20 5
1 1 1
𝑃(𝐵𝑜𝑡ℎ 𝑑𝑖𝑐𝑒 𝑎𝑟𝑒 5) = × =
5 5 25
Substitute 𝑃(𝐴 𝑜𝑟 𝐵) = 𝑃(𝐴) + 𝑃(𝐵) − 𝑃(𝐴 𝑎𝑛𝑑 𝐵)
1 1 1 2 1 10 1 9
+ − = − = − =
5 5 25 5 25 25 25 25
Part C
Two eighteen-sided dice are thrown. What is the probability that the product of the two numbers is a multiple
of 5?
3 1
𝑃(1𝑠𝑡 𝐷𝑖𝑐𝑒 𝑖𝑠 𝑎 5) = =
18 6

P a g e 51 | 121
Aziz Manva (azizmanva@gmail.com)

3 1
𝑃(2𝑛𝑑 𝐷𝑖𝑐𝑒 𝑖𝑠 𝑎 5) = =
18 6
1 1 1
𝑃(𝐵𝑜𝑡ℎ 𝑑𝑖𝑐𝑒 𝑎𝑟𝑒 5) = × =
6 6 36
1 1 1 12 1 11
+ − = − =
6 6 36 36 36 36

Example 1.165
The numbers 1 through 25 are written on 25 cards with one number on each card. Sara picks one of the 25
cards at random. What is the probability that the number on her card will be a multiple of 2 or 5? Express your
answer as a common fraction. (MathCounts 2003 Chapter Sprint)

1 2 3 4 5
6 7 8 9 10
11 12 13 14 15
16 17 18 19 20
21 22 23 24 25

{2,4, … ,24} ⇒ {2 × 1,2 × 2, … ,2 × 12} ⇒ 12 𝑁𝑢𝑚𝑏𝑒𝑟𝑠


{5,10, … ,25} ⇒ {5 × 1,5 × 2, … ,5 × 5} ⇒ 5 𝑁𝑢𝑚𝑏𝑒𝑟𝑠

There are some numbers which occur in both lists will the numbers which are multiples of 𝐿𝐶𝑀(2,5) = 10:
{10,20}

𝑃(2) + 𝑃(5) − 𝑃(2 ∩ 5)


12 5 2 12 + 5 − 2 15 3
+ − = = =
25 25 25 25 25 5

Example 1.166
What is the probability that a number picked at random from 1 to 1000 will be a multiple of 2 or 5?

500 200 100 600 6 3


+ − = = =
1000 1000 1000 1000 10 5
Shortcut
Divide the numbers into groups of 5.
{1 − 5,6 − 10, … ,995 − 1000}

There are only two possible cases: the starting number is odd, or the starting number is even.

Starting Number Odd Starting Number Even


1 2 3 4 5 6 7 8 9 10

As the table shows, for each group (irrespective of the case):


𝑆𝑢𝑐𝑐𝑒𝑠𝑠𝑓𝑢𝑙 𝑂𝑢𝑡𝑐𝑜𝑚𝑒𝑠 3
=
𝑇𝑜𝑡𝑎𝑙 𝑂𝑢𝑡𝑐𝑜𝑚𝑒𝑠 5

Challenge 1.167
What is the probability that a number picked at random from 1 to 1001 will be a multiple of 2 or 5?

P a g e 52 | 121
Aziz Manva (azizmanva@gmail.com)

500 200 100 600


+ − =
1001 1001 1001 1001

Example 1.168
Juan rolls a fair regular octahedral die marked with the numbers 1 through 8. Then Amal rolls a fair six-sided
die. What is the probability that the product of the two rolls is a multiple of 3? (AMC 12B 2002/16)

Method I: Table
2×6
⏟ + 2×8
⏟ − 2×2
⏟ = 12 + 16 − 4 = 24
𝐹𝑖𝑟𝑠𝑡 𝐷𝑖𝑒=3𝑛 𝑆𝑒𝑐𝑜𝑛𝑑 𝐷𝑖𝑒=3𝑛 𝐵𝑜𝑡ℎ 𝐷𝑖𝑒=3𝑛
24 1
=
48 2

Pair of Numbers
1 2 3 4 5 6 7 8
1
2
3 8
4
5
6 8
6 6

Method II: Using the Addition Rule


2 1
𝑃(3 𝑖𝑛 𝑂𝑐𝑡𝑎ℎ𝑒𝑑𝑟𝑎𝑙) = =
8 4
2 1
𝑃(3 𝑖𝑛 𝑆𝑖𝑥 𝑆𝑖𝑑𝑒𝑑) = =
6 3

𝑃(𝑂𝑐𝑡𝑎) + 𝑃(𝑆𝑖𝑥 − 𝑠𝑖𝑑𝑒𝑑) − 𝑃(𝐵𝑜𝑡ℎ)


1 1 1 1 6 1
+ − ( )( ) = =
4 3 3 4 12 2
D. Exclusive OR
To find an exclusive OR, we must subtract the intersection twice (once
for double counting, and once since we do not want to count it at all).

1.169: Exclusive OR
𝑃(𝐴 𝑋𝑂𝑅 𝐵) = 𝑃(𝐴) + 𝑃(𝐵) − 2 ∙ 𝑃(𝐴 𝑎𝑛𝑑 𝐵)

This is not a very common question type, but it has a typical approach,
and only specific techniques will work for this kind of question.

𝑛(𝐴 𝑋𝑂𝑅 𝐵) = 𝑛(𝐴) + 𝑛(𝐵) − 2 ∙ 𝑛(𝐴 ∩ 𝐵)

Example 1.170
An integer is randomly chosen from the integers 1 through 100, inclusive. What is the probability that the
chosen integer is a perfect square or a perfect cube, but not both? Express your answer as a common fraction.
(MathCounts 2007 National Sprint)

P a g e 53 | 121
Aziz Manva (azizmanva@gmail.com)

𝑃𝑆 = 𝑃𝑒𝑟𝑓𝑒𝑐𝑡 𝑆𝑞𝑢𝑎𝑟𝑒 = {1,4,9, … ,100} = {𝟏𝟐 , 22 , 32 , … , 𝟖𝟐 , … , 102 } = 10 𝑁𝑢𝑚𝑏𝑒𝑟𝑠


𝑃𝐶 = 𝑃𝑒𝑟𝑓𝑒𝑐𝑡 𝐶𝑢𝑏𝑒 = {1,8, … ,64} = {𝟏𝟑 , 23 , 33 , 𝟒𝟑 } = 4 𝑁𝑢𝑚𝑏𝑒𝑟𝑠
𝐵𝑜𝑡ℎ 𝑃𝑒𝑟𝑓𝑒𝑐𝑡 𝑆𝑞𝑢𝑎𝑟𝑒 𝑎𝑛𝑑 𝑃𝑒𝑟𝑓𝑒𝑐𝑡 𝐶𝑢𝑏𝑒𝑠 = {13 , 43 } = 2 𝑁𝑢𝑚𝑏𝑒𝑟𝑠
Counting
No. of numbers which are perfect squares, or perfect squares, but not both is given by:
𝑛(𝑃𝑆) + 𝑛(𝑃𝐶) − 2𝑛(𝑃𝑆 ∩ 𝑃𝐶) = 10 + 4 − 2 × 2 = 10
𝑆𝑢𝑐𝑒𝑠𝑠𝑓𝑢𝑙 𝑂𝑢𝑡𝑐𝑜𝑚𝑒𝑠 10 1
𝑃= = =
𝑇𝑜𝑡𝑎𝑙 𝑂𝑢𝑡𝑐𝑜𝑚𝑒𝑠 100 10
Probability
10 4 2 10 1
𝑃(𝑃𝑆) + 𝑃(𝑃𝐶) − 2𝑃(𝑃𝑆 ∩ 𝑃𝐶) = + − 2( )= =
100 100 100 100 10

Example 1.171
What is the probability that an integer randomly chosen from 1 to 1000 is a:
A. Either a perfect square or a perfect cube
B. A perfect square or a perfect cube, but not both

Note that
322 = 1024, 312 < 1000
{12 , 22 , … , 312 }

{13 = 1,23 = 8, … , 103 = 1000}


31 10 1
𝑃(𝑃𝑆) = , 𝑃(𝑃𝐶) = =
1000 1000 100

A number which is both a perfect square and a perfect cube must be a perfect sixth power
16 = 1 2 = 13
26 = 43 = 82 = 64
36 = 272 = 93 = 729

𝑛(𝑃𝑆 𝑎𝑛𝑑 𝑃𝐶) = 𝑛{1,64,729} = 3


Part A
31 10 3 38 19
𝑃(𝑃𝑆 𝑜𝑟 𝑃𝐶) = 𝑃(𝑃𝑆) + 𝑃(𝑃𝐶) − 𝑃(𝑃𝑆 𝑎𝑛𝑑 𝑃𝐶) = + − = =
1000 1000 1000 1000 500
Part B
31 10 3 35 7
𝑃(𝑃𝑆 𝑋𝑂𝑅 𝑃𝐶) = 𝑃(𝑃𝑆) + 𝑃(𝑃𝐶) − 2𝑃(𝑃𝑆 𝑎𝑛𝑑 𝑃𝐶) = + −2× = =
1000 1000 1000 1000 200

Review 1.172
When throwing two dice, what is the probability that the sum of the dice is one out of:
A. One more than a number divisible by both of the two smallest prime numbers
B. A perfect cube
C. A perfect square
Ways to get 7 = [(1, 6), (2, 5), (3, 4)] * 2 = 3 * 2 = 6
Ways to get 8 = [(2, 6), (3, 5)] * 2 + (4, 4) = 2 * 2 + 1 = 5
Ways to get 4 or 9 = [(1, 3), (2, 2), (3, 1)] + [(3, 6), (4, 5)] * 2 = 3 + 4 = 7

P a g e 54 | 121
Aziz Manva (azizmanva@gmail.com)

Probability = Favourable / Total = (6 + 5 +7)/36 = 18/36 = ½

E. Exhaustive Outcomes

1.173: Exhaustive Outcomes


The union of exhaustive outcomes includes the entire sample space.

What this means is, that if you consider a set of exhaustive outcomes, there is no outcome possible which is not
listed.

Example 1.174
Decide whether the outcomes below are exhaustive or not.
A. {𝐻𝑒𝑎𝑑𝑠, 𝑇𝑎𝑖𝑙𝑠} when tossing a coin
B. {𝑃𝑟𝑖𝑚𝑒 𝑁𝑢𝑚𝑏𝑒𝑟, 𝐶𝑜𝑚𝑝𝑜𝑠𝑖𝑡𝑒 𝑁𝑢𝑚𝑏𝑒𝑟} when rolling a fair six-sided die
C. {𝑅𝑒𝑑 𝐶𝑎𝑟𝑑𝑠, 𝐶𝑙𝑢𝑏𝑠, 𝐷𝑖𝑎𝑚𝑜𝑛𝑑𝑠, 𝑆𝑝𝑎𝑑𝑒𝑠} when drawing a card from a pack of cards
D. {𝐸𝑣𝑒𝑛 𝑁𝑢𝑚𝑏𝑒𝑟, 𝑂𝑑𝑑 𝑁𝑢𝑚𝑏𝑒𝑟} when rolling a fair six-sided die
E. {𝑊𝑒𝑒𝑘𝑑𝑎𝑦, 𝑊𝑒𝑒𝑘𝑒𝑛𝑑} when deciding a day of the week at random

Part A
{𝐻} ∪ {𝑇} = {𝐻, 𝑇} ⇒ 𝐸𝑥ℎ𝑎𝑢𝑠𝑡𝑖𝑣𝑒
Part B
{2,3,5} ∪ {4,6} = {2,3,4,5,6} ⇒ 𝑁𝑜𝑡 𝐸𝑥ℎ𝑎𝑢𝑠𝑡𝑖𝑣𝑒 (1 𝑖𝑠 𝑚𝑖𝑠𝑠𝑖𝑛𝑔)
Part C
{𝐷𝑖𝑎𝑚𝑜𝑛𝑑𝑠} ∪ {𝐻𝑒𝑎𝑟𝑡𝑠} ∪ {𝐶𝑙𝑢𝑏𝑠} ∪ {𝑆𝑝𝑎𝑑𝑒𝑠} ⇒ 𝐸𝑥ℎ𝑎𝑢𝑠𝑡𝑖𝑣𝑒
Part D
{1,3,5} ∪ {2,4,6} ⇒ {1,2,3,4,5,6} ⇒ 𝐸𝑥ℎ𝑎𝑢𝑠𝑡𝑖𝑣𝑒
Part E
𝐸𝑥ℎ𝑎𝑢𝑠𝑡𝑖𝑣𝑒

Example 1.175
Write a list of exhaustive outcomes for
A. the days of the week?
B. when rolling a standard six-sided die?

Part A
There are 7 days of the week, and any day must be one of the seven days of the week.
{𝑀𝑜𝑛, 𝑇𝑢𝑒, 𝑊𝑒𝑑, 𝑇ℎ𝑢, 𝐹𝑟𝑖, 𝑆𝑎𝑡, 𝑆𝑢𝑛}
Part B
There are six possible outcomes, given by:
{1,2,3,45,6}

Example 1.176
Is the following set of outcomes exhaustive when rolling a twenty-sided die:
{𝑃𝑟𝑖𝑚𝑒 𝑁𝑢𝑚𝑏𝑒𝑟𝑠, 𝑀𝑢𝑙𝑡𝑖𝑝𝑙𝑒𝑠 𝑜𝑓 2, 𝑀𝑢𝑙𝑡𝑖𝑝𝑙𝑒𝑠 𝑜𝑓 3, 𝑀𝑢𝑙𝑡𝑖𝑝𝑙𝑒𝑠 𝑜𝑓 7}

1 is not included.
Hence, the outcomes are not exhaustive.

P a g e 55 | 121
Aziz Manva (azizmanva@gmail.com)

1.177: Mutually Exclusive and Exhaustive Outcomes


These events are events which are mutually exclusive and whose union is exhaustive. That it, it comprises the
entire sample space.

They are very useful in calculating probabilities. Most of the questions that we do for Probability will rely on
creating outcomes in such a way that they are mutually exclusive and exhaustive.

Example 1.178
𝑀𝑎𝑟𝑘 𝐴𝑙𝑙 𝐶𝑜𝑟𝑟𝑒𝑐𝑡 𝑂𝑝𝑡𝑖𝑜𝑛𝑠
When rolling a standard 15 sided die, we define two exhaustive and mutually exclusive events 𝐴 and 𝐵. Let 𝑛(𝐴)
be the number of events in 𝐴, and 𝑛(𝐵) be the number of events in 𝐵. Then, which of the following are possible:
A. 𝑛(𝐴) is odd and 𝑛(𝐵) is also odd
B. 𝑛(𝐴) is odd and 𝑛(𝐵) is even
C. 𝑛(𝐴) is even and 𝑛(𝐵) is odd
D. 𝑛(𝐴) is even and 𝑛(𝐵) is also even

𝑂𝑑𝑑 + 𝑂𝑑𝑑 = 𝐸𝑣𝑒𝑛 ⇒ 𝑁𝑜𝑡 𝑃𝑜𝑠𝑠𝑖𝑏𝑙𝑒


𝑂𝑑𝑑 + 𝐸𝑣𝑒𝑛 = 𝑂𝑑𝑑 ⇒ 𝑃𝑜𝑠𝑠𝑖𝑏𝑙𝑒
𝐸𝑣𝑒𝑛 + 𝑂𝑑𝑑 = 𝑂𝑑𝑑 ⇒ 𝑃𝑜𝑠𝑠𝑖𝑏𝑙𝑒
𝐸𝑣𝑒𝑛 + 𝐸𝑣𝑒𝑛 = 𝐸𝑣𝑒𝑛 ⇒ 𝑁𝑜𝑡 𝑃𝑜𝑠𝑠𝑖𝑏𝑙𝑒

(Continuation) Example 1.179


A. If the events are exhaustive, but no longer mutually exclusive, then mark all correct options.
B. If the events are mutually exclusive, but no longer exhaustive, then mark all correct options.

Part A
𝑆 = 𝐴 ⇒ 𝐴𝑙𝑙 𝑜𝑝𝑡𝑖𝑜𝑛𝑠 𝑎𝑟𝑒 𝑓𝑒𝑎𝑠𝑖𝑏𝑙𝑒 𝑛𝑜𝑤
Hence, Options A, B, C and D.

Part B
Options A, B, C and D.

Example 1.180
Decide whether the outcomes are:
A. Mutually exclusive OR Not mutually Exclusive
B. Exhaustive OR Not Exhaustive

1. {𝐻, 𝑇} when tossing a fair coin


2. {1, 2, 3, 4, 5, 6} when rolling a fair six-sided die
3. {𝐻𝑒𝑎𝑟𝑡𝑠, 𝐶𝑙𝑢𝑏𝑠, 𝑆𝑝𝑎𝑑𝑒𝑠 𝑎𝑛𝑑 𝐷𝑖𝑎𝑚𝑜𝑛𝑑𝑠} when drawing a card from a well-shuffled standard pack of
cards
{𝑅𝑒𝑑 𝐶𝑎𝑟𝑑, 𝐵𝑙𝑎𝑐𝑘 𝐶𝑎𝑟𝑑} when drawing a card from a well-shuffled standard pack of cards

F. Sum of Probabilities

1.181: Sum of Probabilities


The sum of mutually exclusive and exhaustive events has probability one.

P a g e 56 | 121
Aziz Manva (azizmanva@gmail.com)

𝑝(𝑎1 ) + 𝑝(𝑎2 ) + ⋯ + 𝑝(𝑎𝑛 ) = ∑ 𝑝(𝑎) = 1


𝑎
Where
{𝑎1 , 𝑎2 , … , 𝑎𝑛 } are mutually exclusive and exhaustive events.

Both parts are important here


➢ Mutually Exclusive: Because the probabilities are mutually exclusive, they can be directly added.
➢ Exhaustive: Because they are exhaustive, they cover all possible events.

Example 1.182
Show that the sum of probabilities of mutually exclusive and exhaustive events is one for:
A. Tossing a fair coin
B. The roll of a standard six-sided die
1 2
C. Tossing an unfair coin with probability of heads, which is 3, and probability of tails, which is 3.

1 1
{𝐻, 𝑇} ⇒ 𝑃(𝐻) + 𝑃(𝑇) =
+ =1
2 2
1 1 1 1 6
{1,2,3,4,5,6} ⇒ 𝑃(1) + 𝑃(2) + ⋯ + 𝑃(6) = + + ⋯ + = 6 × = = 1

6 6 6 6 6
6 𝑡𝑖𝑚𝑒𝑠
1 2 3
{𝐻, 𝑇} ⇒ 𝑃(𝐻) + 𝑃(𝑇) = + = = 1
3 3 3

Example 1.183
Show that when rolling a fair six-sided die, the sum of the following probabilities is not one, and explain why.
𝑃(𝑃𝑟𝑖𝑚𝑒) + 𝑃(𝐶𝑜𝑚𝑝𝑜𝑠𝑖𝑡𝑒)

3 2 5
𝑃(𝑃𝑟𝑖𝑚𝑒) + 𝑃(𝐶𝑜𝑚𝑝𝑜𝑠𝑖𝑡𝑒) = + =
6 6 6

1 is neither prime, nor composite.

The probabilities are mutually exclusive, but they are not exhaustive. Hence, the sum of the probabilities is not
one.

Example 1.184
Jane picks a natural number 𝑎 at random from 1 to 10. John picks a natural number 𝑏 at random from 1 to 10.
Identify whether the following sample space is mutually exclusive and exhaustive.
{𝑎 𝑖𝑠 𝑜𝑑𝑑 & 𝑏 𝑖𝑠 𝑜𝑑𝑑, 𝑎𝑏 𝑖𝑠 𝑒𝑣𝑒𝑛}

⏟𝑖𝑠 𝑜𝑑𝑑 & 𝑏 𝑖𝑠 𝑜𝑑𝑑 , ⏟


{𝑎 𝑎𝑏 𝑖𝑠 𝑒𝑣𝑒𝑛}
𝐸𝑣𝑒𝑛𝑡 𝐴 𝐸𝑣𝑒𝑛𝑡 𝐵

𝑎 𝑏 𝑎𝑏
Odd Odd Odd A
Odd Even Even B
Even Odd Even B
Even Even Even B

P a g e 57 | 121
Aziz Manva (azizmanva@gmail.com)

1.185: Complement of an event


The complement of an event represents 𝑎𝑙𝑙 the outcomes which are not in the event.

𝐶𝑜𝑚𝑝𝑙𝑒𝑚𝑒𝑛𝑡 𝑜𝑓𝐴 𝑖𝑠 𝑤𝑟𝑖𝑡𝑡𝑒𝑛 𝑎𝑠 𝐴𝐶

Example 1.186
Consider selecting a day at random from the days of the week. Let
𝐴 = 𝑊𝑒𝑒𝑘𝑑𝑎𝑦 𝑖𝑠 𝑠𝑒𝑙𝑒𝑐𝑡𝑒𝑑
𝐶
What is the complement of A, that is 𝐴 ?

𝐴𝐶 = 𝑊𝑒𝑒𝑘𝑒𝑛𝑑 𝑖𝑠 𝑠𝑒𝑙𝑒𝑐𝑡𝑒𝑑

Example 1.187
Consider rolling a standard, six-sided die. Let
𝑃 = 𝑅𝑜𝑙𝑙 𝑖𝑠 𝑎 𝑝𝑟𝑖𝑚𝑒 𝑛𝑢𝑚𝑏𝑒𝑟
𝐶 = 𝑅𝑜𝑙𝑙 𝑖𝑠 𝑎 𝑐𝑜𝑚𝑝𝑜𝑠𝑖𝑡𝑒 𝑛𝑢𝑚𝑏𝑒𝑟
Is the event 𝑃 the complement of the event C? That is, is true that:
𝑃 = 𝐶𝐶

𝑆𝑎𝑚𝑝𝑙𝑒 𝑆𝑝𝑎𝑐𝑒 = {1,2,3,4,5,6}


𝑃 = {2,3,5} ⇒ 𝑃𝐶 = {1,4,6}
𝐶 = {4,6} ⇒ 𝐶 𝐶 = {1,2,3,5}

From the above,


𝑃 ≠ 𝐶𝐶
That is P is not the complement of the event C.

Why is 𝑃 not the complement of 𝐶?

There are numbers which are neither prime nor composite.


Hence
𝑃 ∪ 𝐶 = {2,3,4,5,6} ≠ {1,2,3,4,5,6}
1.188: Probability of a Complement
𝑃(𝐴) + 𝑃(𝐴𝐶 ) = 1 ⇒ 𝑃(𝐴) = 1 − 𝑃(𝐴𝐶 )

Here, 𝐴 𝑎𝑛𝑑 𝐴𝐶 must be mutually exclusive and exhaustive.

Example 1.189
A gambler wants to cheat with a coin. He wants a heads, and knows that he has a 70% chance of succeeding.
What is the probability that the gambler fails?

7
𝑃(𝐻) = 70% =
10
7 3
𝑃(𝑇) = 1 − 𝑃(𝐻) = 1 − =
10 10

P a g e 58 | 121
Aziz Manva (azizmanva@gmail.com)

Example 1.190
1
An urn has green, red and yellow balls. The probability of picking a green ball at random from the urn is 10.
There are an equal number of red and yellow balls.
A. Find the probability of picking a red ball.
B. Find the minimum number of yellow balls in the urn.

1 9
𝑃(𝑅 𝑂𝑅 𝑌) = 1 − =
10 10
9 1 9
𝑃(𝑅) = × =
10 2 20
The minimum number of yellow balls
=9

Example 1.191
In a history class, the probability of earning an 𝐴 is .7 times the probability of earning a 𝐵, and the probability of
earning a 𝐶 is 1.4 times the probability of earning a 𝐵. Assuming that all grades are 𝐴, 𝐵, or 𝐶, how many 𝐵′𝑠 will
there be in a history class of 31 students? (MathCounts 1991 National Sprint)

Let 𝑏 be the probability of getting a B. Then:


𝑃(𝐴) = 0.7𝑃(𝐵) = 0.7𝑏, 𝑃(𝐶) = 1.4𝑃(𝐵) = 1.4𝑏
Note that all grades are A, B or C, which means that
{𝐴, 𝐵, 𝐶} 𝑖𝑠 𝑒𝑥ℎ𝑎𝑢𝑠𝑡𝑖𝑣𝑒
Note that (from a commonsense assumption), you cannot get two grades at the same time. Hence, the grades
are:
𝑀𝑢𝑡𝑢𝑎𝑙𝑙𝑦 𝐸𝑥𝑐𝑙𝑢𝑠𝑖𝑣𝑒

Hence, since the sum of mutually exclusive and exhaustive events is 1, we must have:
1 10
0.7𝑏
⏟+ ⏟ 𝑏 + 1.4𝑏
⏟ =1⇒𝑏 = = ⇒ 𝑀𝑖𝑛 No. of B ′ s = 10
𝑝(𝐴) 𝑝(𝐵) 𝑝(𝐶)
3.1 31

Example 1.192
A bag contains marbles of five different colours. One marble is chosen at random. The probability of choosing a
brown marble is 0.3. Choosing a brown marble is three times as likely as choosing a purple marble. Choosing a
green marble is equally likely as choosing a purple marble. Choosing a red marble is equally likely as choosing a
yellow marble. The probability of choosing a marble that is either red or green is (Gauss 8 2018/21)

{𝐵𝑟𝑜𝑤𝑛, 𝑃𝑢𝑟𝑝𝑙𝑒, 𝐺𝑟𝑒𝑒𝑛, 𝑅𝑒𝑑, 𝑌𝑒𝑙𝑙𝑜𝑤} ⇒ 𝑀𝑢𝑡𝑢𝑎𝑙𝑙𝑦 𝐸𝑥𝑐𝑙𝑢𝑠𝑖𝑣𝑒 & 𝐸𝑥ℎ𝑎𝑢𝑠𝑡𝑖𝑣𝑒

𝑃(𝐵) + 𝑃(𝑃) + 𝑃(𝐺) + 𝑃(𝑅) + 𝑃(𝑌) = 1


0.3
Substitute 𝑃(𝐵) = 0.3, 𝑃(𝐺) = 𝑃(𝑃) = = 0.1, 𝑃(𝑅) = 𝑃(𝑌):
3
0.3 + 0.1 + 0.1 + 2 ∗ 𝑃(𝑅) = 1 ⇒ 𝑃(𝑅) = 0.25

Since the probabilities are mutually exclusive, I can add the probabilities:
𝑃(𝑅 𝑜𝑟 𝐺) = 𝑃(𝑅) + 𝑃(𝐺) = 0.25 + 0.1 = 0.35

Example 1.193
A complete cycle of a traffic light takes 60 seconds. During each cycle the light is green for 25 seconds, yellow

P a g e 59 | 121
Aziz Manva (azizmanva@gmail.com)

for 5 seconds, and red for 30 seconds. At a randomly chosen time, what is the probability that the light will NOT
be green? (AMC 8 1999/10)

We use the concept of complementary probability:


25 35 7
𝑃(𝐺 𝐶 ) = 1 − 𝑃(𝐺𝑟𝑒𝑒𝑛) = 1 − = =
60 60 12

We can also do this using addition of probabilities because yellow and red are mutually exclusive:
5 30 35 7
𝑃(𝐺𝑟𝑒𝑒𝑛) = 𝑃(𝑅𝑒𝑑) + 𝑃(𝑌𝑒𝑙𝑙𝑜𝑤) = + = =
60 60 60 12

Example 1.194
A board game spinner is divided into three regions labeled 𝐴, 𝐵 and 𝐶. The probability of the arrow stopping on
1 1
region 𝐴 is 3 and on region B is 2. The probability of the arrow stopping on region 𝐶 is: (AMC 8 2002/12)

1 1 1
𝑃(𝐶) = 1 − 𝑃(𝐶 𝐶 ) = 1 − 𝑃(𝐴) − 𝑃(𝐵) = 1 − − =
3 2 6
Example 1.195
Sum greater than two
Sum greater than three

Two dice are tossed. What is the probability that the sum is greater than two?

1 - 1/36=35/36

Example 1.196
Product greater than one
Product greater than two

G. Geometrical Counting

Example 1.197
(AMC 10B 2004/23)

The total number of outcomes is


2 × 2 × … × 2 = 26 = 64

For successful outcomes, we consider cases.

Case I: All faces are a single color

P a g e 60 | 121
Aziz Manva (azizmanva@gmail.com)

In this case, the cube can be placed so that the condition is met.
𝐴𝑙𝑙 𝑅𝑒𝑑: 1 𝐶𝑎𝑠𝑒
𝐴𝑙𝑙 𝐵𝑙𝑢𝑒: 1 𝐶𝑎𝑠𝑒

Case II: Exactly one face of one color


The face which is of a single can be placed face down on the horizontal surface. This case can happen in:
1 𝑅𝑒𝑑, 5 𝐵𝑙𝑢𝑒: 6 𝑊𝑎𝑦𝑠
1 𝐵𝑙𝑢𝑒, 5 𝑅𝑒𝑑: 6 𝑊𝑎𝑦𝑠

Case III: Exactly two faces of one color


If we have two faces of one color, then the two faces which have the same
color need to be opposite each other so that the four vertical faces are the
same color.

Consider the diagram alongside with faces numbered 1 to 6. The successful


cases where two faces are opposite each other are:
(1,3), (2,4), (5,6)
This gives:
2 𝑅𝑒𝑑, 4 𝐵𝑙𝑢𝑒: 3 𝑊𝑎𝑦𝑠
2 𝐵𝑙𝑢𝑒, 4 𝑅𝑒𝑑: 3 𝑊𝑎𝑦𝑠

Total
= 2(1 + 6 + 3) = 2(10) = 20

The probability is:


20 5
=
64 16

1.6 Multiplication Rule-I: Tree Diagrams


A. Independent Outcomes

1.198: Independent Outcomes


If the occurrence of one event does not affect the occurrence of another event, then the events are called
independent.

If one event causes a change in the probability of another then, the two events are not independent.
Specifically
➢ drawing without replacement is dependent.
➢ Drawing with replacement is independent
✓ With the assumption, of course, that the cards are shuffled, etc.

Example 1.199: Identifying Independent versus Non-Independent Events


Identify the following events as independent or dependent.
A. Roll a fair die twice
1
B. Toss an unfair coin with probability of landing on heads = 3 thrice
C. Draw three cards from a well-shuffled pack of cards with replacement
D. Draw a ball from Urn A containing 5 red and 3 green balls. If the ball is red, draw a ball from Urn B
containing 7 yellow and 4 black balls. If the ball is green, draw a ball from Urn C containing 3 blue and 8

P a g e 61 | 121
Aziz Manva (azizmanva@gmail.com)

white balls.

𝐼𝑛𝑑𝑒𝑝𝑒𝑛𝑑𝑒𝑛𝑡: 𝐴, 𝐵, 𝐶 𝑎𝑛𝑑 𝐷

1.200: Drawing with and without replacement


When you draw without replacement, the outcomes are dependent.
When you draw with replacement, the outcomes are independent.

Example 1.201
Draw two balls (with replacement) from an urn containing five red and three green balls.

Part B
Consider the probability that the cards are Aces.
The probability that the first card is an Ace is
4 1
=
52 13

Example 1.202
Show that drawing two cards (without replacement) from a pack of cards is not independent by considering the
probability that the second card is an Ace.

If you get an ace on the first card


You do not replace the first card. The number of cards becomes
52 − 1 = 51
The number of Aces becomes
4−1=3

The probability that the second card is an Ace becomes


4−1 3
=
52 − 1 51

If you do not get an ace on the first card


You do not replace the first card. The number of cards becomes
52 − 1 = 51
The number of Aces remains
4−0=4

The probability that the second card is an Ace becomes


4 4
=
52 − 1 51

3 4

51 51
The probability for the second card being an Ace depends on whether the first card is an Ace or not.
Hence, the probabilities are dependent.

1.203: Multiplication Rule: Independent Events


𝑃(𝐴 𝑎𝑛𝑑 𝐵) = 𝑃(𝐴 ∩ 𝐵) = 𝑃(𝐴)𝑃(𝐵)

P a g e 62 | 121
Aziz Manva (azizmanva@gmail.com)

The probability of two independent events happening together is the product of their probabilities.
B. Tossing Coins

1.204: Tree Diagrams


Tree diagrams are used to represent branching probabilities.
They are one of the most powerful tools to calculate probabilities.

➢ For complicated questions, it may not be reasonable to draw a tree diagram because of the size of the
diagram.

Example 1.205
A fair coin is tossed. Show the possible outcomes and their probabilities using a tree diagram.

1
𝑃(𝐻𝑒𝑎𝑑𝑠) =
2
1
𝑃(𝑇𝑎𝑖𝑙𝑠) =
2

1.206: Tree Diagrams


Tree diagrams show all the probabilities in the sample space.
An important to recognize the branch (or branches) that meet the condition for the event that you are
interested in.

Example 1.207
A fair coin is tossed twice. What is the probability of getting:
A. A head on both the tosses
B. A tail on both the tosses
C. Different outcomes in the two tosses
D. Same outcomes in the two tosses

Tree Diagram
𝑆𝑎𝑚𝑝𝑙𝑒 𝑆𝑝𝑎𝑐𝑒 = {(𝐻𝐻)(𝐻𝑇)(𝑇𝐻)(𝑇𝑇)}

1
𝑃(𝐻, 𝐻) =
4
1
𝑃(𝑇, 𝑇) =
4
Probability of one head and one tail
1 1 2 1
= 𝑃(𝐻, 𝑇) + 𝑃(𝑇, 𝐻) = + = =
4 4 4 2

Probability of same outcomes


1 1 2 1
= 𝑃(𝐻, 𝐻) + 𝑃(𝑇, 𝑇) = + = =
4 4 4 2

Sample Space
Head on both the tosses
= (𝑯𝑯)(𝐻𝑇)(𝑇𝐻)(𝑇𝑇)
Tail on both the tosses

P a g e 63 | 121
Aziz Manva (azizmanva@gmail.com)

= (𝐻𝐻)(𝐻𝑇)(𝑇𝐻)(𝑻𝑻)
Different outcomes in the two tosses
= (𝐻𝐻)(𝑯𝑻)(𝑻𝑯)(𝑇𝑇)
Same outcome in the two tosses
= (𝑯𝑯)(𝐻𝑇)(𝑇𝐻)(𝑻𝑻)

1.208: Multiplication Rule: Dependent Events


𝑃(𝐴 𝑎𝑛𝑑 𝐵) = 𝑃(𝐴 ∩ 𝐵) = 𝑃(𝐴)𝑃(𝐵|𝐴)

where
𝑃(𝐵|𝐴) 𝑖𝑠 𝑡ℎ𝑒 𝑝𝑟𝑜𝑏𝑎𝑏𝑖𝑙𝑖𝑡𝑦 𝑡ℎ𝑎𝑡 𝐵 𝑤𝑖𝑙𝑙 ℎ𝑎𝑝𝑝𝑒𝑛 𝑔𝑖𝑣𝑒𝑛 𝑡ℎ𝑎𝑡 𝐴 ℎ𝑎𝑠 𝑎𝑙𝑟𝑒𝑎𝑑𝑦 ℎ𝑎𝑝𝑝𝑒𝑛𝑒𝑑

Example 1.209: Dependent Events


I draw two cards from a pack of cards, without replacement, and I check their suit. What is the probability that:
A. Both cards are spades
B. None of the cards is a spade
C. One card is a spade, and one card is not a spade

Both cards are spades:


13 12
= × =
52 51
Both cards are not spades:
39 38
= × =
52 51
One card is a spade, and the other is not:
13 39 39 13
× + ×
52 51 52 51

C. Unfair Coins

Example 1.210
1
An unfair coin when tossed, shows up heads with probability 4. The coin is tossed twice. Find the probability of:
A. Two Heads
B. Two Tails
C. One Head and One Tail

P a g e 64 | 121
Aziz Manva (azizmanva@gmail.com)

1 1 1
𝑃(𝐻𝐻) = 𝑃(𝐻)𝑃(𝐻) = ( ) ( ) =
4 4 16
3 3 9
𝑃(𝑇𝑇) = 𝑃(𝑇)𝑃(𝑇) = ( ) ( ) =
4 4 16

Probability of one head and one tail is:


= 𝑃(𝐻𝑇) + 𝑃(𝑇𝐻)
= 𝑃(𝐻)𝑃(𝑇) + 𝑃(𝑇)𝑃(𝐻)
1 3 3 1 3 3
= ( )( ) + ( )( ) = 2 × =
4 4 4 4 16 8

1.211: Structuring the problem


Breaking a problem down into steps in a tree diagram is a crucial skill.

For example, if you are tossing a coin, then the outcome of the first toss, gives you your first set of branches. The
outcome of the second coin toss gives you your second set of branches.

Example 1.212
3
Joy and Thomas attempt a math question independently. The probability that Joy solves the question is , and
4
5
the probability that Thomas solves the question is 6. Find the probability that:
A. Both solve the question
B. Neither solves the question
C. At least one solves the question
D. Either one of Joy or Thomas solves the question, but not both

3 1 5 1
𝑃(𝐽) = ⇒ 𝑃(𝐽′) = , 𝑃(𝑇) = ⇒ 𝑃(𝑇′) =
4 4 6 6
Part A
Probability that both solve the question
3 5 15 5
= 𝑃(𝐽 𝑎𝑛𝑑 𝑇) = 𝑃(𝐽)𝑃(𝑇) = × = =
4 6 24 8
Part B
Probability that neither solves the question
3 5 1 1 1
= 𝑃(𝐽′ 𝑎𝑛𝑑 𝑇 ′ ) = 𝑃(𝐽′ )𝑃(𝑇 ′ ) = (1 − ) (1 − ) = × =
4 6 4 6 24
Part C
𝐽 = 𝐽𝑜𝑦 𝑠𝑜𝑙𝑣𝑒𝑠, 𝑇 = 𝑇ℎ𝑜𝑚𝑎𝑠 𝑆𝑜𝑙𝑣𝑒𝑠
⏟ 𝐽′ 𝑇, 𝐽𝑇 ′ ,
𝐽𝑇, ′
𝐽⏟ 𝑇′
𝐴𝑡 𝑙𝑒𝑎𝑠𝑡 𝑜𝑛𝑒 𝑠𝑜𝑙𝑣𝑒𝑠 𝑁𝑜𝑛𝑒 𝑠𝑜𝑙𝑣𝑒𝑠
Probability that at least one person solves the question:
1 23
= 1 − 𝑃(𝑁𝑜 𝑂𝑛𝑒 𝑆𝑜𝑙𝑣𝑒𝑠) = 1 − =
24 24
Part D
Probability that either one solves, but not both:

P a g e 65 | 121
Aziz Manva (azizmanva@gmail.com)

1 5 3 1 5 3 8 1
𝑃(𝐽′ 𝑎𝑛𝑑 𝑇) + 𝑃(𝐽 𝑎𝑛𝑑 𝑇′) = ( ) ( ) + ( ) ( ) = + = =
4 6 4 6 24 24 24 3

D. Urns with replacement

1.213: Drawing from an Urn with replacement


When you are drawing balls from an urn, and replacing each ball after drawing it, drawing a ball of one type
does not affect the color of the second ball. In other words, the events are independent.

Example 1.214
Sarn has an urn containing three red and two green balls. He draws a ball from the urn, replaces it, and then
draws another ball. Find the probability of getting:
A. Two Red Balls
B. Two Green Balls
C. Two Balls of Different Colours

3 2
𝑃(𝑅) = , 𝑃(𝐺) =
5 5

Probability of two red balls


9
= 𝑃(𝑅𝑅) =
25
Probability of two green balls
4
= 𝑃(𝐺𝐺) =
25
Probability of one ball of each color:
6 6 12
= 𝑃(𝑅𝐺) + 𝑃(𝐺𝑅) = + =
25 25 25

The same calculations can be done using a table.

The drawing is with replacement, so each time the balls are drawn, there is no change in the probability.

3 2
𝑃(𝑅𝑒𝑑) = 𝑟 = ( ) 𝑃(𝐺𝑟𝑒𝑒𝑛) = 𝑔 = ( )
5 5
3 2
3 3 9 3 2 6
𝑃(𝑅𝑒𝑑) = 𝑟 ( ) 𝑟 = × = 𝑔𝑟 = × =
5 5 5 25 5 5 25
2 3 2 6 2 2 4
𝑃(𝐺𝑟𝑒𝑒𝑛) = 𝑔 ( ) 𝑟𝑔 = × = 𝑔2 = × =
5 5 5 25 5 5 25

Both Red Both Green Green + Red


9 4 12
25 25 25

They can be done using binomial expansions.

Let
𝑟 = 𝑝𝑟𝑜𝑏𝑎𝑏𝑖𝑙𝑖𝑡𝑦 𝑜𝑓 𝑐ℎ𝑜𝑜𝑠𝑖𝑛𝑔 𝑎 𝑟𝑒𝑑 𝑏𝑎𝑙𝑙

P a g e 66 | 121
Aziz Manva (azizmanva@gmail.com)

𝑔 = 𝑝𝑟𝑜𝑏𝑎𝑏𝑖𝑙𝑖𝑡𝑦 𝑜𝑓 𝑐ℎ𝑜𝑜𝑠𝑖𝑛𝑔 𝑎 𝑔𝑟𝑒𝑒𝑛 𝑏𝑎𝑙𝑙


Note that since the above events are mutually exclusive and exhaustive:
3 2 5
𝑟+𝑔 = + = = 1
5 5 5
We can create the solution for the above question by squaring both sides of the above identity:
(𝑟 + 𝑔)2 = 𝑟 2 + 2𝑟𝑔 + 𝑔2 = 1

We can substitute the values of 𝑟 and 𝑔 into the above expression to get the needed probabilities:
2
3 2 3 2 3 2 2 2 9 12 4
12 = ( + ) = ( ) + 2( )( ) + ( ) = + +

5 ⏟
5 ⏟5 ⏟5 5 ⏟5 25 25 25
𝑹𝒆𝒅 𝑮𝒓𝒆𝒆𝒏 𝑹𝒆𝒅+𝑹𝒆𝒅 𝑹𝒆𝒅 + 𝑮𝒓𝒆𝒆𝒏 𝑮𝒓𝒆𝒆𝒏+𝑮𝒓𝒆𝒆𝒏
𝑮𝒓𝒆𝒆𝒏+ 𝑹𝒆𝒅

1.215: Binomial Expansions


Binomial expansion can be useful in situations where tree diagrams have too many branches to draw.

Example 1.216
Sarn has an urn containing three red and two green balls. He draws a ball from the urn and replaces it. He
repeats this process three times.
A. Three red balls
B. Two red balls
C. One red ball
D. Three green balls

𝑟𝑟𝑔, 𝑟𝑔𝑟, 𝑔𝑟𝑟

(𝑟 + 𝑔)3 = 𝑟⏟3 + 3𝑟
⏟ 2𝑔 + 3𝑟𝑔2
⏟ ⏟3 = 1
+ 𝑔
𝑇ℎ𝑟𝑒𝑒 𝑇𝑤𝑜 𝑅𝑒𝑑 𝑇𝑤𝑜 𝐺𝑟𝑒𝑒𝑛 𝑇ℎ𝑟𝑒𝑒
𝑅𝑒𝑑 𝑂𝑛𝑒 𝐺𝑟𝑒𝑒𝑛 𝑂𝑛𝑒 𝑅𝑒𝑑 𝐺𝑟𝑒𝑒𝑛

𝑇ℎ𝑟𝑒𝑒 𝑅𝑒𝑑 = 𝑟 3
𝑇𝑤𝑜 𝑅𝑒𝑑 = 3𝑟 2 𝑔 =
𝑂𝑛𝑒 𝑅𝑒𝑑 = 3𝑟𝑔2 =
𝑇ℎ𝑟𝑒𝑒 𝑔𝑟𝑒𝑒𝑛 = 𝑔3 =
E. Mendelian Genetics

1.217: Applications in Biology


One of the important applications of probability is in genetics.

Many real life features of organisms are controlled by a single gene. For example, sickle cell anemia depends on
a single set of genes.

Example 1.218
Suppose that the color of your eyes is controlled by a single gene. Offspring get two genes, one from each of
their parents. The gene for black color 𝑑𝑜𝑚𝑖𝑛𝑎𝑡𝑒𝑠 the gene for brown color. If for a single gene, based on the
population, an offspring has a 70% chance to get a gene for black color, and there are only two possible genes in
the population, then determine:
A. The probability that the offspring will have black eyes.

P a g e 67 | 121
Aziz Manva (azizmanva@gmail.com)

B. The probability that the offspring will have brown eyes.

(𝐵𝑙𝑎𝑐𝑘 + 𝐵𝑟𝑜𝑤𝑛)2 = 𝐵𝑙𝑎𝑐𝑘 2 + 2(𝐵𝑙𝑎𝑐𝑘)(𝐵𝑟𝑜𝑤𝑛) + 𝐵𝑟𝑜𝑤𝑛2

Substitute
𝑃(𝐵𝑙𝑎𝑐𝑘 𝑔𝑒𝑛𝑒) = 0.7
𝑃(𝐵𝑟𝑜𝑤𝑛 𝑔𝑒𝑛𝑒) = 1 − 0.7 = 0.3

To get:
0.72 + 2(0.7)(0.3) + 0.32 = 0.49
⏟ + 0.42
⏟ + 0.09

𝑩𝒍𝒂𝒄𝒌 𝑩𝒍𝒂𝒄𝒌 𝑩𝒓𝒐𝒘𝒏

𝑃(𝐵𝑟𝑜𝑤𝑛) = 0.09

Example 1.219
{𝐵𝐵, 𝐵𝑏, 𝑏𝐵} 𝑖𝑠 𝐵𝑟𝑜𝑤𝑛, {𝑏𝑏}𝑖𝑠 𝐵𝑙𝑢𝑒
Brown genes (𝐵) are dominant over blue genes(𝑏) .
A. Alice has brown eyes. Jack has blue eyes. Jack's genes are bb. Alice's genes are BB.
B. Pam has brown eyes, Bob has blue eyes. Bob's genes are bb. Pam's genes are Bb.

Part A
(B+B)(b+b) =Bb+Bb+Bb+Bb =4Bb=Brown
Ratio: 100%

Part B
(B+b)(b+b) =Bb+Bb+b^2+b^2=2Bb+2b^2
Bb=Brown
Ratio: 50%
b^2 =bb=Blue
Ratio: 50

Part C
(B+b)(B+b)=(B+b)^2=B^2 + 2Bb + b^2
BB=25%
Bb=50%
b^2=bb=25%

Example 1.220: Weather


1 2
The probability that it will rain today is 3. The probability that it will rain tomorrow is 3. Whether it rains today
has no effect on whether it rains tomorrow. That is, the probabilities are independent. Draw a tree diagram
illustrating these possibilities. Use it to calculate the probability of:
A. Raining both days
B. Raining neither of the days
C. Raining exactly one day
D. Raining at least one day
E. Raining at most one day

P a g e 68 | 121
Aziz Manva (azizmanva@gmail.com)

Draw the tree diagram first. Let


𝑅 = 𝑅𝑎𝑖𝑛
𝑁 = 𝑁𝑜 𝑅𝑎𝑖𝑛
Once we have all the “branches”, then we can
calculate the probability of two events
happening together.1

The probability of it raining:


2
𝐵𝑜𝑡ℎ 𝐷𝑎𝑦𝑠 = 𝑅𝑅 =
9
2
𝑁𝑒𝑖𝑡ℎ𝑒𝑟 𝐷𝑎𝑦 = 𝑁𝑁 =
9
1 4 5
𝐸𝑥𝑎𝑐𝑡𝑙𝑦 1 𝐷𝑎𝑦 = 𝑅𝑁 + 𝑁𝑅 = + =
9 9 9
The next two probabilities are best done using
complementary probability:

Part D
𝑃(𝐴𝑡 𝑙𝑒𝑎𝑠𝑡 𝑜𝑛𝑒 𝑑𝑎𝑦) = 𝑃(𝑅 = 1) + 𝑃(𝑅 = 2) = 𝑃(𝑅𝑁) + 𝑃(𝑁𝑅) + 𝑃(𝑅𝑅)

We can write this in terms of complementary probability as:


2 7
𝑃(𝐴𝑡 𝑙𝑒𝑎𝑠𝑡 1 𝐷𝑎𝑦) = 1 − 𝑃(𝑁𝑁) = 1 − =
9 9

Part E
2 7
𝐴𝑡 𝑚𝑜𝑠𝑡 1 𝑑𝑎𝑦 = 1 − 𝑅𝑅 = 1 − =
9 9

Example 1.221
A. There are 5 blue chips and 3 yellow chips in a bag. One chip is drawn from the bag. That chip is placed
back into the bag. A second chip is then drawn. What is the probability that the two selected chips are of
different colors? Express your answer as a common fraction. (MathCounts 2004 Warm-Up 15)
B. Suppose that the chips are not replaced after drawing. Find the probability that the chips are of two
different colours?

Part A
Two different colors can be:
𝐵𝑙𝑢𝑒 + 𝑌𝑒𝑙𝑙𝑜𝑤(𝐵𝑌), 𝑌𝑒𝑙𝑙𝑜𝑤 + 𝐵𝑙𝑢𝑒(𝑌𝐵)

And the probabilities we want are:


15 15 30 15
𝑃(𝐵𝑌) + 𝑃(𝑌𝐵) = + = =
64 64 64 32
Part B
Two different colors can be:
𝐵𝑙𝑢𝑒 + 𝑌𝑒𝑙𝑙𝑜𝑤(𝐵𝑌), 𝑌𝑒𝑙𝑙𝑜𝑤 + 𝐵𝑙𝑢𝑒(𝑌𝐵)

And the probabilities we want are:

1The “independence” mentioned in the question is important. That is what lets us multiply the two probabilities. If the two events are
not independent, we can still calculate the probabilities, but the formula is more complicated.

P a g e 69 | 121
Aziz Manva (azizmanva@gmail.com)

5 3 3 5 15 15
𝑃(𝐵𝑌) + 𝑃(𝑌𝐵) = × + × = 2( ) =
8 7 8 7 56 28

Example 1.222
Regions 𝐴, 𝐵, 𝐶, 𝐽 and 𝐾 represent ponds. Logs leave pond 𝐴 and float down flumes
(represented by arrows) to eventually end up in pond 𝐵 or pond 𝐶. On leaving a pond, the
logs are equally likely to use any available exit flume. Logs can only float in the direction
the arrow is pointing. What is the probability that a log in pond 𝐴 will end up in:
A. Pond J
B. pond B?
C. pond C?
Express all answers as a common fraction. (MathCounts 2004 State Sprint, Adapted)

Part A
As the question says, all logs eventually end up in pond B or pond C. Hence,
the probability of ending in pond J is:
𝑍𝑒𝑟𝑜
Part B
There are two paths that go from pond A to pond B.
The first path is 𝐴 − 𝐾 − 𝐵
1 1 1
× =

3 ⏟
2 6
𝑨−𝑲 𝑲−𝑩
The second path is 𝐴 − 𝐽 − 𝐵:
1 1 1
× =

3 ⏟
3 9

𝑨−𝑱 𝑱−𝑩

The total probability is


1 1 3 2 5
= + = + =
6 9 18 18 18
Part C
Use Complementary Probability:
5 13
1 − 𝑃(𝐵) = 1 − =
18 18
First Path is 𝐴 − 𝐶
1
3
Second Path is 𝐴 − 𝐾 − 𝐶
1 1 1
× =
3 2 6
Second Path is 𝐴 − 𝐽 − 𝐶
1 2 2
× =
3 3 9
Since the log can take any of the paths, we add the probabilities:
1 1 2 6 3 4 13
+ + = + + =
3 6 9 18 18 18 18

P a g e 70 | 121
Aziz Manva (azizmanva@gmail.com)

(Alternate Solution) Example 1.223


Regions A, B, C, J and K represent ponds. Logs leave pond A and float down
flumes (represented by arrows) to eventually end up in pond B or pond C.
On leaving a pond, the logs are equally likely to use any available exit flume.
Logs can only float in the direction the arrow is pointing. What is the
probability that a log in pond A will end up in:
A. Pond J
B. pond B?
C. pond C?
Express all answers as a common fraction. (MathCounts 2004 State Sprint,
Adapted)

Using the tree diagram already above, we can imagine a number of logs at
pond A. Since our final answer has a denominator of 18, we choose 18 logs.

Then, the number of logs that reach each pond can be shown in the diagram
alongside.

F. Paths on a Grid

Example 1.224
Frieda the frog begins a sequence of hops on a 3 × 3 grid of squares, moving one square on each hop and
choosing at random the direction of each hop-up, down, left, or right. She does not hop diagonally. When the
direction of a hop would take Frieda off the grid, she "wraps around" and jumps to the opposite edge. For
example, if Frieda begins in the center square and makes two hops "up", the first hop would place her in the top
row middle square, and the second hop would cause Frieda to jump to the opposite edge, landing in the bottom
row middle square. Suppose Frieda starts from the center square, makes at most four hops at random, and
stops hopping if she lands on a corner square. What is the probability that she reaches a corner square on one
of the four hops? (AMC 10A Spring 2021/23, AMC 12A Spring 2021/23)

Method I: Tree Diagrams C E C


Using the notation 𝐶 = 𝐶𝑜𝑟𝑛𝑒𝑟, 𝐸 = 𝐸𝑑𝑔𝑒, 𝑀 = 𝑀𝑖𝑑𝑑𝑙𝑒, we get the classification E M E
C E C
Frieda starts in the middle. In Hop 1, there are exactly four positions that she can move to, and they are all edge
positions.

1
𝐶( )
4
Frog 1 1
𝑀( ) 𝐸( )
4 4
1
𝐶( )
4

P a g e 71 | 121
Aziz Manva (azizmanva@gmail.com)

Irrespective of which position you move to in the first hop, the positions are all symmetrical. Hence,
𝑤𝑖𝑡ℎ𝑜𝑢𝑡 𝑙𝑜𝑠𝑠 𝑜𝑓 𝑔𝑒𝑛𝑒𝑟𝑎𝑙𝑖𝑡𝑦, assume that Frieda moved to first column, second row.

1 1 1 1 1 1 1 1 16 4 4 1 25
+ ∙ + ∙ + ∙ ∙ = + + + =
⏟ ⏟ ⏟ ⏟
2 4 2 4 2 4 4 2 32 32 32 32 32
𝐶1 𝐶2 𝐶3 𝐶4

Method II: Listing Paths


1 1 1 C E C
𝑃(𝑀 → 𝐸) = 1, 𝑃(𝐸 → 𝐶) = , 𝑃(𝐸 → 𝑀) = , 𝑃(𝐸 → 𝐸) =
2 4 4 E M E
C E C
Think of the possible paths that end in a Corner.
𝑆𝑡𝑎𝑟𝑡 𝑝𝑜𝑠𝑖𝑡𝑖𝑜𝑛 = 𝑀, 𝐿𝑎𝑠𝑡 𝑝𝑜𝑠𝑖𝑡𝑖𝑜𝑛 = 𝐶, 2𝑛𝑑 − 𝑙𝑎𝑠𝑡 𝑝𝑜𝑠𝑖𝑡𝑖𝑜𝑛 = 𝐸
Paths of Length 2:
1 1
𝑃(𝑀𝐸𝐶) = 1 × =
2 2
Path of Length 3:
1 1 1
𝑃(𝑀𝐸𝐸𝐶) = 1 × × =
4 2 8
Path of Length 4:
1 1 1
𝑃(𝑀𝐸𝑀𝐸𝐶) = 1 × × 1 × =
4 2 8
1 1 1 1
𝑃(𝑀𝐸𝐸𝐸𝐶) = 1 × × × =
4 4 2 32

1 1 1 1 25
+ + + =
2 8 8 32 32

1.7 Multiplication Rule-II: No Tree Diagrams


A. Repetition

1.225: Repetition of Events


If an event occurs with probability 𝑝, then the probability that the event will occur 𝑛 times (given

P a g e 72 | 121
Aziz Manva (azizmanva@gmail.com)

independence) is
𝑝 ∙ 𝑝 ∙ … ∙ 𝑝 = 𝑝𝑛

𝒏 𝒕𝒊𝒎𝒆𝒔

Example 1.226
Joe's batting average is .323. (That is, he averages 0.323 hits per at-bat.) What is the probability that he will get
three hits in three at-bats? Express your answer in exponent form. (MathCounts 2001 Workout 8)

This is like tossing an unfair coin.


𝑃(𝐻, 𝐻, 𝐻) = 0.323 × 0.323 × 0.323 = (0.323)3

Example 1.227
Assuming that the birth of a boy or a girl is equally likely, what is the probability that the three children in a
family include at least one boy and one girl? Express your answer as a common fraction. (MathCounts 2003
State Countdown)

Method I: Complementary Probability


This can be done with complementary probability. At least one boy and one girl means you cannot have all boys,
or all girls.
1 3 1
𝑃(𝐴𝑙𝑙 𝐵𝑜𝑦𝑠) = ( ) =
2 8
1 3 1
𝑃(𝐴𝑙𝑙 𝐺𝑖𝑟𝑙𝑠) = ( ) =
2 8
The probability of at least one girl and one boy
1 1 1 3
=1− − =1− =

8 ⏟
8 4 4
𝑷(𝑨𝒍𝒍 𝑩𝒐𝒚𝒔) 𝑷(𝑨𝒍𝒍 𝑮𝒊𝒓𝒍𝒔)

Method II: Sample Space


{𝐵𝐵𝐵, 𝐵𝐵𝐺, 𝐵𝐺𝐵, 𝐵𝐺𝐺, 𝐺𝐵𝐵, 𝐺𝐵𝐺, 𝐺𝐺𝐵, 𝐺𝐺𝐺}

{𝐵𝐵𝐵, 𝐵𝐵𝐺, 𝐵𝐺𝐵, 𝐵𝐺𝐺, 𝐺𝐵𝐵, 𝐺𝐵𝐺, 𝐺𝐺𝐵, 𝐺𝐺𝐺}

6
𝑃=
8

Example 1.228
A standard six-sided is rolled twice. Find the probability that:
A. Two sixes are thrown.
B. Two even numbers are thrown.
C. The same number is thrown twice.
D. The two numbers thrown are different.

Part A
1 1 1
𝑃(6,6) = 𝑃(6)𝑃(6) = ( ) ( ) =
6 6 36
Part B
1 1 1
𝑃(𝐸 𝑎𝑛𝑑 𝐸) = 𝑃(𝐸)𝑃(𝐸) = ( ) ( ) =
2 2 4
Part C

P a g e 73 | 121
Aziz Manva (azizmanva@gmail.com)

𝑃(1,1) + 𝑃(2,2) + 𝑃(3,3) + 𝑃(4,4) + 𝑃(5,5) + 𝑃(6,6)


1 1 1 1 1 1 6 1
= + + + + + = =
36 36 36 36 36 36 36 6

Alternate Method
To get the same number, the first number does not matter. The second number should match the first number,
which will happen with probability.
1
6
Part D
Method I: Complementary Probability

1 5
𝑃(𝐷𝑖𝑓𝑓𝑒𝑟𝑒𝑛𝑡) = 1 − 𝑃(𝑆𝑎𝑚𝑒) = 1 − =
6 6
Method II: Casework
𝑃(1, 𝑁𝑜𝑡 1) + 𝑃(2, 𝑁𝑜𝑡 2) + 𝑃(3, 𝑁𝑜𝑡 3) + 𝑃(4, 𝑁𝑜𝑡 4) + 𝑃(5𝑁𝑜𝑡 ,5) + 𝑃(6, 𝑁𝑜𝑡 6)
1 5 1 5 1 5 1 5 1 5 1 5 1 5 5
= ( )( ) + ( )( ) + ( )( ) + ( )( ) + ( )( ) + ( )( ) = 6( )( ) =
6 6 6 6 6 6 6 6 6 6 6 6 6 6 6

Method III: Symmetry


The first number can be any number. The second number will not match the first with probability
5
6
B. Dice

Example 1.229
A fair nine-sided dice with faces numbered 1 to 9 is rolled twice. The numbers on the rolls are 𝑎 and 𝑏.
What is the probability that:
A. 𝑎 and 𝑏 are both prime
B. 𝑎 and 𝑏 are both composite
C. exactly one of 𝑎 and 𝑏 is prime?
D. at least one of 𝑎 and 𝑏 is prime?

𝑃𝑟𝑖𝑚𝑒𝑠 = {2,3,5,7}
𝐶𝑜𝑚𝑝𝑜𝑠𝑖𝑡𝑒 = {4,6,8,9}
𝑁𝑜𝑡 𝑝𝑟𝑖𝑚𝑒 = {1,4,6,8,9}
Part A

4 4 16
𝐵𝑜𝑡ℎ 𝑃𝑟𝑖𝑚𝑒: × =
9 9 81
Part B
{4,6,8,9}
4 4 16
𝐵𝑜𝑡ℎ 𝐶𝑜𝑚𝑝𝑜𝑠𝑖𝑡𝑒: × =
9 9 81
Part C
4 5
𝑃1 = 𝑃(𝑝𝑟𝑖𝑚𝑒, 𝑛𝑜𝑡 𝑝𝑟𝑖𝑚𝑒) = ×
9 9
5 4
𝑃2 = 𝑃(𝑛𝑜𝑡 𝑝𝑟𝑖𝑚𝑒, 𝑝𝑟𝑖𝑚𝑒) = ×
9 9

P a g e 74 | 121
Aziz Manva (azizmanva@gmail.com)

4 5 40
𝑃1 + 𝑃2 = 2 ( × ) =
9 9 81
Part D
Method I: Complementary Counting
5 5 25 56
1 − 𝑃(𝑁𝑜𝑛𝑒 𝑎𝑟𝑒 𝑃𝑟𝑖𝑚𝑒) = 1 − ( × ) = 1 − =
9 9 81 81

Method II: Direct Method


40 16 56
𝑃(𝐴𝑡 𝑙𝑒𝑎𝑠𝑡 𝑜𝑛𝑒 𝑝𝑟𝑖𝑚𝑒) = + =

81 ⏟
81 81
𝑬𝒙𝒂𝒄𝒕𝒍𝒚 𝑷( 𝑩𝒐𝒕𝒉 )
𝑷( ) 𝑷𝒓𝒊𝒎𝒆
𝑶𝒏𝒆 𝑷𝒓𝒊𝒎𝒆

Challenge 1.230
Clara rolls three fair six-sided dice. She subtracts one from each roll that she gets, and then multiplies the
numbers. What is the probability that her product is zero?

Method I: Complementary Counting


Strategy
Let the numbers on the rolls be 𝑥, 𝑦 and 𝑧. As per the condition given:
(𝑥 − 1)(𝑦 − 1)(𝑧 − 1) = 0
At least one of 𝑥,𝑦 and 𝑧 must be 1.
Complementary Counting
We calculate the probability of getting not a single 1.
5 5 5 125
𝑃(𝑁𝑜𝑛𝑒 𝑖𝑠 1) = ( ) ( ) ( ) =
6 6 6 216

And the probability that we want is:


125 91
𝑃(𝐴𝑡 𝑙𝑒𝑎𝑠𝑡 1) = 1 − 𝑃(𝑁𝑜𝑛𝑒 𝑖𝑠 1) = 1 − =
216 216

Method II: Direct Method


Case 1: All three dice roll a 1:
1 3 1
𝑃1 = ( ) =
6 216
Case 2: Exactly one die rolls a 1:
1 5 5 25
𝑃(𝑂𝑛𝑙𝑦 1𝑠𝑡 𝐷𝑖𝑒 = 1) = ( ) ( ) ( ) =
6 6 6 216
5 1 5 25
𝑃(𝑂𝑛𝑙𝑦 2𝑛𝑑 𝐷𝑖𝑒 = 1) = ( ) ( ) ( ) =
6 6 6 216
5 5 1 25
𝑃(𝑂𝑛𝑙𝑦 3𝑟𝑑 𝐷𝑖𝑒 = 1) = ( ) ( ) ( ) =
6 6 6 216

25 75
𝑃2 = 3 × =
216 216

Case 3: Exactly two dice roll a 1:


1 1 5 5
𝑃(𝐸𝑥𝑎𝑐𝑡𝑙𝑦 1𝑠𝑡 𝑎𝑛𝑑 2𝑛𝑑 𝐷𝑖𝑒 = 1) = ( ) ( ) ( ) =
6 6 6 216
5 1 1 5
𝑃(𝐸𝑥𝑎𝑐𝑡𝑙𝑦 2𝑛𝑑 𝑎𝑛𝑑 3𝑟𝑑 𝐷𝑖𝑒 = 1) = ( ) ( ) ( ) =
6 6 6 216

P a g e 75 | 121
Aziz Manva (azizmanva@gmail.com)

1 5 1 5
𝑃(𝐸𝑥𝑎𝑐𝑡𝑙𝑦 1𝑠𝑡 𝑎𝑛𝑑 3𝑟𝑑 𝐷𝑖𝑒 = 1) = ( ) ( ) ( ) =
6 6 6 216

5 15
𝑃3 = 3 × =
216 216

1 75 15 91
𝑃1 + 𝑃2 + 𝑃3 = + + =
216 216 216 216

Method III: Using Permutations and Combinations


Case 1: All three dice roll a 1:
1 3 1
𝑃1 = ( ) =
6 216
Case 2: Exactly one die rolls a 1:

3 1 5 5 25 75
𝑃2 = ( ) × 𝑃(1 𝐷𝑖𝑒 𝑟𝑜𝑙𝑙𝑠 𝑎 1) = ( ) ( ) ( ) = 3 × =
1 6 6 6 216 216

Case 3: Exactly two dice roll a 1:


3 1 1 5 15
𝑃3 = ( ) × 𝑃(𝐸𝑥𝑎𝑐𝑡𝑙𝑦 2 𝐷𝑖𝑒 𝑟𝑜𝑙𝑙𝑠 𝑎 1) = 3 ( ) ( ) ( ) =
2 6 6 6 216

1 75 15 91
𝑃1 + 𝑃2 + 𝑃3 = + + =
216 216 216 216

Challenge 1.231
Clara rolls 𝑛 fair six-sided dice. She subtracts one from each roll that she gets, and then multiplies the numbers.
What is the probability that her product is zero?

Strategy
Let the numbers on the rolls be 𝑥, 𝑦 and 𝑧. As per the condition given:
(𝑥 − 1)(𝑦 − 1)(𝑧 − 1) = 0
At least one of 𝑥,𝑦 and 𝑧 must be 1.
Complementary Counting
We calculate the probability of getting not a single 1.
5 𝑛
𝑃(𝑁𝑜𝑛𝑒 𝑖𝑠 1) = ( )
6
And the probability that we want is:
5 𝑛 6𝑛 − 5𝑛
𝑃(𝐴𝑡 𝑙𝑒𝑎𝑠𝑡 1) = 1 − 𝑃(𝑁𝑜𝑛𝑒 𝑖𝑠 1) = 1 − ( ) =
6 6𝑛

Example 1.232
A. A standard die is rolled six times. What is the probability that the product of all six rolls is odd? Express
your answer as a common fraction. (MathCounts 2002 National Countdown)
B. In the above, what is the probability that the product is even?

Part A
For the product to be odd, all rolls must be odd:
1 6 1
𝑃=( ) =
2 64

P a g e 76 | 121
Aziz Manva (azizmanva@gmail.com)

Part B
The product can be either odd or even. If it is not odd, then it is even. Calculate using complementary
probability:
1 63
1 − 𝑃(𝑂𝑑𝑑) = 1 − =
64 64

Example 1.233
What is the probability that if we roll 4 fair 6-sided dice, at most 3 of them will show a 4?

We are ok with
𝑂𝑛𝑒 4, 𝑇𝑤𝑜 4′ 𝑠, 𝑇ℎ𝑟𝑒𝑒 4′ 𝑠 𝑏𝑢𝑡 𝑓𝑜𝑢𝑟 4′ 𝑠

𝑃(𝑍𝑒𝑟𝑜 4′ 𝑠) + 𝑃(𝑂𝑛𝑒 4) + 𝑃(𝑇𝑤𝑜 4′ 𝑠) + 𝑃(𝑇ℎ𝑟𝑒𝑒 4′ 𝑠) + 𝑃(𝐹𝑜𝑢𝑟 4′ 𝑠) = 1


𝑃(𝑍𝑒𝑟𝑜 4′ 𝑠) + 𝑃(𝑂𝑛𝑒 4) + 𝑃(𝑇𝑤𝑜 4′ 𝑠) + 𝑃(𝑇ℎ𝑟𝑒𝑒 4′ 𝑠) = 1 − 𝑃(𝐹𝑜𝑢𝑟 4′ 𝑠)

1 4 1 1295
𝑃(𝐴𝑡 𝑚𝑜𝑠𝑡 3 𝑤𝑖𝑙𝑙 𝑠ℎ𝑜𝑤 4) = 1 – 𝑃 (𝐴𝑙𝑙 4 𝑤𝑖𝑙𝑙 𝑠ℎ𝑜𝑤 4) = 1 – ( ) = 1 – =
6 1296 1296

Example 1.234
What is the probability that if we roll 𝑛 fair 6-sided dice, at most 𝑛 − 1 of them will show a 4?

1 𝑛 1
𝑃(𝐴𝑡 𝑚𝑜𝑠𝑡 3 𝑤𝑖𝑙𝑙 𝑠ℎ𝑜𝑤 4) = 1 – 𝑃 (𝐴𝑙𝑙 4 𝑤𝑖𝑙𝑙 𝑠ℎ𝑜𝑤 4) = 1 – ( ) = 1 – 𝑛
6 6

Example 1.235
A fair coin is tossed thrice. What is the probability of getting:
A. A head on all the tosses
B. Same outcomes in all three tosses
C. Exactly two consecutive heads
D. At least two consecutive heads

𝑆𝑎𝑚𝑝𝑙𝑒 𝑆𝑝𝑎𝑐𝑒 = {(𝐻𝐻𝐻)(𝐻𝐻𝑇)(𝐻𝑇𝐻)(𝐻𝑇𝑇)(𝑇𝐻𝐻)(𝑇𝐻𝑇)(𝑇𝑇𝐻)(𝑇𝑇𝑇)}


Part A
1
(𝑯𝑯𝑯)(𝐻𝐻𝑇)(𝐻𝑇𝐻)(𝐻𝑇𝑇)(𝑇𝐻𝐻)(𝑇𝐻𝑇)(𝑇𝑇𝐻)(𝑇𝑇𝑇) =
8

1 1 1 1
× × =

2 ⏟
2 ⏟
2 8
𝐹𝑖𝑟𝑠𝑡 𝑆𝑒𝑐𝑜𝑛𝑑 𝑇ℎ𝑖𝑟𝑑
𝑇𝑜𝑠𝑠 𝑇𝑜𝑠𝑠 𝑇𝑜𝑠𝑠

Part B
2 1
(𝑯𝑯𝑯)(𝐻𝐻𝑇)(𝐻𝑇𝐻)(𝐻𝑇𝑇)(𝑇𝐻𝐻)(𝑇𝐻𝑇)(𝑇𝑇𝐻)(𝑻𝑻𝑻) = =
8 4
1 1 1

1 × × =
𝑭𝒊𝒓𝒔𝒕 𝑻𝒐𝒔𝒔:

2 ⏟
2 4
𝑵𝒐 𝑹𝒆𝒔𝒕𝒓𝒊𝒄𝒕𝒊𝒐𝒏𝒔 𝑺𝒆𝒄𝒐𝒏𝒅 𝑻𝒉𝒊𝒓𝒅
𝑻𝒐𝒔𝒔 𝑻𝒐𝒔𝒔

Bogus Solution for Part C

P a g e 77 | 121
Aziz Manva (azizmanva@gmail.com)

3𝐶2 𝑁𝑜. 𝑜𝑓 𝑊𝑎𝑦𝑠 𝑜𝑓 𝑆𝑒𝑙𝑒𝑐𝑡𝑖𝑛𝑔 2 𝐻𝑒𝑎𝑑𝑠 𝑜𝑢𝑡 𝑜𝑓 3 3


= =
8 𝑇𝑜𝑡𝑎𝑙 𝑊𝑎𝑦𝑠 8
This also includes
𝐻𝑇𝐻
As a valid selection.
And hence, the final answer is more than the correct answer.
Part C
2 1
(𝐻𝐻𝐻)(𝑯𝑯𝑻)(𝐻𝑇𝐻)(𝐻𝑇𝑇)(𝑻𝑯𝑯)(𝑇𝐻𝑇)(𝑇𝑇𝐻)(𝑇𝑇𝑇) = =
8 4

The probability of two consecutive heads is


1 1 1
× =
2 2 4
Since we want exactly two heads, we also need a tail to go with the two heads, which has probability
1
2
And, we have two with two consecutive heads (𝑯𝑯𝑻), (𝑻𝑯𝑯), and the hence the final probability is:
1 1 1
× × ×2
2 2 2
Part D
3
(𝑯𝑯𝑯)(𝑯𝑯𝑻)(𝐻𝑇𝐻)(𝐻𝑇𝑇)(𝑻𝑯𝑯)(𝑇𝐻𝑇)(𝑇𝑇𝐻)(𝑇𝑇𝑇) =
8
To use the multiplication rule, we need to do some casework
2 1 3
+ =

8 ⏟
8 8
𝑃𝑎𝑟𝑡 𝐶 𝑃𝑎𝑟𝑡 𝐴

Example 1.236
Keiko tosses one penny and Ephraim tosses two pennies. What is the probability that Ephraim gets the same
number of heads that Keiko gets? Express your answer as a common fraction. (AMC 8 2000/21)

Zero One Two Total


Keiko 1 1 0 1
2 2
Ephraim 1 1 1 1 1 1 1 1 1 1
× = 2( × ) = × =
2 2 4 2 2 2 2 2 4
1 1 1 1 1 1 0 3
× = × = 𝐴𝑛𝑠 =
2 4 8 2 2 4 8

C. Spinners

Example 1.237
A spinner is divided into five equal parts: two blue, one green and two yellow. It is spun twice, what is the
probability of getting two different colours?

4 1 4 9 16
⏟− [𝑃(𝐵𝐵) + 𝑃(𝐺𝐺) + 𝑃(𝑌𝑌)] = 1 − (
𝑃(𝐷𝑖𝑓𝑓𝑒𝑟𝑒𝑛𝑡 𝐶𝑜𝑙𝑜𝑢𝑟𝑠) = 1 + + ) =1− =
𝑪𝒐𝒎𝒑𝒍𝒆𝒎𝒆𝒏𝒕𝒂𝒓𝒚 𝑪𝒐𝒖𝒏𝒕𝒊𝒏𝒈
25 25 25 25 25
𝑺𝒖𝒃𝒕𝒓𝒂𝒄𝒕 𝒕𝒉𝒆 𝒑𝒓𝒐𝒃𝒂𝒃𝒊𝒍𝒊𝒕𝒚 𝒐𝒇 𝒔𝒂𝒎𝒆 𝒄𝒐𝒍𝒐𝒖𝒓

P a g e 78 | 121
Aziz Manva (azizmanva@gmail.com)

Example 1.238
A spinner is divided into five equal parts: two blue, one green and two yellow. It is spun twice, what is the
probability of getting blue on at least one spin?

2
𝑃(𝐵𝑋) = 𝐵𝑙𝑢𝑒 𝑜𝑛 𝑓𝑖𝑟𝑠𝑡 𝑠𝑝𝑖𝑛 =
5
2
𝑃(𝑋𝐵) = 𝐵𝑙𝑢𝑒 𝑜𝑛 𝑠𝑒𝑐𝑜𝑛𝑑 𝑠𝑝𝑖𝑛 =
5
2 2 4
𝑃(𝐵𝐵) = 𝐵𝑙𝑢𝑒 𝑜𝑛 𝑏𝑜𝑡ℎ 𝑠𝑝𝑖𝑛𝑠 = × =
5 5 25
2 2 4 4 4 20 4 16
𝑃(𝐵𝑋) + 𝑃(𝑋𝐵) − 𝑃(𝐵𝐵) = + − = − = − =
5 5 25 5 25 25 25 25

Counting spins which are both Blue


Name the equal parts
{𝐵1 , 𝐵2 , 𝐺, 𝑌1 , 𝑌2 }
Total Outcomes
= 5 × 5 = 25
Outcomes where both spins give a blue result are
{𝐵1 𝐵1 , 𝐵2 𝐵2 , 𝐵1 𝐵2 , 𝐵2 𝐵1 }
D. Real Life Scenarios

Example 1.239
Jar A has exactly four red buttons and eight blue buttons. Carla then removes the same number of red buttons as
2
blue buttons from Jar A and places them in an empty Jar B. Jar A now has 3 of its original number of buttons. If
Carla were now to randomly choose a button from Jar A and a button from Jar B, what is the probability that the
two chosen buttons will both be red? Express your answer as a common fraction. (MathCounts 2004 National
Sprint)

The total number of buttons in Jar A A B


= 4 + 8 = 12 Red Blue Red Blue
The number of buttons remaining Original 4 8
2 New 2 6 2 2
= × 12 = 8
3
The number of buttons removed
= 12 − 8 = 4
These buttons are
2 𝑅𝑒𝑑, 2 𝐵𝑙𝑢𝑒

The probability of selecting red from both Jar A and Jar B is:
2 2 1
× =
8 4 8

Example 1.240
A. The 600 students at King Middle School are divided into three groups of equal size for lunch. Each group
has lunch at a different time. A computer randomly assigns each student to one of three lunch groups.
The probability that three friends, Al, Bob, and Carol, will be assigned to the same lunch group is
approximately (AMC 8 1986/24)
B. Three houses are available in a locality. Three persons apply for the houses. Each applies for one house

P a g e 79 | 121
Aziz Manva (azizmanva@gmail.com)

without consulting others. The probability that all the three apply for the same house is: (JEE Main
2005)

Part A
Without loss of generality, assign Al to any lunch group. This happens with probability
1
The probability that Bob gets assigned to the same lunch group as Al is:
1
3
The probability that Carol gets assigned to the same lunch group as Al is also:
1
3
The probability then that all three get assigned to the same lunch group is:
1 1 1
1× × =
3 3 9
Part B
Note that the scenario is different, but the question is same (including the numbers).
1
𝑃=
9

(Calc) Example 1.241


In Ms. Smith's class, each student averages one day absent out of thirty. What is the probability that out of any
two students chosen at random, one student will be absent while the other is present? Express your answer as a
percent rounded to the nearest tenth. (MathCounts 1994 Workout 4)

Let the students be 𝑋 and Y. The probabilities are:


29
𝑋(𝑃𝑟𝑒𝑠𝑒𝑛𝑡) = 𝑌(𝑃𝑟𝑒𝑠𝑒𝑛𝑡) =
30
1
𝑋(𝐴𝑏𝑠𝑒𝑛𝑡)
⏟ = 𝑌(𝐴𝑏𝑠𝑒𝑛𝑡) =

30
𝑋

We want exactly one of the two students to be present.


We have four cases in all, out of which we want the probability of two cases:

X is Present X is Absent
29 1
𝑋= 𝑋’ =
30 30
Y is Present 29 29 29 29 2 1 29 29
𝑌= 𝑋𝑌 = × =( ) 𝑋’𝑌 = × =
30 30 30 30 30 30 900
Y is absent 1 29 1 29 1 1 1
𝑌’ = 𝑋𝑌’ = × = 𝑋’𝑌’ = × =
30 30 30 900 30 30 900

29 1 58
2× × = ≈ 6.4%
30 30 900
E. Complementary Probability

Example 1.242
4 3
The probability that A speaks truth is while the probability for B is . The probability that they contradict each
5 4
other when asked to speak on a fact is: (JEE Main 2004)

P a g e 80 | 121
Aziz Manva (azizmanva@gmail.com)

Note: For this kind of question, we consider that a statement is either truthful(𝑇) or a lie (𝐿). We do not
distinguish between different possible lies.

Probability that both speak the truth


4 3 12
= × =
5 4 20

Probability that both lie


4 3 1 1 1
= (1 − ) (1 − ) = ( ) ( ) =
5 4 5 4 20

Probability that both say the same thing


12 1 13
= + =
20 20 20

Probability that they say different things


13 7
=1 − =
20 20

Example 1.243
1 3 1 1
A. If four students independently solve a certain problem correctly with probabilities , , , and
2 4 4 8
respectively, then the probability that the problem is solved correctly by at least one of them. (JEE
Advanced 2013)
1 11 1
B. Four persons can hit a target correctly with probabilities 2, 3,4 and 8 respectively. If all hit at the target
independently, then the probability that the target would be hit, is: (JEE Main, 2019, April 09)
C. A problem in mathematics is given to three students 𝐴, 𝐵, 𝐶 and their respective probability of solving
1 1 1
the problem is 2, 3 and 4. Probability that the problem is solved is: (JEE Main 2002)

Part A
Let the students be 𝑠1 , 𝑠2 , 𝑠3 , 𝑠4 . The probability that a student solves the question correctly is:
1 3 1 1
𝑃(𝑠1 ) = , 𝑃(𝑠2 ) = , 𝑃(𝑠3 ) = , 𝑃(𝑠4 ) =
2 4 4 8
Using complementary probability, the probability that an individual student does not solve the question
correctly is:
1 1 3 1 1 3 1 7
𝑃(𝑠 ′ 1 ) = 1 − = , 𝑃(𝑠 ′ 2 ) = 1 − = , 𝑃(𝑠 ′ 3 ) = 1 − = , 𝑃(𝑠 ′ 4 ) = 1 − =
2 2 4 4 4 4 8 8
The sample space can be divided into two mutually exclusive and exhaustive events:
{𝑁𝑜𝑛𝑒 𝑆𝑜𝑙𝑣𝑒𝑠, 𝐴𝑡 𝐿𝑒𝑎𝑠𝑡 𝑂𝑛𝑒 𝑆𝑜𝑙𝑣𝑒𝑠}
Again, using complementary probability:
1 1 3 7 21 235
𝑃(𝐴𝑡 𝐿𝑒𝑎𝑠𝑡 1) = 1 – 𝑃(𝑁𝑜𝑛𝑒) = 1 – ( × × × ) = 1 − =
2 4 4 8 256 256
Part B
1 2 3 7 7 25
𝑃(𝐴𝑡 𝐿𝑒𝑎𝑠𝑡 1) = 1 – 𝑃(𝑁𝑜𝑛𝑒) = 1 – ( × × × ) = 1 − =
2 3 4 8 32 32
Part C
1 2 3 1 3
𝑃(𝐴𝑡 𝐿𝑒𝑎𝑠𝑡 1) = 1 – 𝑃(𝑁𝑜𝑛𝑒) = 1 – ( × × ) = 1 − =
2 3 4 4 4

P a g e 81 | 121
Aziz Manva (azizmanva@gmail.com)

Example 1.244: Repetition


The minimum number of times one has to toss a fair coin must be tossed so that the probability of getting at
least one head is
A. at least 90% is (JEE Main, 2019, April 8)
B. more than 99% is: (JEE Main, 2019, April 10)

Part A
9
𝑃(𝐴𝑡 𝑙𝑒𝑎𝑠𝑡 𝑜𝑛𝑒 ℎ𝑒𝑎𝑑) > 90% =
10
9 1
𝑃(𝑁𝑜 𝐻𝑒𝑎𝑑𝑠) < 1 − =
10 10
The probability of getting no heads in 𝑛 tosses is:
1 1 1 1 𝑛 1𝑛 1
× × …× 2 =( ) = 𝑛= 𝑛
⏟2 ⏟
2 ⏟
2 2 2 2
𝑵𝒐 𝑯𝒆𝒂𝒅 𝑵𝒐 𝑯𝒆𝒂𝒅 𝑵𝒐 𝑯𝒆𝒂𝒅
𝒊𝒏 𝟏𝒔𝒕 𝑻𝒐𝒔𝒔 𝒊𝒏 𝟐𝒏𝒅 𝑻𝒐𝒔𝒔 𝒊𝒏 𝒏𝒕𝒉 𝑻𝒐𝒔𝒔

1 1
𝑛
<
2 10
Take the reciprocal both sides, and reverse the direction of the inequality3:
2𝑛 > 10
2 = 1,2 = 4,2 = 8 < 10, 24 = 16 > 10
1 2 3

𝑛≥4
Part B
Similarly:
1 1
𝑛
< ⇒ 2𝑛 > 100 ⇒ 𝑛 ≥ 7
2 100

F. Combining Multiplication and Addition Rules

Example 1.245
A. 𝐴, 𝐵, 𝐶 try to hit a target simultaneously but independently. Their respective probabilities of hitting the
315
targets are 4,2,8. The probability that the target is hit by 𝐴 or 𝐵, but not by 𝐶 is: (JEE Main 2013, April
23)
B. Three persons 𝑃, 𝑄 and 𝑅 independently try to hit a target. If the probabilities of their hitting the target
3 1 5
are 4, 2 and 8 respectively, then the probability that the target is hit by 𝑃 or 𝑄, but not by 𝑅, is:(JEE Main
2017, April 8)

Part A
The probability that at least one of A or B hits the target4
3 1 3 1 6+4−3 7
= 𝑃(𝐴 ∪ 𝐵) = + − × = =
4 2 4 2 8 8

2 The three dots her … which indicate continuation are called an 𝑒𝑙𝑙𝑖𝑝𝑠𝑖𝑠.
1 1
3 Note that when we take the reciprocal of positive quantities, the sign of the inequality is reversed. For example: > ⇔ 2 < 3. In
2 3
𝑎 𝑐 𝑏 𝑑
general, for positive 𝑎, 𝑏, 𝑐, 𝑑 if < ⇔ >
𝑏 𝑑 𝑎 𝑐
4 The OR here is the default OR, not exclusive OR, which would have the formula
𝐸𝑥𝑐𝑙𝑢𝑠𝑖𝑣𝑒 𝑂𝑅(𝐴, 𝐵) = 𝑛(𝐴)
⏟ + 𝑛(𝐵)
⏟ − 2𝑛(𝐴 ∩ 𝐵)

𝑬𝒍𝒆𝒎𝒆𝒏𝒕𝒔 𝒊𝒏 𝑨 𝑬𝒍𝒆𝒎𝒆𝒏𝒕𝒔 𝒊𝒏 𝑩 𝑬𝒍𝒆𝒎𝒆𝒏𝒕𝒔 𝒊𝒏 𝑰𝒏𝒕𝒆𝒓𝒔𝒆𝒄𝒕𝒊𝒐𝒏
𝒐𝒇 𝑨 𝒂𝒏𝒅 𝑩

P a g e 82 | 121
Aziz Manva (azizmanva@gmail.com)

The probability that 𝐴 or B hit, but 𝐶 does not hit, is given by:
7 5 7 3 21
𝑃(𝐴 ∪ 𝐵) ∩ 𝑃(𝐶 ′ ) = × (1 − ) = × =
8 8 8 8 64
Part B
This is the same as above
21
64

Example 1.246
I pick two natural numbers less than or equal to 100. The numbers are not necessarily distinct. What is the
probability that their HCF is exactly 16?

Case I
The probability that the first number is a multiple of 16, but not 32, is given by:
3
𝑃{16,48,80} =
100
Then, for the HCF to be 16, the second number can be any multiple of 16:
6 Total
𝑃{16,32,48,64,80,96} =
100 16 32 48 64 80 96
The probability of both happening at the same time will be given 16
by: 32
3 6 18 48
× =
100 100 10,000 64
80
Case II
96
The probability that the number is a multiple of 32 is given by:
3
𝑃{32,64,96} =
100
And then the second number must be a multiple of 16, but not 32:
3
𝑃{16,48,96} =
100
The probability of both happening at the same time will be given by:
6 3 18
× =
100 100 10,000
The final answer is:
18 18 36
+ =
10,000 10,000 10,000

Example 1.247
An urn has five white balls, four black balls and one grey ball. Sophia draws a ball at random from the urn and
replaces it. She then draws another ball from the urn. What is the probability that the two balls have different
colors?

Method I: Complementary Probability


We do this using complementary probability. The probability that the two balls have two different colors is:
25 16 1 42 58 29
1 − 𝑃(𝑆𝑎𝑚𝑒 𝐶𝑜𝑙𝑜𝑟𝑠) = 1 − ( + + )=1− = =
100 100 100 100 100 50

Method II: Adding Probabilities

P a g e 83 | 121
Aziz Manva (azizmanva@gmail.com)

5 4 5 1 4 1 20 5 4 29 29
2( × + × + × ) = 2( + + ) = 2( )=
10 10 10 10 10 10 100 100 100 100 50

White Black Grey


5 4 1
10 10 10
White 5 25
10 100
Black 4 16
10 100
Grey 1 1
10 100

5 4 1
With 𝑤 = ,𝑏 = ,𝑔 =
10 10 10
𝑤+𝑏+𝑔 =1
Square both sides:
(𝑤 + 𝑏 + 𝑔)2 = 𝑤 2 + 𝑏 2 + 𝑔2 + 2(𝑤𝑏 + 𝑏𝑔 + 𝑤𝑔)

Challenge 1.248
Disha rolls two six-sided dice. What is the probability that one of the numbers is even, and the other number is a
multiple of three?

Case I: First number is even and second number is a Pair of Numbers


multiple of three B. 1 2 3 4 5 6
3 2 6 1 (1,1) (2,1) (3,1) (4,1) (5,1) (6,1)
× =

6 ⏟
6 36 2 (1,2) (2,2) (3,2) (4,2) (5,2) (6,2)
𝑬𝒗𝒆𝒏 𝑴𝒖𝒍𝒕𝒊𝒑𝒍𝒆 (1,3) (2,3) (3,3) (4,3) (5,3) (6,3)
3
𝒐𝒇 𝑻𝒉𝒓𝒆𝒆
4 (1,4) (2,4) (3,4) (4,4) (5,4) (6,4)
Case II: Second number is even and first number is a
5 (1,5) (2,5) (3,5) (4,5) (5,5) (6,5)
multiple of three
2 3 6 6 (1,6) (2,6) (3,6) (4,6) (5,6) (6,6)
× =
⏟6 ⏟
6 36
𝑴𝒖𝒍𝒕𝒊𝒑𝒍𝒆 𝑬𝒗𝒆𝒏
𝒐𝒇 𝑻𝒉𝒓𝒆𝒆
However, the two cases are not distinct. Hence, we must also find the probability that both cases hold, which is
given by:
1 1 1
× =

6 ⏟6 36
𝑬𝒗𝒆𝒏 𝑴𝒖𝒍𝒕𝒊𝒑𝒍𝒆 𝑬𝒗𝒆𝒏 𝑴𝒖𝒍𝒕𝒊𝒑𝒍𝒆
𝒐𝒇 𝑻𝒉𝒓𝒆𝒆 𝒐𝒇 𝑻𝒉𝒓𝒆𝒆
And, by the addition rule, the final probability is:
6 6 1 11
+ − =
36 36 36 36

1.8 Parity and Divisibility


A. Parity

1.249: Parity
Parity is the concept of odd and even.

P a g e 84 | 121
Aziz Manva (azizmanva@gmail.com)

An integer which is divisible by 2 is an even number. All others numbers are odd numbers.

Note 1: Divisible means divisible with no remainder.


Note 2: 3 ℎ𝑎𝑠 𝑜𝑑𝑑 𝑝𝑎𝑟𝑖𝑡𝑦, 4 ℎ𝑎𝑠 𝑒𝑣𝑒𝑛 𝑝𝑎𝑟𝑖𝑡𝑦, 6 ℎ𝑎𝑠 𝑒𝑣𝑒𝑛 𝑝𝑎𝑟𝑖𝑡𝑦
Note 3: 0 𝑖𝑠 𝑒𝑣𝑒𝑛

Parity has very deep applications, and it can be used to solve very difficult questions.

1.250: Random Walks

Example 1.251
An ant is crawling along the coordinate plane. Each move, it moves one unit up, down, left, or right with equal
probability. If it starts at (0, 0), what is the probability that it will be at either (2, 1) or (1, 2) after 6 moves?
(CCA Math Bonanza, Individual Round, 2020/1)

(0,0) → (2,1)
(𝐸𝑣𝑒𝑛, 𝐸𝑣𝑒𝑛) → (𝐸𝑣𝑒𝑛, 𝑂𝑑𝑑)

For the change in the 𝑥 direction, we must have:



𝐸𝑣𝑒𝑛 + 𝐸𝑣𝑒𝑛
⏟ = 𝐸𝑣𝑒𝑛
𝑂𝑟𝑖𝑔𝑖𝑛𝑎𝑙 𝐶ℎ𝑎𝑛𝑔𝑒
For the change in the 𝑦 direction, we must have:

𝐸𝑣𝑒𝑛 + 𝑂𝑑𝑑
⏟ = 𝑂𝑑𝑑
𝑂𝑟𝑖𝑔𝑖𝑛𝑎𝑙 𝐶ℎ𝑎𝑛𝑔𝑒

The net change must be:


𝐸𝑣𝑒𝑛 + 𝑂𝑑𝑑
⏟ = 𝑂𝑑𝑑
𝑥 𝑑𝑖𝑟𝑒𝑐𝑡𝑖𝑜𝑛 𝑦 𝑑𝑖𝑟𝑒𝑐𝑡𝑖𝑜𝑛
𝐶ℎ𝑎𝑛𝑔𝑒 𝐶ℎ𝑎𝑛𝑔𝑒

The number of moves required to reach the points is odd.


Number of moves available is even.

Hence, probability = 0.

Example 1.252
P a g e 85 | 121
Aziz Manva (azizmanva@gmail.com)

Currently, it is minute zero, and a scorpion is at vertex 𝐴 of hexagon 𝐴𝐵𝐶𝐷𝐸𝐹. At each minute, he crawls with
1
probability 2 to either of the vertices adjacent to him. For example, at minute one, he will be either at 𝐵, or at 𝐹,
1
each with probability . What is the probability that:
2
A. At minute 2020, he will be at vertex 𝐵.
B. At minute 2021, he will be at vertex 𝐴.

We apply the concept of parity.

Part A
To go from A to B requires 1 jump. 1 is an odd number.
To go and come back to B requires even number of jumps.
Overall, the number of steps to go from A to B is odd.

2020 ⇒𝑃=0
𝑬𝒗𝒆𝒏


2021 ⇒ 𝑇𝑜 𝑟𝑒𝑎𝑐ℎ 𝐴 𝑟𝑒𝑞𝑢𝑖𝑟𝑒𝑠 𝑒𝑣𝑒𝑛 𝑛𝑢𝑚𝑏𝑒𝑟 𝑜𝑓 𝑠𝑡𝑒𝑝𝑠 ⇒ 𝑃 = 0
𝑂𝑑𝑑

Example 1.253
Currently, it is minute zero, and a scorpion is at zero on the number line. At each minute, he crawls with
1
probability 2 to either of the vertices adjacent to him. For example, at minute one, he will be either at 1, or at −1,
1
each with probability 2. What is the probability that:
A. At minute 2020, he will be at −1.
B. At minute 2021, he will be at 0.

We apply the concept of parity.



2020 ⇒ 𝑇𝑜 𝑔𝑜 𝐵 𝑟𝑒𝑞𝑢𝑖𝑟𝑒𝑠 𝑜𝑑𝑑 𝑛𝑢𝑚𝑏𝑒𝑟 𝑜𝑓 𝑠𝑡𝑒𝑝𝑠 ⇒ 𝑃 = 0
𝐸𝑣𝑒𝑛

2021 ⇒ 𝑇𝑜 𝑟𝑒𝑎𝑐ℎ 𝐴 𝑟𝑒𝑞𝑢𝑖𝑟𝑒𝑠 𝑒𝑣𝑒𝑛 𝑛𝑢𝑚𝑏𝑒𝑟 𝑜𝑓 𝑠𝑡𝑒𝑝𝑠 ⇒ 𝑃 = 0
𝑂𝑑𝑑

B. Sums

1.254: Parity when adding

𝑶𝒅𝒅 + 𝑬𝒗𝒆𝒏 = 𝑶𝒅𝒅


𝑬𝒗𝒆𝒏 + 𝑶𝒅𝒅 = 𝑶𝒅𝒅
𝑂𝑑𝑑 + 𝑂𝑑𝑑 = 𝐸𝑣𝑒𝑛
𝐸𝑣𝑒𝑛 + 𝐸𝑣𝑒𝑛 = 𝐸𝑣𝑒𝑛

1.255: Sum of Two Numbers


The sum of two numbers
➢ with the same parity is even 𝑂 + 𝑂 = 𝐸, 𝐸 + 𝐸 = 𝐸
➢ with opposite parities is odd: 𝑂 + 𝐸 = 𝑂, 𝐸 + 𝑂 = 𝑂

𝑂+𝑂 =𝐸
𝐿𝐻𝑆 = 𝑂 + 𝑂 = (2𝑛 + 1) + (2𝑚 + 1) = 2𝑛 + 2𝑚 + 2 = 2(𝑛 + 𝑚 + 1) ⇒ 𝐸𝑣𝑒𝑛

𝐸+𝐸 =𝐸

P a g e 86 | 121
Aziz Manva (azizmanva@gmail.com)

𝐿𝐻𝑆 = 𝐸 + 𝐸 = 2𝑛 + 2𝑚 = 2(𝑛 + 𝑚) ⇒ 𝐸𝑣𝑒𝑛

𝑂+𝐸 =𝑂
𝐿𝐻𝑆 = 𝑂 + 𝐸 = (2𝑛 + 1) + (2𝑚) = 2𝑛 + 2𝑚 + 1 = 2(𝑛 + 𝑚) + 1 ⇒ 𝑂𝑑𝑑

Example 1.256
A fair nine-sided dice with faces numbered 1 to 9 is rolled twice. The numbers on the rolls are 𝑎 and 𝑏. What is
the probability that 𝑎 + 𝑏 is odd? is even?

There are a total of 1 2 3 4 5 6 7 8 9


9 × 9 = 81 𝑂𝑢𝑡𝑐𝑜𝑚𝑒𝑠 1 2 3 4 5 6 7 8 9 10
Since: 2 3 4 5 6 7 8 9 10 11
➢ Total number of outcomes is odd 3 4 5 6 7 8 9 10 11 12
➢ First number is even, and last number is even 4 5 6 7 8 9 10 11 12 13
We get 5 6 7 8 9 10 11 12 13 14
40 𝑃𝑎𝑖𝑟𝑠 + 1 𝐸𝑣𝑒𝑛 = 41 𝐸𝑣𝑒𝑛 𝑁𝑢𝑚𝑏𝑒𝑟𝑠 6 7 8 9 10 11 12 13 14 15
41 7 8 9 10 11 12 13 14 15 16
𝑃(𝐸𝑣𝑒𝑛) = 8 9 10 11 12 13 14 15 16 17
81
9 10 11 12 13 14 15 16 17 18
We will calculate the probability of getting an odd number using complementary probability:
41 40
𝑃(𝑂𝑑𝑑) = 1 − 𝑃(𝐸𝑣𝑒𝑛) = 1 − =
81 81

Alternate Solution: Multiplication Rule (Probability)

5 4 4 5 4 5 40
× + × = 2( × ) =

9 ⏟
9 ⏟
9 ⏟
9 9 9 81
𝐹𝑖𝑟𝑠𝑡 𝑅𝑜𝑙𝑙 𝑆𝑒𝑐𝑜𝑛𝑑 𝑅𝑜𝑙𝑙 𝐹𝑖𝑟𝑠𝑡 𝑅𝑜𝑙𝑙 𝑆𝑒𝑐𝑜𝑛𝑑 𝑅𝑜𝑙𝑙
⏟ 𝑂𝑑𝑑 𝐸𝑣𝑒𝑛 ⏟𝐸𝑣𝑒𝑛 𝑂𝑑𝑑
𝑂𝑑𝑑+𝐸𝑣𝑒𝑛 𝐸𝑣𝑒𝑛+𝑂𝑑𝑑

Example 1.257
Every time these two wheels are spun, two numbers are selected by the pointers.
What is the probability that the sum of the two selected numbers is even? (AMC 8
1989/25)

We tabulate the probabilities of the outcomes:


Odd Even
First Spinner 1 1
2 2
Second Spinner 2 1
3 3
Multiplication Rule
The probability that both are even is:
1 2 1 1 1 1 1
× + × = + =

2 3 ⏟
2 3 3 6 2
Both are odd Both are even

Symmetry
We are looking for

P a g e 87 | 121
Aziz Manva (azizmanva@gmail.com)

𝑂𝑑𝑑
⏟ × 𝑂𝑑𝑑
⏟ + 𝐸𝑣𝑒𝑛
⏟ × 𝐸𝑣𝑒𝑛

𝑆𝑒𝑐𝑜𝑛𝑑 𝐹𝑖𝑟𝑠𝑡 𝑆𝑒𝑐𝑜𝑛𝑑 𝐹𝑖𝑟𝑠𝑡
𝑆𝑝𝑖𝑛𝑛𝑒𝑟 𝑆𝑝𝑖𝑛𝑛𝑒𝑟 𝑆𝑝𝑖𝑛𝑛𝑒𝑟 𝑆𝑝𝑖𝑛𝑛𝑒𝑟
1
Observe the first row of the table. It is symmetrical since both odd and even have probability 2. Substitute those
probabilities:
1 1
𝑂𝑑𝑑
⏟ × + 𝐸𝑣𝑒𝑛
⏟ ×
𝑆𝑒𝑐𝑜𝑛𝑑
2 𝑆𝑒𝑐𝑜𝑛𝑑 2
𝑆𝑝𝑖𝑛𝑛𝑒𝑟 𝑆𝑝𝑖𝑛𝑛𝑒𝑟
1
Factor out 2:

1
( 𝑂𝑑𝑑
⏟ + 𝐸𝑣𝑒𝑛
⏟ )
2 𝑆𝑒𝑐𝑜𝑛𝑑 𝑆𝑒𝑐𝑜𝑛𝑑
𝑆𝑝𝑖𝑛𝑛𝑒𝑟 𝑆𝑝𝑖𝑛𝑛𝑒𝑟
But 𝑃(𝑂𝑑𝑑) + 𝑃(𝐸𝑣𝑒𝑛) = 1, and hence, we must have

1 1 1
( 𝑂𝑑𝑑
⏟ + 𝐸𝑣𝑒𝑛
⏟ ) = (1) =
2 𝑆𝑒𝑐𝑜𝑛𝑑 𝑆𝑒𝑐𝑜𝑛𝑑 2 2
𝑆𝑝𝑖𝑛𝑛𝑒𝑟 𝑆𝑝𝑖𝑛𝑛𝑒𝑟

Example 1.258
The two wheels shown below are spun and the two resulting numbers are added.
The probability that the sum is even is (AMC 8 1994/22)

We tabulate the probabilities of the outcomes:


Odd Even
First Spinner 3 1
4 4
Second Spinner 1 2
3 3

3 1 1 2 3 2 5
× + × = + =

4 3 ⏟
4 3 12 12 12
Both are odd Both are even

Example 1.259
Pick one number at random from the set {1,3,4}, and another number at random from the set {2,3,5,6}. Add the
two numbers. What is the probability that the result is even?

We tabulate the probabilities of the outcomes:


Odd Even
First Set 2 1
3 3
Second Set 1 1
2 2

2 1 1 1 1 1 2 1 3 1
× + × = + = + = =

3 2 ⏟
3 2 3 6 6 6 6 2
Both are odd Both are even

Example 1.260: Without Replacement


Ten balls numbered 1 to 10 are in a jar. Jack reaches into the jar and randomly removes one of the balls. Then Jill

P a g e 88 | 121
Aziz Manva (azizmanva@gmail.com)

reaches into the jar and randomly removes a different ball. The probability that the sum of the two numbers on
the balls removed is even is: (AMC 8 1987/25)

The sum will be even in the following two cases:


𝑂𝑑𝑑 + 𝑂𝑑𝑑 = 𝐸𝑣𝑒𝑛
𝐸𝑣𝑒𝑛 + 𝐸𝑣𝑒𝑛 = 𝐸𝑣𝑒𝑛

The balls are drawn without replacement:


5 4 2
𝑃(𝑂𝑑𝑑, 𝑂𝑑𝑑) = × =
10 9 9
5 4 2
𝑃(𝐸𝑣𝑒𝑛, 𝐸𝑣𝑒𝑛) = × =
10 9 9
2 2 4
𝐹𝑖𝑛𝑎𝑙 𝐴𝑛𝑠𝑤𝑒𝑟 = + =
9 9 9
C. Complementary Parity

1.261: Complementary Parity


Just as we can calculate complementary probability, we can also calculate complementary parity.
This is useful in some contexts in reducing calculations.

Example 1.262
Five balls are numbered with the integers 1 through 5 and placed in a jar. Three are drawn without
replacement. What is the probability that the sum of the three integers on the balls is odd? Express your answer
as a common fraction. (MathCounts 2009 State Countdown)

Complementary Parity
Parity equations with three balls are more complicated than parity equation with two balls.
Hence, we consider parity for the balls two balls left behind.

The total value of the five balls is:


1 + 2 + 3 + 4 + 5 = 15 = 𝑂𝑑𝑑

If you split the balls as 𝑇ℎ𝑟𝑒𝑒 𝐵𝑎𝑙𝑙𝑠 + 𝑇𝑤𝑜 𝐵𝑎𝑙𝑙𝑠, the parity will split as:
Case I: Sum of the integers on the three Balls is odd:
𝑂𝑑𝑑
⏟ + 𝐸𝑣𝑒𝑛
⏟ = 𝑂𝑑𝑑
𝑻𝒉𝒓𝒆𝒆 𝑻𝒘𝒐
𝑩𝒂𝒍𝒍𝒔 𝑩𝒂𝒍𝒍𝒔

Case II: Sum of the integers on the three Balls is even:



𝐸𝑣𝑒𝑛 + 𝑂𝑑𝑑
⏟ = 𝑂𝑑𝑑
𝑻𝒉𝒓𝒆𝒆 𝑻𝒘𝒐
𝑩𝒂𝒍𝒍𝒔 𝑩𝒂𝒍𝒍𝒔

In other words, the three balls will have opposite parity to the two balls. Hence, we want to find the probability
of Case I.
𝑂𝑑𝑑
⏟ ⇒ 𝐸𝑣𝑒𝑛 ⏟
𝑻𝒉𝒓𝒆𝒆 𝑻𝒘𝒐
𝑩𝒂𝒍𝒍𝒔 𝑩𝒂𝒍𝒍𝒔

Probability
Selecting three balls without replacement means that we are rejecting two balls without replacement.

P a g e 89 | 121
Aziz Manva (azizmanva@gmail.com)

To calculate the probability that the two remaining balls are even, we consider the two possible cases:
2 1 1
𝐸 + 𝐸 = 𝐸 ⇒ 𝑃(𝐵𝑜𝑡ℎ 𝐸𝑣𝑒𝑛) = × =

5 ⏟
4 10
𝑭𝒊𝒓𝒔𝒕 𝑺𝒆𝒄𝒐𝒏𝒅
𝑩𝒂𝒍𝒍 𝑩𝒂𝒍𝒍
3 2 3
𝑂 + 𝑂 = 𝐸 ⇒ 𝑃(𝐵𝑜𝑡ℎ 𝑂𝑑𝑑) = × =

5 ⏟
4 10
𝑭𝒊𝒓𝒔𝒕 𝑺𝒆𝒄𝒐𝒏𝒅
𝑩𝒂𝒍𝒍 𝑩𝒂𝒍𝒍
The probability that we want is:
1 3 4 2
𝑃(𝐵𝑜𝑡ℎ 𝐸𝑣𝑒𝑛) + 𝑃(𝐵𝑜𝑡ℎ 𝑂𝑑𝑑) = + = =
10 10 10 5
D. Sums of More than Two Numbers

Example 1.263
Four prime numbers are randomly selected without replacement from the first ten prime numbers. What is the
probability that the sum of the four selected numbers is odd? Express your answer as a common fraction.
(MathCounts 2001 Warm-Up 10)

We do this using casework. There is only one even prime number, which is 2. Suppose it is not selected. Then,
the sum will be
𝑂 + 𝑂 + 𝑂 + 𝑂 = 𝐸 + 𝐸 = 𝐸 ⇒ 𝑁𝑜𝑡 𝑉𝑎𝑙𝑖𝑑
If the number 2 is selected, then we have
𝐸
⏟+ 𝑂 + 𝑂⏟+ 𝑂 = 𝑂 + 𝐸 = 𝑂 ⇒ 𝑉𝑎𝑙𝑖𝑑
𝑂 𝐸

Complementary Probability5
Instead of calculating the probability of getting a single even prime, we calculate the probability of getting all
odd primes:
9 8 7 6 6 3
𝑃(𝐴𝑙𝑙 𝑂𝑑𝑑) = × × × = =
10 9 8 7 10 5
3 2
𝑃(1 𝐸𝑣𝑒𝑛) = 1 − 𝑃(𝐴𝑙𝑙 𝑜𝑑𝑑) = 1 − =
5 5

1.264: Parity when adding Three or More Numbers


If an expression has an
➢ Odd number of odd numbers, then the sum is odd.
➢ Even number of odd numbers, then the sum is even

𝑂+𝑂+𝑂 =𝑂
𝐸+𝐸+𝑂 =𝑂

𝐸+𝑂+𝑂 =𝐸

𝐸+𝐸+𝐸 =𝐸

𝐿𝐻𝑆 = 𝐸1 + 𝐸2 + ⋯ + 𝐸𝑘
If there are 𝑘 even numbers, each must of the form 2𝑛, 𝑛 ∈ ℤ. Hence:
= 2𝑛1 + 2𝑛2 + ⋯ + 2𝑛𝑘

5 This can be solved using combinations. For that solution, see the note on Probability with Permutations and Combinations.

P a g e 90 | 121
Aziz Manva (azizmanva@gmail.com)

= 2(𝑛1 + 𝑛2 + ⋯ + 𝑛𝑘 )
And the last expression must be even.

These are not probabilities.

Example 1.265: Three Numbers


Jeff rotates spinners 𝑃, 𝑄 and 𝑅 and adds the resulting
numbers. What is the probability that his sum is an odd
number? (AMC 8 2006/17)

There are two cases which result in an odd number


𝐶𝑎𝑠𝑒 𝐼: 𝑂 + 𝑂 + 𝑂 = 𝑂
𝐶𝑎𝑠𝑒 𝐼𝐼: 𝐸 + 𝐸 + 𝑂 = 𝑂

Since spinner 𝑄 has only even numbers,


𝐶𝑎𝑠𝑒 𝐼 𝑖𝑠 𝑛𝑜𝑡 𝑣𝑎𝑙𝑖𝑑.

For Case II, we want:


𝐸+𝐸+𝑂 =𝑂
Spinner R has only odd numbers, and it contributes the odd number that we want with probability 1.
Spinner Q has only even numbers, and it contributes one of the two even numbers that we want, again with
probability 1.

Spinner P must contribute an even number, which it will provide with


1
𝑃(𝑃 𝑖𝑠 𝑒𝑣𝑒𝑛) =
3

Example 1.266
The wheel shown is spun twice, so that the numbers indicated
by the pointer are randomly determined (with each number on
the wheel being equally likely). The two numbers determined
in this way are recorded. The first number is divided by 4,
determining one of the remainders 1,2,3 marking the columns
of the checkerboard shown. The second number is divided by
5, determining one of the remainders 1,2,3,4 marking the rows
of the checkerboard. Finally, a checker is placed on the square where this column and row meet. What is the
probability that the checker is placed on a shaded square of the checkerboard? (AMC 10B 2007/19)
Approach
1 2 3 6 7 9
Remainder 1 2 3 2 3 1
when
divided 4
Remainder 1 2 3 1 2 4
when
divided 5

When divided by 4:
1
𝑃(1) = 𝑃(2) = 𝑃(3) =
3
When divided by 5:

P a g e 91 | 121
Aziz Manva (azizmanva@gmail.com)

2 1 1
𝑃(1) = 𝑃(2) = = , 𝑃(3) = 𝑃(4) =
6 3 6
4 1 1
6 18
3 1 1 1
6 18 18
2 1 1
3 9
1 1 1 1/9
3 9
1 1 1
3 3 3
1 2 3

1 1 1 1 2 1 3 1
∙3+ ∙3= + = + = =
9 18 3 6 6 6 6 2
E. Products

1.267: Parity when multiplying


𝑶𝒅𝒅 × 𝑶𝒅𝒅 = 𝑶𝒅𝒅
𝑂𝑑𝑑 × 𝐸𝑣𝑒𝑛 = 𝐸𝑣𝑒𝑛
𝐸𝑣𝑒𝑛 × 𝑂𝑑𝑑 = 𝐸𝑣𝑒𝑛
𝐸𝑣𝑒𝑛 × 𝐸𝑣𝑒𝑛 = 𝐸𝑣𝑒𝑛

Note that there is only one case which results in an odd outcome.

Example 1.268
A fair nine-sided dice with faces numbered 1 to 9 is rolled twice. The numbers on the rolls are 𝑎 and 𝑏. What is
the probability that 𝑎𝑏 is odd? is even?

5 5 25
𝑃(𝑎𝑏 𝑖𝑠 𝑜𝑑𝑑) = × =

9 ⏟
9 81
𝑎 𝑖𝑠 𝑜𝑑𝑑 𝑏 𝑖𝑠 𝑜𝑑𝑑
25 56
𝑃(𝑎𝑏 𝑖𝑠 𝑒𝑣𝑒𝑛) = 1 − 𝑃(𝑎𝑏 𝑖𝑠 𝑜𝑑𝑑) = 1 − =
81 81

Example 1.269
Two different numbers are randomly selected from the set −2, −1,0,3,4,5 and multiplied together. What is the
probability that the product is 0? (AMC 8 2016/13)
Complementary Probability
5 4 4 1
1 − 𝑃(𝑁𝑜𝑛 − 𝑧𝑒𝑟𝑜) = 1 − ( ) ( ) = 1 − =
6 5 6 3
Combinations
(51) 5 5×2 2 1
= = = =
(62) 6×5 6×5 6 3
2

P a g e 92 | 121
Aziz Manva (azizmanva@gmail.com)

Permutations
1×5
⏟ + 5×1

𝐹𝑖𝑟𝑠𝑡 𝑁𝑢𝑚𝑏𝑒𝑟 𝑆𝑒𝑐𝑜𝑛𝑑 𝑁𝑢𝑚𝑏𝑒𝑟
𝑍𝑒𝑟𝑜 𝑍𝑒𝑟𝑜 10 1
= =
6×5 30 3

Example 1.270
Spinner I is divided into four equal sections labeled 2, 3, 4 and 5. Spinner II is divided into five equal sections
labeled 1, 3, 5, 7 and 9. If each spinner is spun and the resulting numbers are multiplied, what is the probability
that the product is a two-digit even number? Express your answer as a common fraction. (MathCounts 2009
State Countdown)

1 3 5 7 9
2 10
3
4 12
5

𝑌𝑒𝑙𝑙𝑜𝑤 𝐵𝑜𝑥𝑒𝑠 7
𝑃= =
𝑇𝑜𝑡𝑎𝑙 𝐵𝑜𝑥𝑒𝑠 20

Example 1.271
Each spinner is divided into 3 equal parts. The results obtained from spinning the
two spinners are multiplied. What is the probability that this product is an even
number? (AMC 8 1991/22)

We use complementary probability. The probability of getting an odd number is:


2 1 2
× =
3 3 9
The probability of getting an even number is:
2 7
𝑃(𝐸𝑣𝑒𝑛) = 1 − 𝑃(𝑂𝑑𝑑) = 1 − =
9 9

Example 1.272
Spinners A and B are spun. On each spinner, the arrow is equally likely to
land on each number. What is the probability that the product of the two
spinners' numbers is even? (AMC 8 2004/21)

We use complementary probability. The probability of getting an odd number is:


1 2 1
× =
2 3 3
The probability of getting an even number is:
1 2
𝑃(𝐸𝑣𝑒𝑛) = 1 − 𝑃(𝑂𝑑𝑑) = 1 − =
3 3

Example 1.273
Integers 𝑎, 𝑏, 𝑐, and 𝑑, not necessarily distinct, are chosen independently and at random from 0 to 2007,
inclusive. What is the probability that 𝑎𝑑 − 𝑏𝑐 is even? (AMC 10A 2007/16, AMC 12A 2007/12)

P a g e 93 | 121
Aziz Manva (azizmanva@gmail.com)

1 1 3 3 1 9 10 5
× + × = + = =

4 4 ⏟
4 4 16 16 16 8
𝑩𝒐𝒕𝒉 𝑶𝒅𝒅 𝑩𝒐𝒕𝒉 𝑬𝒗𝒆𝒏

Example 1.274
Una rolls 6 standard 6-sided dice simultaneously and calculates the product of the 6 numbers obtained. What is
the probability that the product is divisible by 4? (AMC 10B Fall 2021 Fall/14, AMC 12B 2021 Fall/11)

We use complementary probability and break into two cases:

Case I: The product is odd


The product will be odd only when all the numbers rolled are odd, which will happen with probability
1 6 1
( ) =
2 64
Case II: The product is an even number not divisible by 4
This requires some rearrangement of the following outcome with exactly one even number that is not a
multiple of 4 (2 𝑜𝑟 6), and the remaining numbers as odd:
(𝐸𝑣𝑒𝑛, 𝑂𝑑𝑑, 𝑂𝑑𝑑, 𝑂𝑑𝑑, 𝑂𝑑𝑑, 𝑂𝑑𝑑)

The even number can in any of the six positions and hence the number of rearrangement is:
6
Hence, the probability from this case is:
1 5 1 1
( ) × ( ) × ⏟
6 =
⏟2 ⏟ 3 16
𝑅𝑒𝑎𝑟𝑟𝑎𝑛𝑔𝑚𝑒𝑛𝑡𝑠
𝑃(𝑂𝑑𝑑) 𝑃(2 𝑜𝑟 4)

Use the addition rule to find the sum of the probability from the two cases:
1 1 5
+ =
64 16 64

And, finally, the complementary probability (of the product being divisible by 4) is:
5 59
= 1 − 𝑃(𝑁𝑜𝑡 𝐷𝑖𝑣𝑖𝑠𝑖𝑏𝑙𝑒) = 1 − =
64 64
F. Sums and Products

Example 1.275
If 𝑎, 𝑏 and 𝑐 are three (not necessarily different) numbers chosen randomly and with replacement from the set
{1,2,3,4,5} what is the probability that 𝑎𝑏 + 𝑐 is even? (AHSME 1995/20)

𝒂𝒃
3 3 9
𝑃(𝑎𝑏 𝑖𝑠 𝑜𝑑𝑑) ⇒ × =

5 ⏟
5 25
𝑎 𝑖𝑠 𝑜𝑑𝑑 𝑏 𝑖𝑠 𝑜𝑑𝑑
9 16
𝑃(𝑎𝑏 𝑖𝑠 𝑒𝑣𝑒𝑛) ⇒ 1 − =
25 25
𝒂𝒃 + 𝒄 is even
9 3 16 2 27 32 59
𝑃(𝑎𝑏 + 𝑐 𝑖𝑠 𝑒𝑣𝑒𝑛) = × + × = + =

25 ⏟
5 ⏟
25 ⏟
5 125 125 125
𝑎𝑏 𝑖𝑠 𝑜𝑑𝑑 𝑐 𝑖𝑠 𝑜𝑑𝑑 𝑎𝑏 𝑖𝑠 𝑒𝑣𝑒𝑛 𝑐 𝑖𝑠 𝑒𝑣𝑒𝑛

P a g e 94 | 121
Aziz Manva (azizmanva@gmail.com)

2. FURTHER TOPICS
2.1 Dependent Outcomes
2.1: Multiplication Rule: General
𝑃 (𝐴 𝑎𝑛𝑑 𝐵) = 𝑃(𝐴|𝐵) 𝑃(𝐵) = 𝑃(𝐴) 𝑃(𝐵|𝐴)

P(A|B) is the probability of A, given that B has occurred


Complement of Conditional Probability
P(A|B) = 1 – P(AC|B)
If A1,...,Ak are mutually exclusive and exhaustive
P(A1|B) +…+ P(Ak|B) = 1

A. Urns without replacement


When you draw a ball from the urn, but do not replace it, the color of the ball drawn will change the
probabilities for the further balls. In other words, the probability of the second ball is dependent on the color of
the first ball.
Hence, the events are not independent.
B. Dependent Probabilities

2.2: Independent Probabilities


Events A and B are called independent if the happening (𝑜𝑟 𝑛𝑜𝑡 ℎ𝑎𝑝𝑝𝑒𝑛𝑖𝑛𝑔) of event A has no effect on the
probability of event B happening.

For example:
➢ Tossing a coin and the probability of it raining tomorrow

2.3: Dependent Probabilities


If the happening (𝑜𝑟 𝑛𝑜𝑡 ℎ𝑎𝑝𝑝𝑒𝑛𝑖𝑛𝑔) of event A makes a difference to the probabilities for event B, the two
events are called dependent.

➢ Dependent probabilities are also called conditional probabilities.

2.4: Notation: Dependent Probabilities


The probability of event 𝐵 happening given that event 𝐴 has already happened is written:

𝑃 (𝐵 ⏟| 𝐴)
𝒈𝒊𝒗𝒆𝒏

And read
𝑃𝑟𝑜𝑏𝑎𝑏𝑖𝑙𝑖𝑡𝑦 𝑜𝑓 𝐵 𝑔𝑖𝑣𝑒𝑛 𝐴

Example 2.5
Ten people (five boys, five girls) are in the selection process for a chess tournament. One of the boys is called
Ralph. One candidate will be selected at random.
A. What is the probability that Ralph gets selected?

P a g e 95 | 121
Aziz Manva (azizmanva@gmail.com)

B. Your friend is in the selection committee. She knows who got selected. She refuses to tell you the name
of the person who got selected but she tells you, "A girl got selected". What is the probability that Ralph
got selected?
C. Your friend is in the selection committee. She knows who got selected. She refuses to tell you the name
of the person who got selected but she tells you, "A boy got selected". What is the probability that Ralph
got selected?

Part A
{𝑏1 , 𝑏2 , 𝑏3 , 𝑏4 , 𝑏𝑅𝑎𝑙𝑝ℎ , 𝑔1 , 𝑔2 , 𝑔3 , 𝑔4 , 𝑔5 }
1
10

Part B
{𝑔1 , 𝑔2 , 𝑔3 , 𝑔4 , 𝑔5 }
0
= 0
10

Part C
{𝑏1 , 𝑏2 , 𝑏3 , 𝑏4 , 𝑏𝑅𝑎𝑙𝑝ℎ }
1
5

Example 2.6
I toss a standard six-sided die. Let 𝐴 be the probability of getting an even number, and 𝐵 be the probability of
getting a prime number.
A. What is 𝑃(𝐴)
B. What is 𝑃(𝐵)
C. If your friend tosses the die and gets an even number. What is the probability that you friend got a prime
number when he tossed the die.

Part A
{1, 𝟐, 3, 𝟒, 5, 𝟔}
3 1
𝑃(𝐴) = =
6 2
Part B
{1, 𝟐, 𝟑, 4, 𝟓, 6}
3 1
𝑃(𝐵) = =
6 2
Part C
The original equiprobable sample space with 6 outcomes was:
{1,2,3,4,5,6}

Since we know that 𝐴 has happened, the sample space reduces to


{𝟐, 4,6}

In the absence of further information, the sample space is still equiprobable:


1
𝑃(𝐵) =
3
The technically correct way to write this is:

P a g e 96 | 121
Aziz Manva (azizmanva@gmail.com)

1
𝑃(𝐵|𝐴) =
3
C. Tree Diagrams
We introduced multiplication of probabilities using tree diagrams. Most of the questions we did using tree
diagrams were based on independent probabilities. However, we can also use tree diagrams to solve questions
on dependent probabilities.

Example 2.7
Sarn has an urn containing five ochre and three magenta balls. He draws two balls from the urn without
replacement. Find the probability that:
A. Both balls are ochre
B. Both balls are magenta
C. One ball is ochre and the other magenta.

5 4 5
𝑃(𝑂𝑂) =
× =
8 7 14
3 2 3
𝑃(𝑀𝑀) = × =
8 7 28
5 3 3 5 5 3 15
𝑃(𝑂𝑀) + 𝑃(𝑀𝑂) = × + × = 2 ( × ) =
8 7 8 7 8 7 28

Example 2.8
Sarn has an urn containing three red and two green balls. He draws
three balls from the urn without replacement. Find the probability that:
A. The three red balls are drawn.
B. The probability that the two green balls are drawn first
C. There is exactly one green ball among the balls drawn

Probability that the three red balls are drawn first is


given by:
3 2 1 1
𝑃(𝑅𝑅𝑅) = × × =
5 4 3 10
Probability that the two green balls are drawn first is
given by:
2 1 1
𝑃(𝐺𝐺) = × =
5 4 10

Probability that exactly one ball is green:


We have three possible cases:
2 3 2 1
𝑃(𝐺𝑅𝑅) = × × =
5 4 3 5
3 2 2 1
𝑃(𝑅𝐺𝑅) = × × =
5 4 3 5
3 2 2 1
𝑃(𝑅𝑅𝐺) × × =
5 4 3 5
1 1 1 3
+ + =
5 5 5 5
D. Drawing Cards

P a g e 97 | 121
Aziz Manva (azizmanva@gmail.com)

2.9: Drawing Cards


With a standard pack of cards, when drawings cards without replacement, the number of cards/possibilities is:
➢ 52 for the first card
➢ 51 for the second card
➢ 50 for the third card
➢ 52 − (𝑛 − 1) for the 𝑛𝑡ℎ card

Example 2.10
A. You draw seven cards without replacement from a standard pack of cards. When you draw the fifth card,
what is the number of possible cards that you can draw from?
B. You draw nine cards without replacement from a standard pack of cards. When you draw the fourth
card, what is the number of possible cards that you can draw from?

52
⏟ − ⏟
4 = 48
⏟ 𝐶𝑎𝑟𝑑𝑠
𝑂𝑟𝑖𝑔𝑖𝑛𝑎𝑙 𝐶𝑎𝑟𝑑𝑠 𝑅𝑒𝑚𝑎𝑖𝑛𝑖𝑛𝑔
𝑁𝑢𝑚𝑏𝑒𝑟 𝐷𝑟𝑎𝑤𝑛
52
⏟ − ⏟
4 = 48
⏟ 𝐶𝑎𝑟𝑑𝑠
𝑂𝑟𝑖𝑔𝑖𝑛𝑎𝑙 𝐶𝑎𝑟𝑑𝑠 𝑅𝑒𝑚𝑎𝑖𝑛𝑖𝑛𝑔
𝑁𝑢𝑚𝑏𝑒𝑟 𝐷𝑟𝑎𝑤𝑛

Example 2.11
𝐴𝑛𝑠𝑤𝑒𝑟 𝑎𝑠 𝑎𝑛 𝑒𝑥𝑝𝑟𝑒𝑠𝑠𝑖𝑜𝑛. 𝑆𝑖𝑚𝑝𝑙𝑖𝑓𝑖𝑐𝑎𝑡𝑖𝑜𝑛 𝑖𝑠 𝑛𝑜𝑡 𝑒𝑠𝑠𝑒𝑛𝑡𝑖𝑎𝑙
You draw two cards without replacement from a standard pack of cards. What is the probability that:
A. the first card is an Ace?
B. The first card is not an Ace?
C. both cards are Aces?
D. First card is an Ace, and second card is not an Ace
E. First card is not an Ace, and second card is an Ace
F. Exactly one card is an Ace

Parts A and B
4 1
𝐹𝑖𝑟𝑠𝑡 𝐶𝑎𝑟𝑑 𝑖𝑠 𝐴𝑐𝑒 = =
52 13
48 12
𝐹𝑖𝑟𝑠𝑡 𝐶𝑎𝑟𝑑 𝑖𝑠 𝑁𝑜𝑡 𝑎𝑛 𝐴𝑐𝑒 = =
52 13
Part C
Probability that both cards are Aces is given by:
4 3 1 1 1
𝑃(𝐴𝐴) = × = × =
52 51 13 17 221
Part D
4 48 1 16 16
𝑃(𝐴𝐴′ ) = × = × =
52 51 13 17 221
Part E
48 4 16
𝑃(𝐴′𝐴) = × =
52 51 221
Part F
16 16 32
𝑃(𝐴𝐴′ ) + 𝑃(𝐴′ 𝐴) = + =
221 221 221

Example 2.12
You draw two cards without replacement from a standard pack of cards. What is the probability that both cards

P a g e 98 | 121
Aziz Manva (azizmanva@gmail.com)

are Spades?

For both cards to be spades, we must have:


➢ First Card is a Spade
➢ Second Card is a Spade

For both cards to be Spades, we must have both events happen together:
𝑃(𝐵𝑜𝑡ℎ 𝑎𝑟𝑒 𝑆𝑝𝑎𝑑𝑒𝑠) = 𝑃(𝐹𝑖𝑟𝑠𝑡 𝑖𝑠 𝑎 𝑆𝑝𝑎𝑑𝑒) × 𝑃(𝑆𝑒𝑐𝑜𝑛𝑑 𝑖𝑠 𝑎 𝑆𝑝𝑎𝑑𝑒)

Out of the 52 cards, 13 are Spades, so the probability of getting two Spades
is given by:
13 12 1 4 1
= × = × =
52 51 4 17 17

Example 2.13
You draw two cards without replacement from a standard pack of cards. What is the probability that both cards
are Red?

For both cards to be Red, we must have:


➢ First Card is Red
➢ Second Card is Red

For both cards to be Red, we must have both events happen together:
𝑃(𝐵𝑜𝑡ℎ 𝑎𝑟𝑒 𝑅𝑒𝑑) = 𝑃(𝐹𝑖𝑟𝑠𝑡 𝑖𝑠 𝑅𝑒𝑑) × 𝑃(𝑆𝑒𝑐𝑜𝑛𝑑 𝑖𝑠 𝑅𝑒𝑑)

Out of the 52 cards, 26 are Red, so the probability of getting two Red is given
by:
26 25 1 25 25
= × = × =
52 51 2 51 102

Example 2.14
Answer the questions if the cards are drawn:
1. With replacement
2. Without replacement

You draw two cards from a standard pack of cards. What is the probability that:
A. Both cards are red
B. Both cards are diamonds
C. Both cards are face cards
D. Both cards are Kings

With Replacement Without Replacement


Both Cards Red 1 1 1 1 25 25
× = × =
2 2 4 ⏟
2 ⏟
51 102
𝐹𝑖𝑟𝑠𝑡 𝐶𝑎𝑟𝑑 𝑆𝑒𝑐𝑜𝑛𝑑 𝐶𝑎𝑟𝑑
𝑖𝑠 𝑅𝑒𝑑 𝑖𝑠 𝑅𝑒𝑑
Both Cards Diamonds 1 1 1 1 12 1
× = × =
4 4 16 ⏟
4 ⏟
51 17
𝐹𝑖𝑟𝑠𝑡 𝐶𝑎𝑟𝑑 𝑆𝑒𝑐𝑜𝑛𝑑 𝐶𝑎𝑟𝑑
𝑖𝑠 𝐷𝑖𝑎𝑚𝑜𝑛𝑑 𝑖𝑠 𝐷𝑖𝑎𝑚𝑜𝑛𝑑

P a g e 99 | 121
Aziz Manva (azizmanva@gmail.com)

Both Cards Face Cards 12 12 3 3 9 3 11 11


× = × = × =
52 52 13 13 169 ⏟
13 ⏟
51 221
𝐹𝑖𝑟𝑠𝑡 𝐶𝑎𝑟𝑑 𝑆𝑒𝑐𝑜𝑛𝑑 𝐶𝑎𝑟𝑑
𝑖𝑠 𝑎 𝐹𝑎𝑐𝑒 𝐶𝑎𝑟𝑑 𝑖𝑠 𝑎 𝐹𝑎𝑐𝑒 𝐶𝑎𝑟𝑑𝑑
Both Cards Kings 1 1 1 1 3 1
× = × =
13 13 169 ⏟
13 ⏟
51 221
𝐹𝑖𝑟𝑠𝑡 𝐶𝑎𝑟𝑑 𝑆𝑒𝑐𝑜𝑛𝑑 𝐶𝑎𝑟𝑑
𝑖𝑠 𝑎 𝐾𝑖𝑛𝑔 𝑖𝑠 𝑎 𝐹𝑎𝑐𝑒 𝐶𝑎𝑟𝑑𝑑

Example 2.15
Let a die be rolled till 2 is obtained. The probability that 2 is obtained on an even numbered toss is: (JEE Main,
2024)

Let 𝑃(𝑊𝑖𝑛 𝑜𝑛 𝑜𝑑𝑑) = 𝑝, 𝑃(𝑊𝑖𝑛 𝑜𝑛 𝑒𝑣𝑒𝑛) = 𝑞:


𝑝+𝑞 =1

5
Since the probability of winning on an even numbered toss is the same as that for odd, but reduced by 6, since
the first toss is odd numbered.
5
Substitute 𝑞 = 6 𝑝:
5 6 5
𝑝+ 𝑝=1⇒𝑝= ⇒𝑞 =1−𝑝 =
6 11 11

Example 2.16
Winning is sum of 6 on roll of two fair six-sided dice
Player A first
Player B second.
What is the probability that Player A wins?

31 67 36
𝑝+ 𝑝=1⇒ 𝑝=1⇒𝑝=
36 36 67

E. Logic

Example 2.17
Angie has a jar that contains 2 red marbles, 2 blue marbles, and no other marbles. She randomly draws 2
marbles from the jar. If the marbles are the same color, she discards one and puts the other back into the jar. If
the marbles are different colours, she discards the red marble and puts the blue marble back into the jar. She
repeats this process a total of three times. What is the probability that the remaining marble is red? (Gauss
Grade 7 2016/23)

Case I: At Least one Marble is Red


RR / RB
(RBB)

RB (BB) BB (RB)

Case II: Both Marbles are Blue RB (B)

P a g e 100 | 121
Aziz Manva (azizmanva@gmail.com)

F. Selections

Example 2.18
Of the final five contestants in a television show, three are female and two are male. If two are chosen randomly
to be the final contestants, what is the probability that both are female? Express your answer as a common
fraction. (MathCounts 2004 State Sprint)

The probability that the first contestant is female is directly given by:
3
𝑃( 𝑀
⏟ )=
1𝑠𝑡 𝐶𝑜𝑛𝑡𝑒𝑠𝑡𝑎𝑛𝑡
5
Once the first contestant is selected, we are interested in the conditional probability that the second contestant
will be female, given that the first contestant is female, which is given by:
2 1
𝑃 ( 𝐹𝑒𝑚𝑎𝑙𝑒
⏟ | 𝑀
⏟ )= =
2𝑛𝑑 𝐶𝑜𝑛𝑡𝑒𝑠𝑡𝑎𝑛𝑡 1𝑠𝑡 𝐶𝑜𝑛𝑡𝑒𝑠𝑡𝑎𝑛𝑡
4 2
And the final probability is given by:
3 1 3
𝑃( 𝑀
⏟ ) × 𝑃 ( 𝐹𝑒𝑚𝑎𝑙𝑒
⏟ | 𝑀⏟ )= × =
1𝑠𝑡 𝐶𝑜𝑛𝑡𝑒𝑠𝑡𝑎𝑛𝑡 2𝑛𝑑 𝐶𝑜𝑛𝑡𝑒𝑠𝑡𝑎𝑛𝑡 1𝑠𝑡 𝐶𝑜𝑛𝑡𝑒𝑠𝑡𝑎𝑛𝑡
5 2 10

Example 2.19
In a class of 20 students, all but 4 of the students put their names on a typed assignment. If the teacher
randomly guesses, what is the probability that she correctly guesses which paper belongs to each of the four
remaining students? Express your answer as a common fraction. (MathCounts 2009 State Countdown)

Multiplying Probabilities
1 1 1
× × × ⏟
1

4 ⏟
3 ⏟
2 𝐹𝑜𝑢𝑟𝑡ℎ 𝑆𝑡𝑢𝑑𝑒𝑛𝑡
𝐹𝑖𝑟𝑠𝑡 𝑆𝑡𝑢𝑑𝑒𝑛𝑡 𝑆𝑒𝑐𝑜𝑛𝑑 𝑆𝑡𝑢𝑑𝑒𝑛𝑡 𝑇ℎ𝑖𝑟𝑑 𝑆𝑡𝑢𝑑𝑒𝑛𝑡 𝐶𝑜𝑟𝑟𝑒𝑐𝑡
𝐶𝑜𝑟𝑟𝑒𝑐𝑡 𝐶𝑜𝑟𝑟𝑒𝑐𝑡 𝐶𝑜𝑟𝑟𝑒𝑐𝑡

Multiplication Principle
The number of ways of correctly guessing each student’s paper is only
1
But, by the multiplication principle, the options that she for guessing are:
4 × 3 × 2 × 1 = 4! = 24
And the probability is:
𝑆𝑢𝑐𝑒𝑠𝑠𝑓𝑢𝑙 𝑂𝑢𝑡𝑐𝑜𝑚𝑒𝑠 1 1
= =
𝑇𝑜𝑡𝑎𝑙 𝑂𝑢𝑡𝑐𝑜𝑚𝑒𝑠 4! 24

G. Drawing without replacement

Example 2.20
A jar contains two red marbles, three green marbles, ten white marbles and no other marbles. Two marbles are
randomly drawn from this jar without replacement. What is the probability that these two marbles drawn will
both be red? Express your answer as a common fraction. (MathCounts 2008 Chapter Sprint)

Multiplying Probabilities

P a g e 101 | 121
Aziz Manva (azizmanva@gmail.com)

2 1 1
× =
15 14 105

Combinations
2
( ) 1 1
2 = =
15 15 × 14 105
( )
2 2

Example 2.21
Seven cards each containing one of the following letters C, B, T, A, E, M and H are placed in a hat. Each letter is
used only once. Stu will pull four cards out at random and without replacement. What is the probability that Stu
pulls out M, A, T, H in this order? Express your answer as a common fraction. (MathCounts 2009 Warm-up 3)

Multiplying Probabilities

1 1 1 1 1
× × × =

7 ⏟
6 ⏟5 ⏟
4 840
𝑃(𝑀) 𝑃(𝐴 | 𝑀) 𝑃(𝑇 | 𝑀,𝐴) 𝑃(𝐻 | 𝑀,𝐴,𝑇)

Multiplication Principle
Successful Outcomes
= 1 𝑊𝑎𝑦
Total Outcomes
= 7 × 6 × 5 × 4 = 840

Probability
1
=
840

Example 2.22
A bag contains two red beads and two green beads. You reach into the bag and pull out a bead, replacing it with
a red bead regardless of the color you pulled out. What is the probability that all beads in the bag are red after
three such replacements? (AMC 10B 2003/21)

You make three replacements. For all beads in the bag to


be red, the green beads must both be pulled out, and
replaced with a red bead.
Red (1/2) Green

This can happen in the following ways:


𝑅𝐺𝐺
⏟ , 𝐺𝑅𝐺 ⏟ , 𝐺𝐺𝑅
⏟ Green Red Green
𝑪𝒂𝒔𝒆 𝑰 𝑪𝒂𝒔𝒆 𝑰𝑰 𝑪𝒂𝒔𝒆 𝑰𝑰𝑰

Case I: Green Green Red


The first bead is red and the other two beads are green:
1 1 1 1
× × =
2 2 4 16
Case II:
The middle bead is red and the other two beads are green.

P a g e 102 | 121
Aziz Manva (azizmanva@gmail.com)

1 3 1 3
× × =
2 4 4 32
Case III:
The last bead is red and the other two beads are green.
1 1 1
× ×1=
2 4 8

1 3 1 9
+ + =
16 32 8 32
H. Multi-Tier Selections

2.23: Multi-Tier Selections


Multi-Tier selections require you to do action A, and then pick one of two further things to do based on the
result of action A.

➢ For example, you have a coin, a black vase, and a white vase. You toss the coin. If the coin is heads, you
draw a ball from a black vase. If the coin is tails, you draw a ball from a white vase.
➢ You roll a standard six-sided die. If the outcome is prime, you spin a spinner. If the outcome is
composite, you draw a card from a standard pack of card. If the outcome is neither of the two, you roll
the die again.

Example 2.24
A bag 𝐴 contains 4 black and 6 red balls and bag 𝐵 contains 7 black and 3 red balls. A die is thrown. If 1 or 2
appears on the die, then bag 𝐴 is chosen, otherwise bag 𝐵. If two balls are drawn at random (without
replacement) from the selected bag, find the probability of one of them being red, and another black. (CBSE
2015)

The probabilities for the die are:


2 1
𝑃(𝐴) = 𝑃(1 𝑜𝑟 2) = =
6 3
4 2
𝑃(𝐵) = 𝑃(3,4,5,6) = =
6 3

If bag A is selected, the probability of getting one red and one black ball from the bag is:
6 4 4 6 6×4 8
𝑃(𝑅𝑒𝑑 𝑜𝑟 𝐵𝑙𝑎𝑐𝑘|𝐴) = 𝑃(𝑅𝐵) + 𝑃(𝐵𝑅) = ( ) ( ) + ( ) ( ) = =
10 9 10 9 5 × 9 15

If bag B is selected, the probability of getting one red and one black ball from the bag is:
3 7 7 3 3×7 7
𝑃(𝑅𝑒𝑑 𝑜𝑟 𝐵𝑙𝑎𝑐𝑘|𝐵) = 𝑃(𝑅𝐵) + 𝑃(𝐵𝑅) = ( ) ( ) + ( ) ( ) = =
10 9 10 9 5 × 9 15

The answer that we want is:


𝑃(𝐴)𝑃(𝑅𝑒𝑑 𝑜𝑟 𝐵𝑙𝑎𝑐𝑘|𝐴) + 𝑃(𝐵)𝑃(𝑅𝑒𝑑 𝑜𝑟 𝐵𝑙𝑎𝑐𝑘|𝐵)
1 8 2 7 8 + 14 22
= ( )( ) + ( )( ) = =
3 15 3 15 45 45

Example 2.25
Three machines 𝐸1 , 𝐸2 and 𝐸3 in a certain factory producing electric bulbs produce 50%, 25% and 25% of the
total daily output of electric bulbs. It is known that 4% of the bulbs produced by each of machines 𝐸1 and𝐸2 are
defective, and that 5% of those produced by machine 𝐸3 are defective. If one bulb is picked at random from a

P a g e 103 | 121
Aziz Manva (azizmanva@gmail.com)

day’s production, calculate the probability that it is defective. (CBSE 2015)

1
1 44 17
4% 𝑜𝑓 50% + 4% 𝑜𝑓 25% + 5% 𝑜𝑓 25% = 2% + 1% + 1.25% = 4 % = =
4 100 400

𝑃(𝐸1 )𝑃(𝐷|𝐸1 ) + 𝑃(𝐸2 )𝑃(𝐷|𝐸2 ) + 𝑃(𝐸3 )𝑃(𝐷|𝐸3 )


1 1 1 1 1 1 1 1 1 8+4+5 17
= ( )( )+ ( )( )+ ( )( ) = + + = =
2 25 4 25 4 20 50 100 80 400 400

Example 2.26
You draw three cards from a standard deck of 52 cards, one at a time, and without replacement. What is the
probability that the first is an ace, the second is a red card, and the third is a red face card?

Example 2.27
Shadowheart is playing Dungeons and Dragons. She rolls a eighteen sided die and gets a positive even number.
Astarion is playing Descent into Avernus. He rolls a twenty-eight-sided die and rolls a multiple of three. What is
the probability that they both roll the same number?

𝑆ℎ𝑎𝑑𝑜𝑤ℎ𝑒𝑎𝑟𝑡 = {2,4,6, . . .18} = 9 𝑁𝑢𝑚𝑏𝑒𝑟𝑠


𝐴𝑠𝑡𝑎𝑟𝑖𝑜𝑛 = {3,6,9, . . .27} = 9 𝑁𝑢𝑚𝑏𝑒𝑟𝑠

P a g e 104 | 121
Aziz Manva (azizmanva@gmail.com)

𝐶𝑜𝑚𝑚𝑜𝑛 𝑁𝑢𝑚𝑏𝑒𝑟𝑠 = {6,12,18}


Direct Method
Both Shadowheart and Astarion must get the same numbers. Hence, by the Addition Rule, we want to find:
1 1 1 1 1 1 1 1
𝑃(6) + 𝑃(12) + 𝑃(18) = ( × ) + ( × ) + ( × ) = 3 ( ) =
9 9 9 9 9 9 81 27
Method II
Start with Shadowheart. For us to count the outcome as successful, Shadowheart must get any of three
numbers:
3 1
𝑃(6 𝑜𝑟 12 𝑜𝑟 18) = =
9 3

Once we pick a particular value for Shadowheart, that value is locked, and hence the only way that Astarion will
get the same number as Shadowheart is if he gets the exact number, which has probability
1
9

The combination of the two probabilities is:


1 1 1
× =
3 9 27

Example 2.28: Two Containers


Container I holds 8 red balls and 4 green balls; containers II and III each hold 2 red balls and 4 green balls. A
container is selected at random and then a ball is randomly selected from that container. What is the probability
that the ball selected is green? Express your answer as a common fraction. (MathCounts 2003 State Sprint)

1 4 2 4 1 1 2 2 5
× + × = × + × =

3 ⏟
12 ⏟
3 ⏟
6 3 3 3 3 9

𝑷(𝑰) 𝑷(𝑮𝒓𝒆𝒆𝒏 | 𝑰) ⏟
𝑷(𝑰𝑰 𝒐𝒓 𝑰𝑰𝑰) 𝑷(𝑮𝒓𝒆𝒆𝒏 | 𝑰𝑰 𝒐𝒓 𝑰𝑰𝑰)
𝑪𝒐𝒏𝒕𝒂𝒊𝒏𝒆𝒓 𝑰 𝑪𝒐𝒏𝒕𝒂𝒊𝒏𝒆𝒓 𝑰𝑰 𝒂𝒏𝒅 𝑰𝑰𝑰

Example 2.29
Arjun has a bag that contains 5 balls and Becca has a bag that contains 3 balls. Arjun’s bag contains 2 red balls, 1
green, 1 yellow, and 1 violet ball. Becca’s bag contains 2 black balls and 1 orange ball. Arjun randomly chooses 1
ball from his bag and puts it into Becca’s bag. Becca then randomly chooses 1 ball from her bag and puts it into
Arjun’s bag. Again, Arjun randomly chooses 1 ball from his bag and puts it into Becca’s bag. After these
exchanges, there are 4 balls in each bag. What is the probability that each bag contains exactly 3 different
colours of balls? (Gauss 7/2022/25)

Red Green Yellow Violet Black Orange Total


Arjun 2 1 1 1 5
Becca 2 1 3

Since we want three different colors, in each bag, we must have:


➢ Two balls of one color
➢ Two balls of two different colors

Currently we have two red in Arjun's bag, and two black in Becca's bag. This arrangement cannot be
interchanged (since Becca only transfers one ball).
Hence, the final arrangement must be the same as the current one (as far as the red and the black are
concerned).

P a g e 105 | 121
Aziz Manva (azizmanva@gmail.com)

We calculate the probability of the above using casework.

Case I: Arjun selects a red for the first ball


The selected red must come back from Becca.
2
Step I: 2 Red out of 5 Balls: 5
1
Step II: 1 Red out of 4 Balls: 4
3
Step III: 3 non-red out of 5 balls: 5
2 1 3 3
𝑃𝑟𝑜𝑏𝑎𝑏𝑖𝑙𝑖𝑡𝑦 = ∙ ∙ =
5 4 5 50

Case IIA: Arjun selects a non-red for the first ball, and Becca selects a black for her ball
3
Step I: 3 non-red out of 5 Balls:
5
1
Step II: 2 Black out of 4 Balls: 2
1
Step III: 1 Black out of 5 balls: 5
3 1 1 3
𝑃𝑟𝑜𝑏𝑎𝑏𝑖𝑙𝑖𝑡𝑦 = ∙ ∙ =
5 2 5 50

Case IIB: Arjun selects a non-red for the first ball, and Becca selects a non-black for her ball
3
Step I: 3 non-red out of 5 Balls: 5
1
Step II: 2 non-Black out of 4 Balls:
2
3
Step III: 3 non-red out of 5 balls: 5
3 1 3 9
𝑃𝑟𝑜𝑏𝑎𝑏𝑖𝑙𝑖𝑡𝑦 = ∙ ∙ =
5 2 5 50

Final answer
3 3 9 15 3
𝑃𝑟𝑜𝑏𝑎𝑏𝑖𝑙𝑖𝑡𝑦 = + + = =
50 50 50 50 10
I. Birthday Problem

Example 2.30
For this question assume that no birthdays fall on 29𝑡ℎ Feb. What is the probability that
A. no two people in a group of 5 people have the same birthday?
B. out of 32 individuals, at least two have common birthdays?
C. At least two people in a group of 𝑛 people have the same birthday?

Part A
365 364 363 362 361 365!
𝑃(𝐷𝑖𝑓𝑓) = ( )( )( )( )( )=
365 365 365 365 365 360! 3655
365!
𝑃(𝑆𝑎𝑚𝑒) = 1 − 𝑃(𝐷𝑖𝑓𝑓) = 1 −
360! 3655
Part B

365 364 334 365!


𝑃(𝐷𝑖𝑠𝑡𝑖𝑛𝑐𝑡) = × × …× =
365 365 365 333! 36532
365!
𝑃(𝐴𝑡 𝑙𝑒𝑎𝑠𝑡 𝑜𝑛𝑒 𝑐𝑜𝑚𝑚𝑜𝑛) = 1 − 𝑃(𝐷𝑖𝑠𝑡𝑖𝑛𝑐𝑡) = 1 −
333! 36532

P a g e 106 | 121
Aziz Manva (azizmanva@gmail.com)

Part C
365 364 363 365 − (𝑛 − 1) 365!
𝑃(𝐷𝑖𝑓𝑓) = ( )( )( ) × …× ( )=
365 365 365 365 (365 − 𝑛)! 365𝑛
365!
𝑃(𝑆𝑎𝑚𝑒) = 1 − 𝑃(𝐷𝑖𝑓𝑓) = 1 −
(365 − 𝑛)! 365𝑛

Example 2.31
What is the number of people needed in a group so that the probability of at least two people having the same
1
birthday is greater than 2 ?

365!
< 0.5
(365 − 𝑛)! 365𝑛

Example 2.32
Bag A contains 1 white, 2 blue and 3 red balls. Bag B contains 3 white, 3 blue and 2 red balls. Bag C contains 2
white, 3 blue and 4 red balls. One bag is selected at random, and then two balls are drawn from the selected bag.
Find the probability that the balls drawn are white and red. (ISC 2017)

2.2 Bayes Theorem


A. Conditional Probability: Visual Interpretation
The key in conditional probability is that you apply some condition.

2.33: Conditional Probability


𝑃(𝐴 ∩ 𝐵)
𝑃(𝐴|𝐵) =
𝑃(𝐵)

Begin by considering the general universal set which has two events, event 𝐴 and event 𝐵.

➢ 𝑃(𝐴|𝐵) is the conditional probability of event 𝐴 given that event 𝐵 has already happened.
➢ Since event 𝐵 has already happened, we do not need to consider anything outside of event 𝐵. Hence,
event B is colored blue, and everything outside of event 𝐵 is colored white, and hence not applicable in
this context.

P a g e 107 | 121
Aziz Manva (azizmanva@gmail.com)

➢ We want the probability of event 𝐴, given that event B has happened, which is the green region out of
the blue region.

𝑆𝑢𝑐𝑐𝑒𝑠𝑠𝑓𝑢𝑙 𝑂𝑢𝑡𝑐𝑜𝑚𝑒𝑠 𝐺𝑟𝑒𝑒𝑛 𝑅𝑒𝑔𝑖𝑜𝑛 𝑃(𝐴 ∩ 𝐵)


𝑃(𝐴|𝐵) = = =
𝑇𝑜𝑡𝑎𝑙 𝑂𝑢𝑡𝑐𝑜𝑚𝑒𝑠 𝐵𝑙𝑢𝑒 𝑅𝑒𝑔𝑖𝑜𝑛 + 𝐺𝑟𝑒𝑒𝑛 𝑅𝑒𝑔𝑖𝑜𝑛 𝑃(𝐵)

Example 2.34
Passing a subject requires minimum 35 marks out of 100. What is the probability that you have got 34 marks
given that you have passed Maths.

Define a few sets:


𝑈 is the universal set of all outcomes for students who appeared for
the exam
𝑃 is the set of students who passed
𝐹 is the set of students who failed
The set of students outside both 𝑃 and 𝐹 represents the set of
students whose result has not been declared yet.

Now we can conclude:


𝑃(𝐹 ∩ 𝑃) = 0 𝑏𝑒𝑐𝑎𝑢𝑠𝑒 𝐹 𝑎𝑛𝑑 𝑃 𝑎𝑟𝑒 𝑚𝑢𝑡𝑢𝑎𝑙𝑙𝑦 𝑒𝑥𝑐𝑙𝑢𝑠𝑖𝑣𝑒
𝑃(34|𝑃) = 0 𝑏𝑒𝑐𝑎𝑢𝑠𝑒 𝑝𝑎𝑠𝑠𝑖𝑛𝑔 𝑟𝑒𝑞𝑢𝑖𝑟𝑒𝑠 𝑚𝑖𝑛𝑖𝑚𝑢𝑚 35

Example 2.35
You roll a standard six-sided die. What is the probability that the outcome is odd given that the outcome is
prime.

𝑂 is the set of odd outcomes.


𝑃 is the set of prime outcomes.
U is the set of all outcomes (universal set).

The required probability is then:


𝐺𝑟𝑒𝑒𝑛 𝑅𝑒𝑔𝑖𝑜𝑛 𝑃(𝐴 ∩ 𝐵) 2
𝑃(𝐴|𝐵) = = =
𝐵𝑙𝑢𝑒 𝑅𝑒𝑔𝑖𝑜𝑛 + 𝐺𝑟𝑒𝑒𝑛 𝑅𝑒𝑔𝑖𝑜𝑛 𝑃(𝐵) 3

Example 2.36
Ten people (five boys, five girls) are in the selection process for a
chess tournament. One of the boys is called Ralph. One candidate will be selected at random. What is the
probability that:

P a g e 108 | 121
Aziz Manva (azizmanva@gmail.com)

A. Ralph gets selected. In other words, find 𝑃(𝑅𝑎𝑙𝑝ℎ).


B. a boy gets selected? A girl getting selected
C. Ralph getting selected given that a boy got selected?
D. Ralph getting selected given that a girl got selected?
E. Ralph got selected as the product of the probabilities of parts B and C.

Parts A and B
The sample space is:
{𝑏𝑅𝑎𝑙𝑝ℎ , 𝑏2 , … , 𝑏5 , 𝑔1 , 𝑔2 , … 𝑔5 }
𝑆𝑢𝑐𝑐𝑒𝑠𝑠𝑓𝑢𝑙 𝑂𝑢𝑡𝑐𝑜𝑚𝑒𝑠 1
𝑃(𝑅𝑎𝑙𝑝ℎ) = =
𝑇𝑜𝑡𝑎𝑙 𝑂𝑢𝑡𝑐𝑜𝑚𝑒𝑠 10
𝑆𝑢𝑐𝑐𝑒𝑠𝑠𝑓𝑢𝑙 𝑂𝑢𝑡𝑐𝑜𝑚𝑒𝑠 5 1
𝑃(𝐵𝑜𝑦) = = =
𝑇𝑜𝑡𝑎𝑙 𝑂𝑢𝑡𝑐𝑜𝑚𝑒𝑠 10 2
𝑆𝑢𝑐𝑐𝑒𝑠𝑠𝑓𝑢𝑙 𝑂𝑢𝑡𝑐𝑜𝑚𝑒𝑠 5 1
𝑃(𝐺𝑖𝑟𝑙) = = =
𝑇𝑜𝑡𝑎𝑙 𝑂𝑢𝑡𝑐𝑜𝑚𝑒𝑠 10 2

Part C

The original sample space is:


{𝑏𝑅𝑎𝑙𝑝ℎ , 𝑏2 , … , 𝑏5 , 𝒈𝟏 , 𝒈𝟐 , … 𝒈𝟓 }
But since we know that a boy has been selected the girls must be
removed from the sample space, and it now becomes:
{𝑏𝑅𝑎𝑙𝑝ℎ , 𝑏2 , … , 𝑏5 }

𝑆𝑢𝑐𝑐𝑒𝑠𝑠𝑓𝑢𝑙 𝑂𝑢𝑡𝑐𝑜𝑚𝑒𝑠 1
𝑃(𝑅𝑎𝑙𝑝ℎ|𝐵𝑜𝑦) = =
𝑇𝑜𝑡𝑎𝑙 𝑂𝑢𝑡𝑐𝑜𝑚𝑒𝑠 5
Part D

{𝑔1 , 𝑔2 , … 𝑔5 }

𝑆𝑢𝑐𝑐𝑒𝑠𝑠𝑓𝑢𝑙 𝑂𝑢𝑡𝑐𝑜𝑚𝑒𝑠 0
𝑃(𝑅𝑎𝑙𝑝ℎ|𝐺𝑖𝑟𝑙) = =
𝑇𝑜𝑡𝑎𝑙 𝑂𝑢𝑡𝑐𝑜𝑚𝑒𝑠 5

B. Bayes Theorem: Visual Interpretation

2.37: Bayes Theorem

𝑃(𝐴1 and 𝐵) 𝑃(𝐴1 )𝑃(𝐵|𝐴1 )


𝑃(𝐴1 |𝐵) = =
𝑃(𝐵) 𝑃(𝐴1 )𝑃(𝐵|𝐴1 + 𝑃(𝐴2 )𝑃(𝐵|𝐴2 ) + 𝑃(𝐴3 )𝑃(𝐵|𝐴3 )
)
If possible, use a tree diagram instead of Bayes Theorem.

P a g e 109 | 121
Aziz Manva (azizmanva@gmail.com)

Example 2.38
Ten people (five boys, five girls) are in the selection process for a chess tournament. One of the boys is called
Ralph. One candidate will be selected at random.
A. Identify the different cases in terms of gender for the selection.

Part A
The two cases are:
➢ Case I: A boy is selected
➢ Case II: A girl is selected

There are
1 1 1 1 1
𝑃(𝑅𝑎𝑙𝑝ℎ) = 𝑃(𝑅𝑎𝑙𝑝ℎ|𝐵𝑜𝑦)𝑃(𝐵𝑜𝑦) + 𝑃(𝑅𝑎𝑙𝑝ℎ|𝐺𝑖𝑟𝑙)𝑃(𝐺𝑖𝑟𝑙) = × +0× = +0=
5 2 2 10 10

C. Tree Diagrams

Example 2.39
A bag contains 3 red and 7 black balls. Two balls are selected at random one by one without replacement. If the
second selected ball happens to be red, what is the probability that the first selected ball is also red? (CBSE
2014)

Successful Outcome
The successful outcome that we are interested in is
➢ First ball being red
➢ & Second ball also being
Which can be written as
3 2
𝑃(𝑅𝑅) = ×
10 9
Total Outcomes
There are two ways in which you can get the second ball to
be red.
Case I: The first ball is red
3 2
= 𝑃(𝑅𝑅) = × (𝐶𝑎𝑙𝑐𝑢𝑙𝑎𝑡𝑒𝑑 𝑎𝑏𝑜𝑣𝑒)
10 9
Case II: The first ball is black
7 3
= 𝑃(𝐵𝑅) = ×
10 9

P a g e 110 | 121
Aziz Manva (azizmanva@gmail.com)

We want:
3 2 2
𝑃(𝑆𝑢𝑐𝑐𝑒𝑠𝑠𝑓𝑢𝑙 𝑂𝑢𝑡𝑐𝑜𝑚𝑒𝑠) 𝑃(𝑅𝑅) 10 ×9 30 2
= = = = =
𝑃(𝑇𝑜𝑡𝑎𝑙 𝑂𝑢𝑡𝑐𝑜𝑚𝑒𝑠) 𝑃(𝑅𝑅) + 𝑃(𝐵𝑅) 3 2 7 3 2 7 9
10 × 9 + 10 × 9 30 + 30

Example 2.40
A bag which contains 4 balls. Two balls are drawn at random (without replacement), and are found to be white.
What is the probability that all the balls in the bag are white? (CBSE 2014, 2015)

Case I: Bag has 4 white balls


1 1 6
×1×1= =
3 3 18
Case II: Bag has 3 white balls
1 3 2 1 3
× × = =
3 4 3 6 18
Case III: Bag has 2 white balls
1 2 1 1
× × =
3 4 3 18

1 6
3 18 6 3
= = = =
6 3 1 10 10 5
18 + 18 + 18 18

Example 2.41
𝐴𝑛𝑠𝑤𝑒𝑟 𝑒𝑎𝑐ℎ 𝑝𝑎𝑟𝑡 𝑖𝑛𝑑𝑒𝑝𝑒𝑛𝑑𝑒𝑛𝑡𝑙𝑦
Ronald always put 4 balls in a vase. There is 20% probability that he will put 4 white balls, 30% probability that
he will put 3 white balls, 15% probability that he will put 2 white balls, and 17% probability that he will put 1
white ball.
A. While putting balls into the vase, what is the probability that Ronald will put zero white balls in the
vase?
B. Two balls are drawn at random (without replacement), and are found to be white. What is the
probability that all the balls in the bag are white?

P a g e 111 | 121
Aziz Manva (azizmanva@gmail.com)

Part A
= 100% − (20% + 30% + 15% + 17%) = 100% − 82% = 18%
Part B
Case I: Bag has 4 white balls

𝑃(𝑊𝑊|𝐶𝑎𝑠𝑒 𝐼) = 1

Case II: Bag has 3 white balls


3 3 2 3 1 3
𝑃(𝑊𝑊 & 𝐶𝑎𝑠𝑒 𝐼𝐼) = × × = × =
10 4 3 10 2 20
Case III: Bag has 2 white balls
3 2 1 3 1
𝑃(𝑊𝑊 & 𝐶𝑎𝑠𝑒 𝐼𝐼𝐼) = × × = ×
20 4 3 20 6
Substitute
1 1
×1 1 1 8
= 5 = 5 = = =
1 3 1 3 1 1 3 1 3 1 3 1 15 15
( × 1) + (10 × 2) + (20 × 6) + (2 × 2 ) + (4 × 6)] 1 + 4 + 8
5 5 [1 8

Example 2.42
A bag 𝑋 contains 4 white balls and 2 black balls, while bag 𝑌 contains 3 white and 3 black balls. Two balls are
drawn (without replacement) at random from one of the bags, and were found to be one white and one black.
Find the probability that the balls were drawn from bag 𝑌. (CBSE 2016)

P a g e 112 | 121
Aziz Manva (azizmanva@gmail.com)

The probability of picking bag 𝑋 and then getting one white


and one black ball is:
𝑃(𝑋)[𝑃(𝑊𝐵) + 𝑃(𝐵𝑊)]
1 4 2 2 4 1 4 2 4
= [ × + × ] = ×2( × ) =
2 6 5 6 5 2 6 5 15

The probability of picking bag 𝑌 and then getting one white


and one black ball is:
𝑃(𝑌)[𝑃(𝑊𝐵) + 𝑃(𝐵𝑊)]
1 3 3 3 3 1 3 3 3
= [ × + × ] = ×2( × ) =
2 6 5 6 5 2 6 5 10

3 3 3
𝑆𝑢𝑐𝑐𝑒𝑠𝑠𝑓𝑢𝑙 𝑂𝑢𝑡𝑐𝑜𝑚𝑒𝑠 10 10 10 3 30 9
= = = = = × =
𝑇𝑜𝑡𝑎𝑙 𝑂𝑢𝑡𝑐𝑜𝑚𝑒𝑠 4 3 8+9 17 10 17 17
+
15 10 30 30

Example 2.43
A fair die is rolled. If face 1 turns up, a ball is drawn from bag 𝐴. If face 2 or 3 turns up, a ball is drawn from bag
𝐵. If face 4 or 5 or 6 turns up, a ball is drawn from bag 𝐶. Bag A contains 3 red and 2 white balls, bag 𝐵 contains
3 red and 4 white balls, and bag 𝐶 contains 4 red and 5 white balls. The die is rolled, a bag is picked up, and a
ball is drawn. If the drawn ball is red, what is the probability that it is drawn from bag 𝐵? (ISC 2017)

Probability of getting red ball from bag A


1 3
= ×
6 5
Probability of getting red ball from bag B
1 3
= ×
3 7
Probability of getting red ball from bag C
1 4
× =
2 9
1 3 1
3×7 = 7 1
= ×
630
=
90
1 3 1 3 1 4 1 1 2 7 63 + 90 + 140 293
6 × 5 + 3 × 7 + 2 × 9 10 + 7 + 9

Example 2.44
An urn contains four fair dice. Two have faces numbered 1, 2, 3, 4, 5 and 6; one has faces numbered 2,2,4,4,6
and 6; and one has all six faces numbered six. One of the dice is randomly selected from the urn and rolled. The
same die is rolled a second time. Calculate the probability that a 6 is rolled both times. (SOA, Exam P)

Case I: The die with faces numbered 1, 2, 3, 4, 5 and 6 is selected


1 1 1 1
× × =
2 6 6 72

Case II: The die with faces numbered 2,2,4,4,6 and 6 is selected
1 1 1 1 2
× × = =
4 3 3 36 72

P a g e 113 | 121
Aziz Manva (azizmanva@gmail.com)

Case III: The die with all faces numbered 6 is selected


1 1 18
×1×1= =
4 4 72

1 2 18 21 7
+ + = =
72 72 72 72 24

Example 2.45
Seventy percent of the lost luggage at a certain airport are subsequently recovered. Of the recovered luggage,
60% are of a premium brand 𝐹𝐴𝑁𝐶𝑌, whereas 90% of the non-recovered luggage are not of the brand 𝐹𝐴𝑁𝐶𝑌.
Suppose that a luggage is lost. If it is a brand 𝐹𝐴𝑁𝐶𝑌, what is the probability that it will be recovered. (Actuarial
Maths, Finan)

Fancy Not Fancy Total


R 60% 𝑜𝑓 70% = 42% 70%
NR 3% 90% 𝑜𝑓 30% = 27% 30%
Total 45% 100%

𝑆𝑢𝑐𝑐𝑒𝑠𝑠𝑓𝑢𝑙 𝑂𝑢𝑡𝑐𝑜𝑚𝑒𝑠 42% 14


𝑃𝑟𝑜𝑏𝑎𝑏𝑖𝑙𝑖𝑡𝑦 = = =
𝑇𝑜𝑡𝑎𝑙 𝑂𝑢𝑡𝑐𝑜𝑚𝑒𝑠 45% 15

D. Contingency Table Method

Example 2.46
Upon arrival at a hospital’s emergency room, patients are categorized according to their condition as critical,
serious, or stable. In the past year:
A. 10% of the emergency room patients were critical
B. 30% of the emergency room patients were serious
C. The rest of the emergency room patients were stable
D. 40% of the critical patients died
E. 10% of the serious patients died
F. 1% of the stable patients died
Given that a patient survived, what is the probability that the patient was categorized as serious upon arrival?
(SOA, Exam P)

Table Method
Classification Died Survived Total
Critical 4% 6% 10%
Serious 3% 27% 30%
Stable 0.6% 59.4% 60%
Total 7.6% 92.4% 100%

P a g e 114 | 121
Aziz Manva (azizmanva@gmail.com)

27
≈ 29.22%
92.4
Tree Diagram
Draw a tree diagram.
The required probability is then:
(30)(90) 2700
=
(10)(60) + (30)(90) + (60)(99) 600 + 2700 + 5940

Which simplifies to:


2700
= = 45/154
9240

Example 2.47
In a group of health insurance policyholders, 20% have high blood pressure and 30% have high cholesterol. Of
the policyholders with high blood pressure, 25% have high cholesterol. A policyholder is randomly selected
from the group. Calculate the probability that a policyholder has high blood pressure, given that the
policyholder has high cholesterol. (SOA, Exam P)

The number of policyholders with high cholesterol and high blood pressure
= 25% 𝑜𝑓 𝑡ℎ𝑒 𝑝𝑜𝑙𝑖𝑐𝑦ℎ𝑜𝑙𝑑𝑒𝑟𝑠 𝑤𝑖𝑡ℎ ℎ𝑖𝑔ℎ 𝑏𝑙𝑜𝑜𝑑 𝑝𝑟𝑒𝑠𝑠𝑢𝑟𝑒
= 25% 𝑜𝑓 20%
= 5%

HC No HC Total
HBP 25% 𝑜𝑓 20% = 5% 20%
No HBP 25%
Total 30%

𝑆𝑢𝑐𝑐𝑒𝑠𝑠𝑓𝑢𝑙 𝑂𝑢𝑡𝑐𝑜𝑚𝑒𝑠 5% 5 1
𝑃𝑟𝑜𝑏𝑎𝑏𝑖𝑙𝑖𝑡𝑦 = = = =
𝑇𝑜𝑡𝑎𝑙 𝑂𝑢𝑡𝑐𝑜𝑚𝑒𝑠 30% 30 6

Example 2.486
A blood test indicates the presence of a particular disease 95% of the time when the disease is actually present.
The same test indicates the presence of the disease 0.5% of the time when the disease is not present. One
percent of the population actually has the disease. Calculate the probability that a person has the disease given
that the test indicates the presence of the disease. (SOA, Exam P)

Classification Presence No Total


Presence
Has Disease 95% 𝑜𝑓 1% 1%
Do not have the 0.5% 𝑜𝑓 99% 99%
disease
Total 100%
(95%)(1%) (1)(95)
=
(95%)(1%) + (0.5%)(99%) (1)(95) 1
+ (2) (99)

6 This is a classic question. You will find some version of this in many different textbooks.

P a g e 115 | 121
Aziz Manva (azizmanva@gmail.com)

Multiply numerator and denominator by 2:


95 × 2 190 190
= =
1
95 × 2 + ( ) (99) × 2 190 + 99 289
2
E. Relative Probabilities

2.49: Relative Probabilities


Relative probabilities are when we have the ratio of probabilities rather the numerical value of probabilities.

For instance, if the probability of event B is twice the probability of event A, then we can write:
𝑃(𝐴): 𝑃(𝐵) = 1: 2 = 𝑥: 2𝑥

Note that we can introduce a variable if we want, but the ratio remains the same even without the variable.

Example 2.50
A health study tracked a group of persons for five years. At the beginning of the study, 20% were classified as
heavy smokers, 30% as light smokers, and 50% as nonsmokers. Results of the study showed that light smokers
were twice as likely as nonsmokers to die during the five-year study, but only half as likely as heavy smokers. A
randomly selected participant from the study died over the five-year period. Calculate the probability that the
participant was a heavy smoker. (SOA, Exam P)

Let
Classification Died Survived Total
𝑃(𝐷𝑖𝑒| 𝑁𝑜𝑛𝑠𝑚𝑜𝑘𝑒𝑟) = 𝑥 Heavy 4𝑥 × 20% 20%
𝑃(𝐷𝑖𝑒| 𝐿𝑖𝑔ℎ𝑡 𝑆𝑚𝑜𝑘𝑒𝑟) = 2𝑥 Light 2𝑥 × 30% 30%
𝑃(𝐷𝑖𝑒| 𝐻𝑒𝑎𝑣𝑦 𝑆𝑚𝑜𝑘𝑒𝑟) = 4𝑥 Nonsmoker 𝑥 × 50% 50%
Total 100%

4𝑥 × 20% 80𝑥% 80𝑥% 8


= = = =
4𝑥 × 20% + 2𝑥 × 30% + 1𝑥 × 50% 80𝑥% + 60𝑥% + 50𝑥% 190𝑥% 19

2.51: Shortcut Method

𝑥: 2𝑥: 4𝑥 = 1: 2: 4

Example 2.52
A health study tracked a group of persons for five years. At the beginning of the study, 20% were classified as
heavy smokers, 30% as light smokers, and 50% as nonsmokers. Results of the study showed that light smokers
were twice as likely as nonsmokers to die during the five-year study, but only half as likely as heavy smokers. A
randomly selected participant from the study died over the five-year period. Calculate the probability that the
participant was a heavy smoker. (SOA, Exam P)

Classification Died Survived Total


Heavy 4 × 20% 20%
Light 2 × 30% 30%
Nonsmoker 1 × 50% 50%
Total 100%

P a g e 116 | 121
Aziz Manva (azizmanva@gmail.com)

4 × 20 80 80 8
= = =
4 × 20 + 2 × 30 + 1 × 50 80 + 60 + 50 190 19

Example 2.53
For A, B and C, the chances of being selected as the manager of a firm are 4: 1: 2 respectively. The probabilities
for them to introduce a radical change in the marketing strategy are 0.3, 0.8 and 0.5 respectively. If a change
takes place, then find the probability that it is due to the appointment of B. (ISC 2013)

Method I
1 8 8 8
( )( ) 8 4
7 10 = 70 = 70 = =
4 3 1 8 2 5 12 8 10 30 30 15
(7) (10) + (7) (10) + (7) (10) 70 + 70 + 70 70
Method II: Shortcut
1(8) 8 4
= =
4(3) + 1(8) + 2(5) 30 15

F. Forming Equations

(Calculator) Example 2.54


A student takes an examination consisting of 20 true-false questions. The student knows the answer to 𝑁 of the
questions, which are answered correctly and guesses the answers to the rest. The conditional probability that
the student knows the answer to a question, given that the student answered it correctly, is 0.824. Calculate 𝑁.
(SOA, Exam P)

From the tree diagram above, the condition probability, in terms of N, of the student knowing the answer to the
questions, given that he got it correct is:
𝑁 𝑁
20 20 𝑁 𝑁
= = =
𝑁 20 − 𝑁 1 1
20 + ( 20 ) (2) 𝑁 + 2 (20 − 𝑁) 𝑁 + 10 − 0.5𝑁 0.5𝑁 + 10
20

But from the question, we know the numerical value of the expression is 0.824. Hence, equate the two
quantities to get:

P a g e 117 | 121
Aziz Manva (azizmanva@gmail.com)

𝑁
= 0.824
0.5𝑁 + 10

Cross-multiply
𝑁 = 0.824(0.5𝑁) + 8.24
𝑁 − 0.412𝑁 = 8.24
0.588𝑁 = 8.24
8.24
𝑁= = 14.01 ≈ 14
0.588

G. Further Calculations

Example 2.55
A jar has 1000 coins, of which 999 are fair and 1 is double-headed. Pick a coin at random, and toss it 10 times.
Given that you see 10 heads, what is the probability that the next toss of that coin is also a head?7

Fair Coin
Probability of picking a fair coin
999
= = 0.999
1000
Probability of getting 10 heads with a fair coin
1 10
= ( ) = 0.510
2
The probability of picking a fair coin and getting 10 heads
𝑃(𝐹𝑎𝑖𝑟 𝑐𝑜𝑖𝑛, 10 𝐻𝑒𝑎𝑑𝑠) = 0.999 × 0.510

Double-Headed Coin
Probability of picking a double-headed coin
1
=
= 0.001
1000
Probability of getting 10 heads with a double-headed coin
=1
The probability of picking a fair coin and getting 10 heads
𝑃(𝐷𝑜𝑢𝑏𝑙𝑒 − 𝐻𝑒𝑎𝑑𝑒𝑑 𝐶𝑜𝑖𝑛, 10 𝐻𝑒𝑎𝑑𝑠) = 0.001 × 1 = 0.001

Bayes Theorem
The probability of the coin being fair or double-headed using Bayes Theorem:
0.999 × 0.510
𝑃(𝐹𝑎𝑖𝑟) = ≈ 49.38%
0.999 × 0.510 + 0.001

Using complementary probability, the probability of the coin being double headed is:
𝑃(𝐷𝑜𝑢𝑏𝑙𝑒 𝐻𝑒𝑎𝑑𝑒𝑑) = 1 − 𝑃(𝐹𝑎𝑖𝑟) = 1 − 49.38% = 50.62%

Required Probability
The probability that the next coin toss is a head
=⏟ 0.5 × 49.38% + 1 × 50.62%
⏟ = 75.31%
𝑃(𝐻𝑒𝑎𝑑𝑠|𝐹𝑎𝑖𝑟) 𝑃(𝐻𝑒𝑎𝑑𝑠|𝐷𝑜𝑢𝑏𝑙𝑒 𝐻𝑒𝑎𝑑𝑒𝑑)

7 This can be found on ycombinator as a “filter” question.

P a g e 118 | 121
Aziz Manva (azizmanva@gmail.com)

H. Expected Value

Example 2.56

The expected value of number of red cars that have an accident and the claim exceeding the deductible is:
(300)(0.1)(0.9) = 27

The expected value of number of green cars that have an accident and the claim exceeding the deductible is:
(700)(0.05)(0.8) = 28

𝑆𝑢𝑐𝑐𝑒𝑠𝑠𝑓𝑢𝑙 𝑂𝑢𝑡𝑐𝑜𝑚𝑒𝑠 27 27
𝑃𝑟𝑜𝑏𝑎𝑏𝑖𝑙𝑖𝑡𝑦 = = =
𝑇𝑜𝑡𝑎𝑙 𝑂𝑢𝑡𝑐𝑜𝑚𝑒𝑠 27 + 28 55

I. Further Questions

Example 2.57
An island contains has exactly 100 residents, comprising a knight and 99 spies. The Knights always tell the
truth, but spies lie and tell the truth with equal probability. You meet a resident of the island, and ask him 8
questions that you know the answer to. He tells the truth all 8 times. What is the probability that he is a Knight?

P a g e 119 | 121
Aziz Manva (azizmanva@gmail.com)

Convert to coins.

𝐾𝑛𝑖𝑔ℎ𝑡 = 𝐷𝑜𝑢𝑏𝑙𝑒 𝐻𝑒𝑎𝑑𝑒𝑑 𝐶𝑜𝑖𝑛 = 𝐷𝐻


𝑆𝑝𝑦 = 𝐹𝑎𝑖𝑟 𝐶𝑜𝑖𝑛 = 𝐹
𝐻 8 = 𝐸𝑖𝑔ℎ𝑡 𝐻𝑒𝑎𝑑𝑠

𝑃(𝐷𝐻) = 𝑃𝑟𝑜𝑏𝑎𝑏𝑖𝑙𝑖𝑡𝑦 𝑜𝑓 𝑐ℎ𝑜𝑜𝑠𝑖𝑛𝑔 𝑑𝑜𝑢𝑏𝑙𝑒 ℎ𝑒𝑎𝑑𝑒𝑑 𝑐𝑜𝑖𝑛


𝑃(𝐹) = 𝑃𝑟𝑜𝑏𝑎𝑏𝑖𝑙𝑖𝑡𝑦 𝑜𝑓 𝑐ℎ𝑜𝑜𝑠𝑖𝑛𝑔 𝑎 𝑓𝑎𝑖𝑟 𝑐𝑜𝑖𝑛
𝑃(𝐻 8 |𝐷𝐻) = 𝑃𝑟𝑜𝑏𝑎𝑏𝑖𝑙𝑖𝑡𝑦 𝑜𝑓 𝑔𝑒𝑡𝑡𝑖𝑛𝑔 8 ℎ𝑒𝑎𝑑𝑠 𝑔𝑖𝑣𝑒𝑛 𝑡ℎ𝑎𝑡 𝑡ℎ𝑒 𝑐𝑜𝑖𝑛 𝑖𝑠 𝑑𝑜𝑢𝑏𝑙𝑒 ℎ𝑒𝑎𝑑𝑒𝑑
𝑃(𝐻 8 |𝐹) = 𝑃𝑟𝑜𝑏𝑎𝑏𝑖𝑙𝑖𝑡𝑦 𝑜𝑓 𝑔𝑒𝑡𝑡𝑖𝑛𝑔 8 ℎ𝑒𝑎𝑑𝑠 𝑔𝑖𝑣𝑒𝑛 𝑡ℎ𝑎𝑡 𝑡ℎ𝑒 𝑐𝑜𝑖𝑛 𝑖𝑠 𝑓𝑎𝑖𝑟

By Bayes Theorem:

𝑃(𝐷𝐻)𝑃(𝐻 8 |𝐷𝐻) 0.01(1)


𝑃(𝐷𝐻|𝐻 8 ) = =
𝑃(𝐹)𝑃(𝐻 |𝐹 ) + 𝑃(𝐷𝐻)𝑃(𝐻 |𝐷𝐻) 0.99 ( 1 ) + 0.01(1)
8 8
28

Example 2.58
A bag contains one marble, which is either green or blue, with equal probabilities. A green marble is put in the
bag (so there are two marbles now), and then a random marble is taken out. The marble taken out is green.
What is the probability that the remaining marble is also green. (Introduction to Probability, Blitzstein)

𝑂𝑟𝑖𝑔𝑖𝑛𝑎𝑙 𝑀𝑎𝑟𝑏𝑙𝑒: 𝐺 = 𝐺𝑟𝑒𝑒𝑛, 𝐵 = 𝐵𝑙𝑢𝑒


𝐷𝑟𝑎𝑤𝑛 𝑀𝑎𝑟𝑏𝑙𝑒: 𝑔 = 𝐺𝑟𝑒𝑒𝑛, 𝑏 = 𝐵𝑙𝑢𝑒

𝑃(𝐺) is the probability of the original marble being green.


𝑃(𝐵) is the probability of the original marble being blue.
𝑃(𝑔|𝐺) is the probability of the drawn marble being green given
that the original marble was green.
𝑃(𝑔|𝐵) is the probability of the drawn marble being green given
that the original marble was blue.

By Bayes Theorem:
𝑃(𝐺)𝑃(𝑔|𝐺) 0.5(1)
𝑃(𝐺|𝑔) = =
𝑃(𝐺)𝑃(𝑔|𝐺) + 𝑃(𝐵)𝑃(𝑔|𝐵) 0.5(1) + 0.5(0.5)

Example 2.59
There are two coasters/

Side 1 Side 2
A Brown White
B Brown Brown

If you pick one pancake, and it is brown on one side, there are three possibilities:
𝐴 − 1, 𝐵 − 1, 𝐵 − 2 ⇒ 3 𝑂𝑢𝑡𝑐𝑜𝑚𝑒𝑠

Out of these, the brown possibilities are:

P a g e 120 | 121
Aziz Manva (azizmanva@gmail.com)

𝐵 − 1, 𝐵 − 2 ⇒ 2 𝑂𝑢𝑡𝑐𝑜𝑚𝑒𝑠
Then, the probability that the other side is brown is:
2
3

2.3 Conditional Independence


A. Basics

2.60: Conditional Independence


𝑃(𝐴 ∩ 𝐵|𝐸) = 𝑃(𝐴|𝐸) ∙ 𝑃(𝐵|𝐸)

𝑃(𝐴 ∩ 𝐵|𝐸) = 𝑃(𝐴|𝐸) ∙ 𝑃(𝐵|𝐸)

Using conditional probability definitions, the RHS becomes:


𝑃(𝐴 ∩ 𝐸) 𝑃(𝐵 ∩ 𝐸)
𝑃(𝐴 ∩ 𝐵|𝐸) = ∙
𝑃(𝐸) 𝑃(𝐸)

Or, using Bayes Theorem, the RHS becomes:


𝑃(𝐸|𝐴) ∙ 𝑃(𝐴) 𝑃(𝐸|𝐵) ∙ 𝑃(𝐵)
𝑃(𝐴 ∩ 𝐵|𝐸) = ∙
𝑃(𝐸) 𝑃(𝐸)

2.4 Further Topics

61 Examples

P a g e 121 | 121

You might also like